SlideShare a Scribd company logo
1 of 54
ĐẠI HỌC THÁI NGUYÊN
TRƢỜNG ĐẠI HỌC KHOA HỌC
---------------------------
Tải tài liệu tại sividoc.com
Viết đề tài giá sinh viên – ZALO:0973.287.149-TEAMLUANVAN.COM
NGUYỄN KIM THU
MỘT SỐ PHƢƠNG PHÁP TÌM CỰC TRỊ
CỦA CÁC HÀM PHÂN THỨC
SINH BỞI SỐ TỰ NHIÊN
LUẬN VĂN THẠC SĨ TOÁN HỌC
THÁI NGUYÊN - 2018
ĐẠI HỌC THÁI NGUYÊN
TRƢỜNG ĐẠI HỌC KHOA HỌC
---------------------------
Tải tài liệu tại sividoc.com
Viết đề tài giá sinh viên – ZALO:0973.287.149-TEAMLUANVAN.COM
NGUYỄN KIM THU
MỘT SỐ PHƢƠNG PHÁP TÌM CỰC TRỊ
CỦA CÁC HÀM PHÂN THỨC
SINH BỞI SỐ TỰ NHIÊN
Chuyên ngành: Phƣơng pháp Toán sơ cấp
Mã số: 8 46 01 13
LUẬN VĂN THẠC SĨ TOÁN HỌC
NGƯỜI HƯỚNG DẪN KHOA HỌC
(Xác nhận)
GS.TSKH. Nguyễn Văn Mậu
THÁI NGUYÊN - 2018
i
Viết đề tài giá sinh viên – ZALO:0973.287.149-TEAMLUANVAN.COM
Mnc lnc
M ĐAU iii
1 Phân thfíc hfiu t v i h so nguyên 1
1.1. Tính chat cơ bản của đa thác với h so nguyên ..................... 1
1.2. Phân thác hǎu tỉ với h so nguyên và phân thác nh n giá
trị hǎu tỉ................................................................................4
1.3. Bieu dien đơn vị thành tőng của các phân so Ai C p với
mau so nguyên dương đ c bi t ............................................... 8
2 Các phương pháp giải toán cfic trị dạng phân thfíc sinh
b i so hfiu t 12
2.1. M®t so phương pháp giải bài toán cực trị của đa thác và
phân thác hǎu t với h so nguyên....................................... 12
2.1.1. Phương pháp so sánh b c hai.................................... 12
2.1.2. Phương pháp so sánh phân thác dạng b c hai trên
b c nhat....................................................................15
2.1.3. Phương pháp tìm cực trị với ràng bu®c theo tőng
các so ...............................................................................21
2.2. Sả dụng phân thác chính quy giải các bài toán cực trị
liên quan.......................................................................................27
3 M t so dạng toán liên quan 32
3.1. M®t so dạng toán cực trị trên t p so nguyên....................... 32
3.2. M®t so dạng toán ve so tự nhiên tà các đe thi Olympic ........38
KET LU N 44
ii
Viết đề tài giá sinh viên – ZALO:0973.287.149-TEAMLUANVAN.COM
TÀI LI U THAM KHẢO 45
iii
Viết đề tài giá sinh viên – ZALO:0973.287.149-TEAMLUANVAN.COM
M ĐAU
Chuyên đe ve đa thác và phân thác là m®t chuyên đe rat quan trong
ở b c trung hoc phő thông và trung hoc cơ sở. Các tính chat của đa
thác và phân thác liên quan ch t chě với các tính chat của so nguyên và
so hǎu t . M®t trong các phương pháp khảo sát đa thác và phân thác
hǎu t rat hǎu hi u là vi c sả dụng các công cụ hǎu ích tà vi c khảo
sát các tính chat so hoc của các so tự nhiên và so hǎu t .
Trong các kì thi hoc sinh giỏi toán các cap, các bài toán liên quan
tới đa thác và phân thác với h so nguyên (ta goi chung là phân thác
sinh bởi so tự nhiên) thường xuyên được đe c p. Nhǎng dạng toán này
thường được xem là thu®c loại khó can các kien thác sâu sac ve so hoc
ket hợp với các kien thác ve đa thác và phân thác thường không nam
trong chương trình chính thong của chương trình toán b c trung hoc
phő thông.
Đe đáp áng nhu cau boi dương giáo viên và boi dương hoc sinh giỏi
ve chuyên đe đa thác và phân thác với h so nguyên và h so hǎu t ,
em chon đe tài lu n văn “M®t so phương pháp tìm cực trị của các hàm
phân thác sinh bởi so tự nhiên”.
Mục tiêu của lu n văn nham h thong m®t so kien thác ve so hoc và
đa thác với h so nguyên và cung cap m®t so phương pháp tìm cực trị
của các hàm phân thác sinh bởi so tự nhiên. Tiep theo, xét các bài toán
cực trị, khảo sát phương trình, bat phương trình cùng m®t so dạng liên
quan.
Lu n văn gom phan mở đau, ket lu n và ba chương.
Chương 1. Phân thác hǎu t với h so nguyên.
Chương 2. Các phương pháp giải toán cực trị dạng phân thác sinh
iv
Viết đề tài giá sinh viên – ZALO:0973.287.149-TEAMLUANVAN.COM
bởi so hǎu t .
Chương 3. M®t so dạng toán liên quan.
Tiep theo, trong các chương đeu trình bày h thong các bài t p áp
dụng và giải các đe thi HSG quoc gia và Olympic liên quan.
1
Viết đề tài giá sinh viên – ZALO:0973.287.149-TEAMLUANVAN.COM
∈
Chương 1
Phân thfíc hfiu t v i h so nguyên
1.1. Tính chat cơ bản của đa thfíc v i h so nguyên
Trong phan này, trình bày m®t so tính chat cơ bản của đa thác với
h so nguyên.
Định nghĩa 1.1 (xem [1]-[2]) Cho L ⊂ R. Đa thác P(x) ∈ L[x] được
goi là khả quy trên L[x] neu ton tại đa thác Q(x) và T (x) cùng thu®c
L[x] với các b c lớn hơn 0 sao cho P (x) = Q(x).T (x). Trong trường hợp
ngược lại thì được goi là bat khả quy trên L[x].
Định nghĩa 1.2 (xem [1]-[2]) T p hợp tat cả các đa thác khả quy trên
L[x] được ký hi u là L∗
[x].
Tính chat 1.1 Moi đa thác P (x) ∈ R[x] với b c lớn hơn bang 2 đeu
phân tích được thành nhân tả b c nhat và nhân tả b c hai nên cũng có
the coi P(x) ∈ R∗
[x].
Định nghĩa 1.3 (xem [1]-[2]) Đa thác thu®c Z[x] được goi là đa thác
nguyên bản neu b® các h so của nó nguyên to cùng nhau (có the không
đôi m®t nguyên to cùng nhau).
Tính chat 1.2 Neu f(x) Z[x] thì ton tại duy nhat m®t đa thác
a
nguyên bản và m®t phân so toi giản , a ∈ Z, b ∈ N∗
sao cho f(x) =
a b
b
f1(x).
Bo đe 1.1 (Bő đe Gauss) Tích của hai đa thúc nguyên bản là m®t đa
thúc nguyên bản.
2
Viết đề tài giá sinh viên – ZALO:0973.287.149-TEAMLUANVAN.COM
Σ
m m
1
j!(m − j)!
1 2
α1!.α2! 1 2
1
(m − αn)!.αn!
n
ChGng minh. Cho hai đa thác nguyên bản
P (x) = anxn
an−1xn−1
+ · · · + a1x + a0
và
Q(x) = bmxm
+ bm−1xm−1
+ · · · + b1x + b0
thì P (x).Q(x) = cm+nxm+n
+ cm+n−1xm+n−1
+ · · · + c1x + c0.
Giả sả tích trên không nguyên bản thì ton tại m®t so nguyên to p là ước
chung của các h so c0, c1, . . . , cm+n.
Vì P nguyên bản nêu goi i là so nhỏ nhat mà ai không chia het cho p
và j là so nhỏ nhat sao cho bj không chia het cho p. Khi đó, xét cj+i ta
thay h so tương áng không chia het cho p, vô lý. V y tích trên nguyên
bản.
Tính chat 1.3 Neu đa thác P(x) ∈ Z[x], deg P > 1 mà không thu®c
Z∗
[x] thì nó cũng không thu®c Q∗
[x].
Định lj 1.1 (xem [1]-[2]) Cho các đa thúc f(x) = anxn
+ an−1xn−1
+
· · · + a1x + a0 ∈ Z[x], an 0, a, b là hai so nguyên khác nhau. Khi đó,
f(a) − f(b).(a − b).
Bo đe 1.2 (Khai trien Newton) Cho n và m là các so nguyên dương.
Với bat kỳ x = (x1, x2, · · · , xn) trong Rn
, ta có
(x1 + x2 + · · · + xn )m
=
|α|=m
m!
xα
, (1.1)
α!
trong đó α! = α1!α2! · · · αn! với α = (α1, α2, · · · , αn) trong Nn
, xα
=
xα1
xα2
. . . xαn
và tőng chạy qua tat cả α có the có trong Nn
thóa mãn
1 2 n
|α| = α1 + α2 + · · · + αn = m.
ChGng minh. Với n = 2 theo nhị thác Newton, ta có
(x +x )m
=
Σ m!
xj
xm−j
=
Σ m!
xα1
xα2
, α = j, α = m−j.
Giả sả đȁng thác (1.18) đã đúng cho đen n − 1. Đ t X = x1 + x2 + .. +
xn−1, α′ = (α1, α2, . . . , αn). Khi đó ta có
m
(x + x + · · · + x )m
= (X + x )m
=
Σ m!
Xm−αn
xαn
αn=0
|α|=m
j=0
2 1 2
2 n n
3
Viết đề tài giá sinh viên – ZALO:0973.287.149-TEAMLUANVAN.COM
m
Σ
1
2
1
1
1
αn=0 |α′=m−αn
=
Σ m!
xαn
.(x + x + · · · + x )m−αn
αn=0
m
(m − αn)!.αn!
n−1
=
Σ m!
xαn
.
Σ (m − αn)!
xα′
=
Σ m!
xα1
xα2 ............
xαn
.
|α|=m
Bő đe được cháng minh.
α1!α2! . . . αn! 1 2 n
Định lj 1.2 (Khai trien Taylor, xem [1]-[2]) Cho m®t đa thúc
n
f(x) = ajxj
. (1.2)
j=0
Khi đó, h so thú j của f(x) có the được bieu dien bới
a =
1
f(j)
(0), (1.3)
j
j!
trong đó f(j)
(0) úng với đạo hàm cap j tại 0
Bo đe 1.3 Cho n là m®t so nguyên dương. Ta đ¾t
g(x) =
Khi đó g(n)
(0) = n!.
ChGng minh. Ta có
x + x2
+ · · · +
n
xn
n
. (1.4)
g(x) = xn
1
1 +
2
x + · · · +
n
xn−1
n
= xn
h(x), (1.5)
trong đó
h(x) =
1
1 +
2
x + · · · +
n
xn−1
n
. (1.6)
Áp dụng công thác Leibniz
2
n
(m − αn)!.αn!
n
α1!α2! . . . αn−1!
1
4
Viết đề tài giá sinh viên – ZALO:0973.287.149-TEAMLUANVAN.COM
n
→
∈
g(n)
(x)
=
Σ
n! d n−j
xn
.
d j
h(x)
=
Do đó, ta thu được
j=0
Σ
j=0
(n − j)!j!
n!n!
(n −
j)!j!j!
dx
xj
.
dx
d j
h(x).
dx
(1.7)
g(n)
(0) = n!h(0) = n!. (1.8)
Bő đe được cháng minh.
1.2. Phân thfíc hfiu tỉ v i h so nguyên và phân thfíc
nh n giá trị hfiu tỉ
Tiep theo, ta nhac lại m®t so tính chat của phân thác hǎu tỉ với h so
nguyên và các dạng phân thác nh n giá trị hǎu tỉ trên t p so tự nhiên.
Định nghĩa 1.4 Hàm so f: R R được goi là phân thác hǎu tỉ neu
P (x)
ton tại các đa thác P(x), Q(x) sao cho f (x) =
Q (x)
(1). Khi P(x) và
Q(x) là các đa thác nguyên to cùng nhau thì (1) được goi là phân thác
hǎu tỉ chính tac.
Neu đa thác P(x) và Q(x) là các đa thác có h so hǎu tỉ thì bang
P′ (x)
vi c quy đong mau so ta sě đưa f(x) ve dạng f (x) =
Q′ (x)
, trong đó
P′(x) và Q′(x) là các đa thác có h so nguyên. Do v y phân thác hǎu
P (x)
tỉ f (x) =
Q (x)
được goi là phân thác hǎu tỉ có h so nguyên neu như
P(x), Q(x) ∈ Q[x].
Bài toán 1.1 Cho phân thác hǎu tỉ f (x) =
Cháng minh rang a, b Q.
1
1
ax + b
∈ Q với moi x ∈ Z.
1
Lài giai. Vì f (x) =
ax + b
∈ Q với moi x ∈ Z nên ax + b = với
f (x)
moi x ∈ Z. V y ax + b ∈ Q[x] hay a, b ∈ Q.
n
5
Viết đề tài giá sinh viên – ZALO:0973.287.149-TEAMLUANVAN.COM
Bài toán 1.2 Cháng minh rang neu f (x) =
C
1
ax + b
∈ Q với moi x ∈ Z
thì f(x) có dạng f(x) =
Lài giai. Vì f (x) =
Ax + B
với A, B, C thu®c Z
1
ax + b
∈ Q với moi x ∈ Z nên ta có a, b ∈ Q
⇒ a =
m
,b =
n
e
f
(m, n, e, f ∈ Z).
Khi đó 1
f(x) = m e
x +
nf
= =
mfx + ne
C
.
Ax + B
n f
ax + b
Bài toán 1.3 Cho phân thác hǎu tỉ f (x) =
cx + d
∈ Q với moi x ∈ Z.
Ax + B
Cháng minh rang f(x) có the bieu dien dưới dạng f (x) =
với A, B, C, D ∈ Z.
Cx + D
(1)
Lài giai. Neu ad − bc = 0 thì f(x) = const nên bieu dien (1) là hien
nhiên.
Xét trường hợp ad − bc 0:
Neu c = 0 thì bieu dien (1) là hien nhiên.
Neu c /= 0 thì sả dụng phân tích f(x) − f(0) =
dạng bieu dien (1).
1
ta sě được
αx + β
Nh n xét rang ket quả của bài toán cũng đúng trong trường hợp bài
toán tőng quát.
Bài toán 1.4 Cháng minh rang neu phân thác f(x) =
ax + b
cx + d
∈ Q, ∀x ∈
Z thì f(x) có dạng f(x) =
Ax + B
Cx + D
trong đó A, B, C, D là các so nguyên.
M
Q(x) − Q(1) e Q(x) −
N
Lài giai. Ta có
E
=
x − 1 cx + d
∈ Q ∀x ∈ Z nên
=
x − 1
.
Cx + D
Ta có M, N, E, C, D là các so nguyên nên de dàng suy ra đpcm.
Bài toán 1.5 Cháng minh rang neu
ax2
+ bx + c
f(x) =
dx + e
∈ Q
6
Viết đề tài giá sinh viên – ZALO:0973.287.149-TEAMLUANVAN.COM
/
D
với moi x ∈ Z thì f(x) có dạng f(x) =
thu®c Z.
Ax2
+ Bx + C
với A, B, C, D, E
Dx + E
Lài giai. Ta có
Q(x) − Q (1) px + q
= ∈ Q ∀x ∈ Z.
Suy ra
M
Q(x) −
N
x − 1
M′x + N′
cx + d
′ ′ ′′ ′′
=
x − 1
tà đó ta suy ra đpcm.
, (M, N, M , N , M , N
M′′x + N′′ ∈ Z)
Bài toán 1.6 Cháng minh rang neu
ax2
+ bx + c
f(x) =
dx + e
∈ Q
với moi x ∈ Z thì f(x) có dạng f(x) = Cx +D ho c f(x) = Ax2
+Bx+
C ∈ Z với moi x ∈ Z, trong đó C, D thu®c Z.
Lài giai.
Neu d = 0 thì ta có ngay f(x) = Ax2
+ Bx + C ∈ Z với moi x ∈ Z.
Neu d = 0 khi đó ta có
Q(x) − Q(0)
x
=
a1x + b1
dx + e
∈ Z với moi x ∈ Z.
Suy ra
Q(x) − M A1 A2
= + ∈ Z, ∀x ∈ Z,
trong đó A1, A2, D, E, M ∈ Z ⇒ A2 = 0 ⇒ A1.D nên ta có đpcm.
Bài toán 1.7 Cho phân thác hǎu tỉ f(x) =
P (x)
Q(x)
∈ Q ∀x ∈ Z,
(P(x), Q(x)) = 1. Cháng minh rang f(x) có the bieu dien được dưới
dạng phân thác của hai đa thác với h so nguyên.
ChGng minh. Giả sả:
P(x) = a0 + a1x + · · · + amxm
Q(x) = b0 + b1x + · · · + bnxn
Tại x = j (j = 0, 1, . . . , m + n) hàm f (x) nh n các giá trị hǎu tỉ tương
áng là cj. Khi đó ta có h phương trình tuyen tính với m + n + 2 ȁn:
a0, a1, . . . , am, b0, b1, . . . , bn dạng a0 + a1k + · · · + amkm
− b0ck − b1ckk −
· · · − bnckkn
= 0
trong đó k = 0, 1, . . . , m + n. Hai nghi m của h này cho ta hai c p đa
x Dx + E
7
Viết đề tài giá sinh viên – ZALO:0973.287.149-TEAMLUANVAN.COM
Q − Q
. .
i
Q
. .
. .
j
.
.
−
j
. .
p
.
.
−
p
.
.
−
p
. .
Xét hi u
x + qi
−
1 + qi
=
(x + qi) (1 + qi)
≤ 0 do đó .x + qk . ≤
−
k=j+1
thác P (x), Q(x) và P1(x), Q1(x) có tính chat
P (k) − ckQ(k) = 0, P1(k) − ckQ1(k) = 0, ∀k = 0, m + n.
Hai c p nghi m này cho ta đa thác
g(x) = P (x)Q1(x) − P1(x)Q(x), degg(x) ≤ m + n
nh n giá trị 0 tại m+n+1 điem nên g(x) ≡ 0. Do P(x) và Q(x) nguyên
to cùng nhau nên
P (x) = cP1(x); Q(x) = cQ1(x).
V y h đã cho chỉ có m®t nghi m với sự sai khác m®t thàa so, nghi m
đã nh n được là các so hǎu tỉ.
Bài toán 1.8 Cho p là m®t so nguyên dương, q ∈ [0, 1]. Giả sả x ∈
qp+1, 1 và f(x) =
p
k=1
x qk
x + qk
. Cháng minh rang |f(x)| ≤
p
k=1
1 − qk
1 + qk
Lài giai. Ta có 0 < qp+1
< qp
< · · · < q < 1. Với qj+1
≤ x ≤ qj
thì khi
i .x − qi . x − qi
i ≥ j + 1 ta có x ≥ q nên =
x + q
x + qi
.
x − qi
1 − qi
2qi
(x − 1) Q
p .x − qk .
p
k=j+1
1 − qk
.1 + qk .
Với k = 1, 2, . . . , j ta có
x − qj−(k−1)
.x + qj−(k−1) .
qj−(k−1) x
.
x + qj−(k−1)
Ta lại có
qj−(k−1) − x 1 − qk
2 qj−(k−1) − x
nên
x + qj−(k−1)
−
1 + qk
=
(1 + qk) x + qj−(k−1)
≤ 0
Tà đó ta được
k
Y
=1
x qk
.x + qk . ≤
k
Y
=1
1 − qk
.1 + qk .
k
Y
=1
x qk
.x + qk . ≤
k
Y
=1
1 qk
.1 + qk . ⇔ |f(x)| ≤
k
Y
=1
1 − qk
.1 + qk .
.
.
.
=
8
Viết đề tài giá sinh viên – ZALO:0973.287.149-TEAMLUANVAN.COM
1.3. Bieu dien đơn vị thành tong của các phân so Ai
C p v i mȁu so nguyên dương đ c bi t
Ta xét các cách bieu dien của 1 như là tőng của các phân so đơn vị.
Ví dụ, ta có the dùng 2, 3 và 6 cho
và 3, 4, 4, 8,và 24 cho
1 1 1
+ +
2 3 6
= 1 (1.9)
1 1 1
+ +
3 4 4
1 1
+ +
8 24
= 1. (1.10)
Ta đã biet rang m®t phân so tùy ý luôn có the được viet thành tőng
của các phân so đơn vị. Tiep theo, đe có các bieu dien này, ta khảo sát
lời giải của phương trình Diophantine dạng
1 1
+
x1 x2
1
+ · · · +
x
= 1, (1.11)
trong đó các ȁn xj là các so nguyên không nhat thiet phải phân bi t với
j = 1, 2, · · · , n.
Bảng 1:Tő hợp có the có của α với n = 6
α1 α2 α3 α4 α5 α6 Mau so
6 0 0 0 0 0 6! = 720
4 1 0 0 0 0 4!.2 = 48
3 0 1 0 0 0 3!.3 = 18
2 2 0 0 0 0 2!.2!.22
= 16
2 0 0 1 0 0 2!.4 = 8
1 1 1 0 0 0 2.3 = 6
1 0 0 0 5 0 5
0 3 0 0 0 0 3!.23
= 48
0 1 0 1 0 0 2.4 = 8
0 0 2 0 0 0 2!.32
= 18
0 0 0 0 0 1 6
Đe giải thích cho ket quả áng với n = 6, ta xét các tő hợp có the có
n
9
Viết đề tài giá sinh viên – ZALO:0973.287.149-TEAMLUANVAN.COM
Y
Σ Y 1
Σ
2 1 x x
n
Σ 2 n
1
2
1
của
α = (α1, α2, α3, α4, α5, α6), αj ∈ N, (1.12)
thỏa mãn đieu ki n
α1 + 2α2 + 3α3 + 4α4 + 5α5 + 6α6 = 6. (1.13)
Tiep theo, xét đại lượng
6
αj!jαj
= α1!1α1
.α!.2α2
. · · · .α6!.6α6
(1.14)
j=1
Tính tőng của các nghịch đảo của các đại lượng trong bảng so trong
Bảng 1, ta thu được
1
+
720
1 1 1 1
+ + +
48 18 16 8
1 1
+ + +
6 5
1 1
+ +
48 8
1 1
+
18 6
= 1. (1.15)
Tà ví dụ trên, ta phát bieu ket quả sau.
Định lj 1.3 (xem [1]) Với moi so nguyên dương n, ta luôn có đȁng
thúc
n
αj!jαj
= 1, (1.16)
α∈Sn j=1
trong đó tőng trên Sn chạy qua tat cả α = (α1, α2, · · · , αn) có the có
trong Nn
thóa mãn đieu ki n
ChGng minh. ta có
n
jαj = n. (1.17)
j=1
x1 + x2
+ · · · +
m
n
n
|α|=m
m!
xα1
.
α!
n
1 α2
2 2 . · · · .
1 αn
n n
= m!
Σ
Y 1
x
jαj
.
|α|=m j=1
αj!jαj j
(1.18)
Cho m = n và xj = t với j = 1, 2, · · · , n dan đen
t + t2
+ · · · +
n
tn
n
= n!
|
Σ
α|=n
Y
j=1
1
αj!jαj
tjαj
. (1.19)
x
1
2
1
n
=
10
Viết đề tài giá sinh viên – ZALO:0973.287.149-TEAMLUANVAN.COM
Σ Y 1
Σ Y 1
So sánh h so của tn
trong cả hai đong nhat thác. ta thu được n! đoi
với ve trái. M t khác h so tn
trong ve phải là tőng của tat cả so hạng
với tn
mà được viet bởi
n
, (1.20)
αj!jαj
α∈Sn j=1
trong đó tőng chạy qua tat cả α có the có trong Sn xác định bởi
Sn = {α ∈ Nn
; α1 + 2α2 + · · · + nαn = n} . (1.21)
Do đó, ta thu được
n
αj!jαj
= 1 (1.22)
α∈Sn j=1
và ta có đieu phải cháng minh.
Bài toán 1.9 Xét phương trình vô định sau đây
α1 + 2α2 + 3α3 + 4α4 = 4.
Tính tőng nghịch đảo các đại lượng
Πα = α1!1α1
.α2!2α2
.α3!3α3
.α4!4α4
,
trong đó α = (α1, α2, α3, α4) là các nghi m nguyên không âm phân bi t
của phương trình đã cho.
Lài giai. De thay rang phương trình đã cho có các nghi m nguyên
không âm chỉ là
α = (α1, α2, α3, α4) = (4, 0, 0, 0), (0, 0, 0, 1), (1, 0, 1, 0), (0, 2, 0, 0), (2, 1, 0, 0).
Tiep theo, tính các đại lượng
Πα = α1!1α1
.α2!2α2
.α3!3α3
.α4!4α4
, α = (α1, α2, α3, α4).
Ta có
Π1 = 4!14
.0!20
.0!30
.0!40
= 24,
Π2 = 0!10
.0!20
.0!30
.1!41
= 4,
11
Viết đề tài giá sinh viên – ZALO:0973.287.149-TEAMLUANVAN.COM
Suy ra
Π3 = 1!11
.0!20
.1!31
.0!40
= 3,
Π4 = 0!10
.2!22
.0!30
.0!40
= 8,
Π5 = 2!12
.1!21
.0!30
.0!40
= 4.
1 1
+
Π1 Π2
1 1
+ +
Π3 Π4
1 1 1 1 1
= + + + +
24 4 3 8 4
= 1.
Bài toán 1.10 Xét phương trình vô định sau đây
α1 + 2α2 + 3α3 + 4α4 + 5α5 = 5.
Tính tőng nghịch đảo các đại lượng
Πα = α1!1α1
.α2!2α2
.α3!3α3
.α4!4α4
.α5!5α5
,
trong đó α = (α1, α2, α3, α4, α5) là các nghi m nguyên không âm phân
bi t của phương trình đã cho.
Lài giai. De thay rang phương trình đã cho có các nghi m nguyên
không âm chỉ là
α = (α1, α2, α3, α4, α5) = (5, 0, 0, 0, 0), (0, 0, 0, 0, 1), (0, 1, 1, 0, 0),
(1, 2, 0, 0, 0), (2, 0, 1, 0, 0), (1, 0, 0, 1, 0), (3, 0, 1, 0, 0).
Tiep theo, ta tính các đại lượng
Πα = α1!1α1
.α2!2α2
.α3!3α3
.α4!4α4
.α5!5α5
, α = (α1, α2, α3, α4, α5).
Ta có
Suy ra
Π1 = 5!15
.0!20
.0!30
.0!40
.0!.50
= 120,
Π2 = 0!10
.0!20
.0!30
.1!40
.1!51
= 5,
Π3 = 0!11
.1!21
.1!31
.0!40
.0!50
= 6,
Π4 = 1!11
.2!22
.0!30
.0!40
.0!50
= 8,
Π5 = 2!12
.0!20
.1!31
.0!40
.0!50
= 4,
Π6 = 1!11
.0!20
.0!30
.1!41
.0!50
= 4,
Π7 = 3!13
.0!20
.1!31
.0!40
.0!50
= 18.
1 1
+
Π1 Π2
1 1 1
+ + +
Π3 Π4 Π5
1 1
+ +
Π6 Π7
1 1
= + +
120 5
1 1 1
+ + +
6 8 4
1 1
+
4 18
= 1.
12
Viết đề tài giá sinh viên – ZALO:0973.287.149-TEAMLUANVAN.COM
Chương 2
Các phương pháp giải toán cfic trị
dạng phân thfíc sinh b i so hfiu t
2.1. M t so phương pháp giải bài toán cfic trị của đa
thfíc và phân thfíc hfiu t v i h so nguyên
Trong phan này ta khảo sát lớp phân thác hǎu t với h so nguyên.
Đe giải các bài toán cực trị tương áng ta xét m®t so phương pháp đ c
thù như so sánh b c hai và so sánh dạng dạng phân thác b c hai trên
b c nhat tőng quát.
2.1.1. Phương pháp so sánh b c hai
Ta đã biet, đoi với dạng tam thác b c hai
f(x) = ax2
+ bx + c, a > 0.
ta luôn có
f(x) ≥ f(y) + (2ax + b)(x − y), ∀x, y ∈ R. (∗)
Th t v y, ta thay
f(x) ≥ f(y) + (2ax + b)(x − y) ⇔ a(x − y)2
≥ 0, ∀x, y ∈ R.
Sả dụng phương pháp so sánh b c hai, ta giải quyet được các dạng
toán cực trị ở các nút nguyên trên trục toa đ® mà các phương pháp
truyen thong không thực hi n được.
13
Viết đề tài giá sinh viên – ZALO:0973.287.149-TEAMLUANVAN.COM
Bài toán 2.1 Xét các so tự nhi n a, b, c thỏa mãn đieu ki n
a ≥ 9
a + b ≥ 11
a + b + c ≥ 12.
Tìm giá trị nhỏ nhat của bieu thác
M = a2
+ b2
+ c2
.
Lài giai. Áp dụng bat đȁng thác (*) áng với f(x) = x2
, ta thu được
a2
≥ 92
+ 2 × 9(a − 9), ∀a ∈ N.
Tương tự, ta cũng có
b2
≥ 22
+ 2 × 2(b − 2), ∀b ∈ N
và
Tà đó, suy ra
c2
≥ 1 + 2(c − 1), ∀c ∈ N.
M = a2
+ b2
+ c2
≥ 92
+ 22
+ 1 + 2[9(a − 9) + 2(b − 2) + (c − 1)].
M t khác, theo giả thiet thì
9(a−9)+2(b−2)+(c−1) = (a−9+b−2+c−1)+(a−9+b−2)+7(a−9) ≥ 0.
Suy ra
M ≥ 92
+ 22
+ 1 = 86
nên min M = 86 khi a = 9, b = 2, c = 1.
Bài toán 2.2 Xét các so tự nhi n a, b, c thỏa mãn đieu ki n
a ≥ 9
a + b ≥ 11
a + b + c ≥ 12.
Tìm giá trị nhỏ nhat của bieu thác
M = f(a) + f(b) + f(c),
trong đó f(x) = 3x2
+ 2x − 10.
14
Viết đề tài giá sinh viên – ZALO:0973.287.149-TEAMLUANVAN.COM
Lài giai. Áp dụng bat đȁng thác (*), ta có
f(a) ≥ f(9) + (6 × 9 + 2)(a − 9), ∀a ∈ N.
Tương tự, ta cũng có
f(b) ≥ f(2) + (6 × 2 + 2)(b − 2), ∀a ∈ N.
và
Tà đó, suy ra
f(a) ≥ f(9) + (6 × 1 + 2)(c − 1), ∀a ∈ N.
M = f(a)+f(b)+f(c) ≥ f(9)+f(2)+f(1)+[(6×9+2)(a−9)+(6×2+2)(b−2)+(6×1+2
M t khác, theo giả thiet thì
(6×9+2)(a−9)+(6×2+2)(b−2)+(6×1+2)(c−1) = 8(a−9+b−2+c−1)+6(a−9+b−2)+
Suy ra
M ≥ f(9) + f(2) + f(1) = 237
nên min M = 237 khi a = 9, b = 2, c = 1.
Bài toán 2.3 Xét các so tự nhi n a, b, c thỏa mãn đieu ki n
c ≤ b ≤ a ≤ 9
a + b ≤ 11
a + b + c ≤ 12.
Tìm giá trị lớn nhat của bieu thác
M = a2
+ b2
+ c2
.
Lài giai. Áp dụng bat đȁng thác (*) áng với f(x) = x2
, ta thu được
92
≥ a2
+ 2 × a(9 − a), ∀a ∈ N.
Tương tự, ta cũng có
22
≥ b2
+ 2 × b(2 − b), ∀a ∈ N.
và
1 ≥ c2
+ 2 × 1(1 − c), ∀a ∈ N.
15
Viết đề tài giá sinh viên – ZALO:0973.287.149-TEAMLUANVAN.COM
≤ ∞
Tà đó, suy ra
M = a2
+ b2
+ c2
≤ 92
+ 22
+ 1 − 2[a(9 − a) + b(2 − b) + c(1 − c)].
M t khác, theo giả thiet thì
a(9−a)+b(2−b)+c(1−c) = (a−b)(9−a)+(b−c)(9−a+2−b)+c(9−a+2−b+1−c) ≥ 0.
Suy ra
M ≤ 92
+ 22
+ 1 = 86
nên max M = 86 khi a = 9, b = 2, c = 1.
2.1.2. Phương pháp so sánh phân thfíc dạng b c hai trên b c
nhat
Trước het ta xét bài toán ve xác định các khoảng (α, β) với (−∞ ≤
α < β + ) sao cho áng với moi hàm so b c hai g(x) = ax2
+ bx +
g(x)
c, a > 0, hàm so có dạng f(x) =
thác
x − d
, g(d) /= 0, ta đeu có bat đȁng
ho c
f(u)
f′(v)
≥
f(u)
f (v)
f′(v)
+ u − v, ∀u, v ∈ (α, β), u
f(v)
v (∗∗)
f′(v)
≤
f′(v)
+ u − v, ∀u, v ∈ (α, β), u =
/ v (∗ ∗ ∗)
và xét các áp dụng liên quan.
Ý tưởng chính của phương pháp là sả dụng phép so sánh phân thác
b c hai trên b c nhat tőng quát.
Đe ý rang, trong hau het các dạng toán cực trị có ràng bu®c theo
tőng ho c tích trên t p so tự nhiên thì dau đȁng thác xảy ra không ở
b® so bang nhau. Vì v y, ta can đen m®t công cụ hǎu hi u là định lý
Karamata cho các b® so được sap thá tự.
Trước het, ta nhac lại bat đȁng thác Karamata [1-3].
Định lj 2.1 (Bat đȁng thúc Karamata) Xét hai dãy so {xk, yk ∈ (a, b), k =
1, 2, . . . , n}, thóa mãn đieu ki n
x1 ≥ x2 ≥ · · · ≥ xn, y1 ≥ y2 ≥ · · · ≥ yn
16
Viết đề tài giá sinh viên – ZALO:0973.287.149-TEAMLUANVAN.COM
x1 + x2 ≥ y1 + y2
và
x1 ≥ y1
. . . . . .
x1 + x2 + · · · + xn−1 ≥ y1 + y2 + · · · + yn−1
x1 + x2 + · · · + xn = y1 + y2 + · · · + yn
Khi đó, úng với moi hàm so loi khả vi ϕ(x)(ϕ′′(x) ≥ 0) trên (α, β),
ta đeu có
ϕ(x1) + ϕ(x2) + · · · + ϕ(xn) ≥ ϕ(y1) + ϕ(y2) + · · · + ϕ(yn).
Đȁng thúc xảy ra khi xi = yi, i = 1, 2, . . . n.
Ta cũng phát bieu tương tự đoi với hàm so lõm bang cách đői chieu dau
bat đȁng thác.
Nh n xét 2.1 Neu hàm so y = ϕ(x) liên tục và có đạo hàm cap 2 trên
(α, β) và ϕ′′(x) ≥ 0, ∀x ∈ (α, β) thì
ϕ(x) ≥ ϕ′(x0)(x − x0) + ϕ(x0), ∀x, x0 ∈ (α, β).
Neu ϕ′′(x) ≤ 0, ∀x ∈ I(a; b) thì
ϕ(x) ≤ ϕ′(x0)(x − x0) + ϕ(x0), ∀x, x0 ∈ (α, β).
Đȁng thác trong hai bat đȁng thác trên xảy ra khi x = x0.
Nh n xét rang đe x − d 0 với moi x ∈ (α, β) thì d ∈
/ (α, β). Đieu này
tương đương với ho c d ≤ α ho c d ≥ β.
Tà đó suy ra
H quả 2.1 Đieu ki n can đe có bat đȁng thác (∗∗) với moi u, v ∈ (α, β)
và u /= v là ho c d ≤ α ho c d ≥ β.
Tiep theo, ta xác định đieu ki n đủ đe có (∗∗) .
Ta có
f(x) = ax + ad + b +
và
g(d)
x − d
f′(x) = a −
g(d)
, f′′(x) =
2g(d)
. (2.1)
(x − d)2 (x − d)3
17
Viết đề tài giá sinh viên – ZALO:0973.287.149-TEAMLUANVAN.COM
a
Trường hợp 1. Xét g(d) < 0. Khi đó f′(x) > 0 với moi x
f′′(x) > 0 khi x > d và f′′(x) < 0 khi x < d.
V y nên khi x < d thì f′′(x) < 0 và f′(x) > 0.
d. Ta có
Theo Bő đe cơ bản, ta có (∗ ∗ ∗) luôn đúng áng với moi u, v < d,
u /= v.
Khi x > d thì f′′(x) > 0 và f′(x) > 0. và (∗∗) luôn đúng với moi
u, v > d, u /= v.
Trường hợp 2. Xét g(d) > 0. Khi đó f′′(x) < 0 khi x > d và f′′(x) > 0
khi x < d.
Xét phương trình
f′(x) = 0 ⇔ x1,2 = d ∓
r
g(d)
, x < d < x2.
Khi đó f′(x) > 0 khi x > x2 ho c x < x1 và f′(x) < 0 ⇔ x1 < x < x2.
V y nên
- khi x < β ≤ x1 thì f′(x) > 0 và f′′(x) > 0. Khi đó (∗∗) đúng với moi
u, v ∈ (α, β),
- khi x > α ≥ x2 thì f′(x) > 0 và f′′(x) < 0. Khi đó (∗ ∗ ∗) đúng với
moi u, v ∈ (α, β),
- khi x ∈ (x1, d) thì f′(x) < 0 và f′′(x) > 0. Khi đó (∗ ∗ ∗) đúng với moi
u, v ∈ (x1, d),
- khi x ∈ (d, x2) thì f′(x) < 0 và f′′(x) < 0. Khi đó (∗∗) đúng với moi
u, v(d, x2).
V y ta có the phát bieu các ket quả nh n được như sau.
g(x)
Định lj 2.2 Giả sủ f(x) = với g(x) = ax2
+ bx + c, a > 0 và
g(d) < 0.
x − d
Khi đó, đieu ki n can và đủ (đoi với (α, β)) đe có bat đȁng thúc (∗∗)
là α ≥ d và đieu ki n can và đủ (đoi với (α, β)) đe có bat đȁng thúc
(∗ ∗ ∗) là β ≤ d.
Tương tự, ta cũng có ket quả sau
g(x)
Định lj 2.3 Giả sủ f(x) = với g(x) = ax2
+ bx + c, a > 0 và
g(d) < 0
x − d
Khi đó, đieu ki n can và đủ (đoi với (α, β)) đe có bat đȁng thúc (∗∗)
là α ≤ x1 ho¾c d ≤ α < β < x2, tương tự, đieu ki n can và đủ (đoi với
1
18
Viết đề tài giá sinh viên – ZALO:0973.287.149-TEAMLUANVAN.COM
Σ − Σ −
≥
r
Σ − Σ −
≤
− − −
(α, β)) đe
x1,2 = d ∓
có bat đȁng thúc (∗ ∗ ∗) là α ≥ x2 ho¾c x1 ≤ α < β ≤ d với
g(d)
.
a
Tà các định lý 2.2-2.3, ta phát bieu các h quả đe sả dụng trong các
bài toán áp dụng ở phan sau khi b® so gom các so nguyên.
g(x)
H quả 2.2 Giả sủ f(x) =
x − d
với g(x) = ax2
+ bx + c, a > 0 có
g(d) < 0 và α ≥ d ho¾c g(d) > 0 và α ≤ x1 ho¾c d ≤ α < β < x2.
Khi đó, với moi b® so u1, u2, u1 ∈ (α, β) và v1, v2, v3 ∈ (α, β) sao cho
u1 + u2 + u3 = v1 + v2 + v3, ta đeu có
g(uk)
3
g(vk)
3
uk d
g(d)
vk d .
g(d)
k=1 a −
(v — d)2
k=1 a −
(v — d)2
g(x)
H quả 2.3 Giả sủ f(x) =
x − d
với g(x) = ax2
+ bx + c, a > 0 có
g(d) < 0 và β ≤ d. ho¾c g(d) > 0 và α ≥ x2 ho¾c x1 ≤ α < β ≤ d với
g(d)
x1,2 = d ∓
a
.
Khi đó, với moi b® so u1, u2, u1 ∈ (α, β) và v1, v2, v3 ∈ (α, β) sao cho
u1 + u2 + u3 = v1 + v2 + v3, ta đeu có
g(uk)
3
g(vk)
3
uk d
g(d)
vk d .
g(d)
k=1 a −
(v — d)2
k=1 a −
(v — d)2
Bài toán 2.4 Xét các so x, y, z ≥ 1 có x + y + z = 10. Cháng minh bat
đȁng thác
9(x 1)2
+
4(x + 1)
Lài giai. Xét hàm so
4(y 1)2
+
3(y + 1)
9(z 1)2
8(z + 1)
≥
157
.
30
(x − 1)2
4
Ta có
f(x) = = x − 3 + .
x + 1 x + 1
f′
(x) =
(x − 1)(x + 3)
(x + 1)2
≥ 0, ∀x ≥ 1
r
k k
k k
19
Viết đề tài giá sinh viên – ZALO:0973.287.149-TEAMLUANVAN.COM
9
,
−
−
−
−
−
−
4
− −
= x − 3 + .
và
f′′(x) =
8
(x + 1)3 > 0, ∀x ≥ 1.
Theo h quả 2.2, ta có
f(u)
f′(v)
≥
f (v)
f′(v)
+ u − v, ∀u, v ∈ [1, +∞), u =
/ v (∗∗)
Chon 3 so v ∈ [1, +∞) sao cho f′(v) ∈
,4
;
3
;
8
và có tőng bang 10.
9
Ta thu được v1 = 2, v2 = 3, v3 = 5. Tiep theo, the vào (∗∗), ta được
hay
9(x 1)2
4(x + 1)
≥
4(y 1)2
3(y + 1)
≥
9(z 1)2
8(z + 1)
≥
9(x 1)2
4(x + 1)
≥
4(y 1)2
3(y + 1)
≥
9(z 1)2
8(z + 1)
≥
f(2)
f′(2)
+ x − 2, ∀x ∈ [1, +∞)
f(3)
f′(3)
+ y − 3, ∀y ∈ [1, +∞)
f(5)
f′(5)
+ z − 2, ∀z ∈ [1, +∞)
1/2
5/9
+ x − 2, ∀x ∈ [1, +∞)
1
3/4
+ y − 3, ∀y ∈ [1, +∞)
8/3
8/9
+ z − 2, ∀z ∈ [1, +∞)
C®ng các ve tương áng ta thu được
9(x − 1)2
4(y − 1)2
9(z − 1)2
1/2 1 8/3 157
+
4(x + 1)
đpcm.
+
3(y + 1)
8(z + 1)
≥
+ + = ,
5/9 3/4 8/9 30
Bài toán 2.5 Xét các so x, y, z ∈ (−1, 1) có x + y + z = 1. Tìm giá trị
lớn nhat của bieu thác
M :=
9(x − 1)2
5(x + 1)
4(y 1)2
+
5(y + 1)
49(z 1)2
+ .
95(z + 1)
Lài giai. Xét hàm so
f(x) =
(x − 1)2
4
x + 1 x + 1
20
Viết đề tài giá sinh viên – ZALO:0973.287.149-TEAMLUANVAN.COM
5 5 95
1 1 1
Ta có
và
f′
(x) =
(x − 1)(x + 3)
(x + 1)2 < 0, ∀x ∈ (−1, 1)
f′′(x) =
8
(x + 1)3 > 0, ∀x ∈ (−1, 1).
Theo h quả 2.3, ta có
f(u)
f′(v)
≤
f (v)
f′(v)
+ u − v, ∀u, v ∈ (−1, 1), u v (∗ ∗ ∗)
Chon 3 so v ∈ (−1, 1) sao cho f′(v) ∈
,
−
9
; −
4
; −
49,
và có tőng
bang 1. Ta thu được v1 =
ta được
2
,v2 =
3
,v3 =
6
. Tiep theo, the vào (∗ ∗ ∗),
9(x − 1)2
f(1/2) 1
5(x + 1)
≥ f′(1/2)
+ x −
2
, ∀x ∈ (−1, 1)
4(y − 1)2
f(1/3) 1
5(y + 1)
≥ f′(1/3)
+ y −
3
, ∀y ∈ (−1, 1)
49(z − 1)2
f(1/6) 1
hay
95(z + 1)
≥ f′(1/6)
+ z −
6
, ∀z ∈ (−1, 1)
9(x − 1)2
9 1
—
5(x + 1)
≥ −
30
+ x −
2
, ∀x ∈ (−1, 1)
4(y − 1)2
4 1
—
5(y + 1)
≥ −
15
+ y −
3
, ∀y ∈ (−1, 1)
49(z − 1)2
35 1
hay
—
95(z + 1)
≥ −
114
+ z −
6
, ∀z ∈ (−1, 1)
9(x − 1)2
9 1
5(x + 1)
≤ 30
− x +
2
, ∀x ∈ (−1, 1)
4(y − 1)2
4 1
5(y + 1)
≤ 15
− y +
3
, ∀y ∈ (−1, 1)
49(z − 1)2
35 1
95(z + 1)
≤ 114
− z +
6
, ∀z ∈ (−1, 1)
21
Viết đề tài giá sinh viên – ZALO:0973.287.149-TEAMLUANVAN.COM
≤
− − −
C®ng các ve tương áng ta thu được
9(x 1)2
M =
5(x + 1)
4(y 1)2
+
5(y + 1)
49(z 1)2
+
95(z + 1)
9 4 35
V y max M =
9 4
+ +
30 5
≤
35
114
+ + .
30 5 114
đạt được khi x =
1
, y =
2
1 1
, z = .
3 6
2.1.3. Phương pháp tìm cfic trị v i ràng bu c theo tong các
so
Đoi với các bài toán tìm cực trị của m®t so bieu thác với ràng bu®c
theo tőng của các so tự nhiên, ta thường sả dụng phương pháp làm đeu.
Bài toán 2.6 Cho các so nguyên dương a, b, c có a + b + c = 100. Tìm
giá trị lớn nhat của
M = abc.
Lài giai. đây, vì ba so a, b, c là các so nguyên dương nên tà giả thiet
suy ra chúng không the bang nhau.
Giả sả a ≥ b ≥ c thì a ≥ 34. Áp dụng bat đȁng thác AM-GM cho b®
so 33a, 34b, 34c, ta thu được
√
3
33a.34b.34c
33a + 34b + 34c
3
=
34 × 100 − a
3
34 × 100 − 34
3
= 33 × 34.
Suy ra M ≤ 34 × 332
.
V y max M = 34 × 332
.
Tiep theo, ta xét bài toán tìm cực tieu.
Bài toán 2.7 Cho các so nguyên dương a, b, c có a + b + c = 100. Tìm
giá trị nhỏ nhat của
M = abc.
Lài giai. Ta cháng minh min M = 98.
Giả sả a ≥ b ≥ c thì a ≥ 34.
Neu c ≥ 2 thì b ≥ 2 nên abc ≥ 34 × 2 × 2 > 98.
≤
22
Viết đề tài giá sinh viên – ZALO:0973.287.149-TEAMLUANVAN.COM
a(c + a) b(a + b)
Xét c = 1. Khi đó a + b = 99 và a ≥ 50.
Neu b ≥ 2 thì abc ≥ 50 × 2 × 1 > 98.
Khi b = 1 thì a = 98 và abc = 98.
V y nên min M = 98.
Bài toán 2.8 Cho các so nguyên dương a, b, c có a + b + c = 9. Cháng
minh rang
ab bc ca 3
Lài giai. Đ t
c(c + b)
+
a(a + c)
+
b(a + b)
≥
2
.
ab
M =
c(c + b)
bc
+
a(a + c)
ca
+ .
b(a + b)
Theo bat đȁng thác AM-GM, ta có
r
ab
+
r
bc
+
r
ac 2
c a
=
s
ab
b
√
b + c +
s
bc
√
c + a +
r
ac √
a + b
2
≤
h ab
+
bc
+
ca i
[2(a + b + c)] = M[2(a + b + c)].
M t khác cũng theo bat đȁng thác AM-GM, thì
(a + b + c)2
≥ 3(ab + bc + ca)
nên r
ab
+
r
bc
+
r
ac 2
≥ 3(a + b + c).
Suy ra M ≥
c a b
3
, hay
2
ab bc ca 3
c(c + b)
+
a(a + c)
+
b(a + b)
≥
2
.
Dau đȁng thác xảy ra khi a = b = c = 3.
Bài toán 2.9 Cho x, y, z là các so nguyên thỏa mãn đieu ki n x+y+z =
6. Cháng minh rang
8x
+ 8y
+ 8z
4x+1 + 4y+1 + 4z+1
≥ 1.
c(b + c)
c(c + b) a(a + c) b(a + b)
23
Viết đề tài giá sinh viên – ZALO:0973.287.149-TEAMLUANVAN.COM
3
3 2
3
Lài giai. Đ t 2x
= a, 2y
= b, 2z
= c, (a, b, c > 0) ⇒ abc = 64. Đieu can
cháng minh trở thành a3
+ b3
+ c3
≥ 4a2
+ 4b2
+ 4c2
.
Áp dụng bat đȁng thác AM-GM ta được:
a3
+ b3
+ c3
≥ 3.abc = 3.64
a3
+ a3
+ 64 ≥ 12a2
b3
+ b3
+ 64 ≥ 12b2
c3
+ c3
+ 64 ≥ 12c2
3 3 2 2 2
⇒ 3(a + b + c ) + 3.64 ≥ 3.64 + 12(a + b + c )
3 2 2
⇔ a + b + c ≥ 4(a + b + c ) (đieu phải cháng minh).
Dau đȁng thác xảy ra khi a = b = c = 4 khi đó x = y = z = 2.
Bài toán 2.10 Với ba so nguyên dương a, b, c thỏa mãn đieu ki n a +
b + c = 9. Cháng minh rang
a b c 1
9 + b2
+
9 + c2
+
9 + a2
≥
2
.
Lài giai. Áp dụng bat đȁng thác AM-GM, ta có
9a ab2
ab2
ab
9 + b2
= a −
9 + b2
≥ a −
6b
= a −
6
.
Tương tự, ta có
9b bc
9 + c2
≥ b −
6
9c ac
9 + a2
≥ c −
6
.
C®ng ve với ve ba bat đȁng thác trên ta có
9a 9b 9c 1 ab + bc + ca
9 + b2
+
9 + c2
+
9 + a2
≥ a+b+c−
6
(ab+bc+ca) = 9−
6
.
Ta lại có
Do đó
ab + bc + ca ≤
(a + b + c)2
= 27.
3
9a 9b 9c 1 9
9 + b2
+
9 + c2
+
9 + a2
≥ 9 −
6
(ab + bc + ca) ≥
2
.
a b c 1
⇒
9 + b2
+
9 + c2
+
9 + a2
≥
2
.
Đȁng thác xảy ra khi và chỉ khi a = b = c = 3.
24
Viết đề tài giá sinh viên – ZALO:0973.287.149-TEAMLUANVAN.COM
Σ
−
−
−
Bài toán 2.11 Cho m,n là hai so nguyên dương với m ≥ n ≥ 2. Giả
sả x1; x2; . . . xn là các so nguyên dương có tőng bang m. Tìm giá trị lớn
1
nhat và giá trị nhỏ nhat của bieu thác S = n .
kxk
k=1
Lài giai. Vì có hǎu hạn b® so (x1, x2, . . . xn) thỏa mãn đieu ki n của
bài toán nên ta có hǎu hạn giá trị của S. Do đó ton tại giá trị nhỏ nhat
N và giá trị lớn nhat L của S.
Ta cháng minh L đạt được khi xn = m + 1 − n, x1 = x2 = · · · =
xn−1 = 1.
Th t v y, giả sả L đạt được với b® so (x1, x2, . . . xn) mà xn ≤ m+1−n.
Khi đó ton tại xi > 1 (1 ≤ i ≤ n − 1). Đ t
x′
i = xi − 1, x′
n = xn + 1, x′
k = xk (k /= n; i; 1 ≤ k ≤ n − 1) .
Khi đó b® so x1
′
; x2
′
; . . . x′
n thỏa mãn đieu ki n của bài toán. Do
ix′
i + nx′
n
suy ra
= i (xi − 1) + n (xn + 1) = ixi + nxn + n − i > ixi + nxn
n n
Σ
kx′
k >
Σ
kxk.
k=1
Đieu này trái với giả thiet của L.
V y
k=1
L =
n(n − 1)
+ n (m + 1 n) =
2
2mn + n n2
.
2
Cháng minh tương tự, ta có N đạt được khi
x1 = m + 1 − n, x2 = x3 = · · · = xn = 1
hay
(n2
+ 2m n)
N = .
2
Bài toán 2.12 Cho x, y ∈ N∗
; x + y = 2n + 1 với n là hang so nguyên
dương cho trước. Tìm giá trị lớn nhat và giá trị lớn nhat của bieu thác
P = x!y!
Lài giai. Ta cháng minh bieu thác P đạt giá trị lớn nhat L = (2n)!
và giá trị nhỏ nhat N = n!(n + 1)!
25
Viết đề tài giá sinh viên – ZALO:0973.287.149-TEAMLUANVAN.COM
−
−
Th t v y giả sả N đạt được với b® so (x; y) thỏa mãn đieu ki n của
bài toán áng với x < n.
Đ t
x′ = x + 1; y′ = y − 1
Khi đó, ta có b® so(x′; y′) thỏa mãn đieu ki n của bài toán.
Và x′!y′! = (x + 1)!(y 1)! = x!y!
x + 1
y
< x!y! = N vì x + 1 < n + 1 < y
Trái với giả thiet của N.
V y giá trị nhỏ nhat N = n!(n + 1)!
Giả sả L đạt được với b® so (x; y) thỏa mãn đieu ki n của bài toán
mà 1 < x < y.
Đ t
x′ = x − 1; y′ = y + 1.
Khi đó, ta có b® so (x′; y′) thỏa mãn đieu ki n của bài toán và
x′!y′! = (x 1)!(y + 1)! = x!y!
y + 1
x
> x!y! = L
do y + 1 > y > x, trái với giả thiet.
V y giá trị lớn nhat L = (2n)!
Bài toán 2.13 Xét phương trình vô định sau đây
α1 + 2α2 + 3α3 + 4α4 = 4.
Tìm giá trị nhỏ nhat của tőng các nghịch đảo của các đại lượng
Πα = α1!1α1
.α2!2α2
.α3!3α3
.α4!4α4
,
trong đó α = (α1, α2, α3, α4) là các nghi m nguyên không âm phân bi t
của phương trình đã cho.
Lài giai. De thay rang phương trình đã cho có các nghi m nguyên
không âm chỉ là
α = (α1, α2, α3, α4) = (4, 0, 0, 0), (0, 0, 0, 1), (1, 0, 1, 0), (0, 2, 0, 0), (2, 1, 0, 0).
Tiep theo, tính các đại lượng
Πα = α1!1α1
.α2!2α2
.α3!3α3
.α4!4α4
, α = (α1, α2, α3, α4).
26
Viết đề tài giá sinh viên – ZALO:0973.287.149-TEAMLUANVAN.COM
Ta có
Suy ra
Π1 = 4!14
.0!20
.0!30
.0!40
= 24,
Π2 = 0!10
.0!20
.0!30
.1!41
= 4,
Π3 = 1!11
.0!20
.1!31
.0!40
= 3,
Π4 = 0!10
.2!22
.0!30
.0!40
= 8,
Π5 = 2!12
.1!21
.0!30
.0!40
= 4.
T :=
1 1
+
Π1 Π2
1 1
+ +
Π3 Π4
1 1
= +
24 4
1 1 1
+ + +
3 8 4
= 1.
V y nên min T = 1.
Bài toán 2.14 Xét phương trình vô định sau đây
α1 + 2α2 + 3α3 + 4α4 + 5α5 = 5.
Tìm giá trị nhỏ nhat của tőng các nghịch đảo của các đại lượng
Πα = α1!1α1
.α2!2α2
.α3!3α3
.α4!4α4
.α5!5α5
,
trong đó α = (α1, α2, α3, α4, α5) là các nghi m nguyên không âm phân
bi t của phương trình đã cho.
Lài giai. De thay rang phương trình đã cho có các nghi m nguyên
không âm chỉ là
α = (α1, α2, α3, α4, α5) = (5, 0, 0, 0, 0), (0, 0, 0, 0, 1), (0, 1, 1, 0, 0),
(1, 2, 0, 0, 0), (2, 0, 1, 0, 0), (1, 0, 0, 1, 0), (3, 0, 1, 0, 0).
Tiep theo, ta tính các đại lượng
Πα = α1!1α1
.α2!2α2
.α3!3α3
.α4!4α4
.α5!5α5
, α = (α1, α2, α3, α4, α5).
Ta có
Π1 = 5!15
.0!20
.0!30
.0!40
.0!.50
= 120,
Π2 = 0!10
.0!20
.0!30
.1!40
.1!51
= 5,
Π3 = 0!11
.1!21
.1!31
.0!40
.0!50
= 6,
Π4 = 1!11
.2!22
.0!30
.0!40
.0!50
= 8,
27
Viết đề tài giá sinh viên – ZALO:0973.287.149-TEAMLUANVAN.COM
Σ
Σ Σ
∈ ∈
Π5 = 2!12
.0!20
.1!31
.0!40
.0!50
= 4,
Π6 = 1!11
.0!20
.0!30
.1!41
.0!50
= 4,
Π7 = 3!13
.0!20
.1!31
.0!40
.0!50
= 18.
Suy ra
1
S :=
Π1
1 1 1
+ + +
Π2 Π3 Π4
1 1 1
+ + +
Π5 Π6 Π7
1
= +
120
1 1 1
+ +
5 6 8
1 1 1
+ + +
4 4 18
= 1.
V y nên min S = 1.
2.2. Sfi dnng phân thfíc chính quy giải các bài toán
cfic trị liên quan
Định nghĩa 2.1 Hàm f(x) xác định trên t p các so thực dương, không
đong nhat với 0 được goi là phân thác chính quy neu f(x) =
Σ
n
n
k=1
akx
α
k
(1)
Ví dn 2.1 De dàng kiem cháng các hàm so sau đây là phân thác chính
quy: f (x) = 1 + 2x + x2
+
1
+
1
; f (x) = 2x2
+ 3x3
+
1
+
2
.
1
x x3 1
x5 x4
Định nghĩa 2.2 Hàm f(x) xác định trên t p các so thực dương, không
đong nhat với 0 được goi là phân thác chính quy hǎu tỉ neu f(x) =
n
k=1
akx
α
n
k
(1) trong đó ak N, αk Z, k = 1, n;
k=1
akαk = 0.
Tà định nghĩa, de dàng ta suy ra các tính chat sau:
Tính chat 2.1 Neu f (x) là hàm phân thác chính quy, thì f (x) > 0 áng
với moi x > 0.
Tính chat 2.2 Neu f (x) và g(x) là các hàm phân thác chính quy, thì
với moi c p so dương α, β hàm so h(x) = αf(x) + βg(x) cũng là hàm
phân thác chính quy.
Tính chat 2.3 Neu f (x) và g(x) là các hàm phân thác chính quy, thì
hàm so h(x) = f(g(x)) cũng là hàm phân thác chính quy.
k=1
trong đó ak ≥ 0, k = 1, n; akαk = 0.
28
Viết đề tài giá sinh viên – ZALO:0973.287.149-TEAMLUANVAN.COM
Σ
t¾p x1, . . . , xm > 0 dạng f(x1, x2, . . . , xm) =
đó
akx k1
x
k2
. . . x km
m
Σ
Tính chat 2.4 Neu f(x) là hàm phân thác chính quy, thì hàm so
h(x) = [f(x)]m
, (m ∈ N∗
) cũng là hàm phân thác chính quy.
Định nghĩa 2.3 Hàm so f(x1, x2, . . . , xm) không đong nhat 0, xác định
trên t p các so thực dương, được goi là hàm phân thác chính quy hǎu
n
tỉ m bien neu f(x1, x2, . . . , xm) = akx
α
k1
xαk2
. . . xαkm
trong đó
1 2 m
k=1
ak ∈ N, αkj ∈ Z, k = 1, n, j = 1, m
a1α11 + a2α21 + · · · + anαn1 = 0
a1α12 + a2α22 + · · · + anαn2 = 0
. . .
a1α1n + a2α2n + · · · + anαnm = 0
Tà định nghĩa trên, ta có các tính chat sau:
Tính chat 2.5 Hàm so f(x1, x2, . . . , xm) là hàm phân thác chính quy
hǎu tỉ khi và chỉ khi các hàm phân thác thành phan của f(1, 1, . . . , xk, 1, . . . , 1)
cũng là hàm phân thác chính quy hǎu tỉ.
Ví dn 2.2 De dàng kiem tra được hàm so
f(x, y) = 3x3
y8
+ 4x4
y9
+
5
x5y12
chính quy, và các hàm so là các phân thác thành phan là:
f1(x) = 3x3
+ 4x4
+ 5x−5
; f2(y) = 3x8
+ 4x9
+ 5x−12
.
Định nghĩa 2.4 Hàm so f (x1, x2, . . . , xm) là hàm phân thác chính quy
khi và chỉ khi các hàm phân thác thành phan của f (x1, x2, . . . , xm) cũng
là các hàm phân thác chính quy.
Định lj 2.4 Với mői hàm phân thúc f(x1, x2, . . . , xm) chính quy trên
n
α α α
ak ∈ N, αkj ∈ Z, k = 1, n, j = 1, m
a1α11 + a2α21 + · · · + anαn1 = 0
a1α12 + a2α22 + · · · + anαn2 = 0
. . .
a1α1n + a2α2n + · · · + anαnm = 0
2
1
k=1
trong
29
Viết đề tài giá sinh viên – ZALO:0973.287.149-TEAMLUANVAN.COM
Σ
Σ Σ Σ
Σ
Σ
Σ Σ
q
Σ
Σ Σ
αk −
p
= ak
p
= q − p.
p
= 0.
Σ
q q Σ Σ
ta đeu có f(x1, . . . , xm) ≥
n
k=1
ak.
ChGng minh. Áp dụng bat đȁng thác AM-GM, ta có
n
akxαk1
xαk2
. . . xαkm
f(x1, x2, . . . , xm) 1 2 m
= k=1
a1 + a2 + . . . + an a1 + a2 + . . . + an
n
≥ x1k=1
akαk1
n
x2k=1
akαk2
n
. . . xmk=1
akαkn
1
a1+a2+...+an
= 1
n
do
k=1
akαkj = 0, j = 1, m .
Dau "=" xảy ra khi và chỉ khi x1 = x2 = · · · = xm = 1.
H quả 2.4 Với mői hàm phân thúc f(x1, x2, . . . , xm) chính quy trên
t¾p
x1, . . . , xm > 0 ta đeu có: min f(x1, x2, . . . , xm) = f(1, 1, . . . , 1).
Nh n xét 2.2 Với moi hàm phân thác dạng g(x) =
n
k=1
akx
α
k
, ak ≥ 0,
n n
đ t ak = p, akαk = q thì hàm so
k=1 k=1
f(x) = x−p g(x)
là m®t hàm phân thác chính quy.
ChGng minh. Ta có
n n
f(x) = x− g(x) = x− a xαk
= a xαk− q
nên
Σ
p
q Σ
p
k
k=1
Σ
k p
k=1
q q
Tà đó ta thu được định lý quan trong sau đây.
n
Định lj 2.5 Moi hàm phân thúc dạng g(x) =
k=1
akx
α
k
, ak ≥ 0, k = 1, n
q
đeu có tính chat g(x) ≥ g(1)xp , ∀x > 0 trong đó
n k=1
ak
= p,
akαk −
k=1
a
k
!
n n n
k=1 k=1
30
Viết đề tài giá sinh viên – ZALO:0973.287.149-TEAMLUANVAN.COM
n
k=1
ak
αk
=
q.
31
Viết đề tài giá sinh viên – ZALO:0973.287.149-TEAMLUANVAN.COM
q
ChGng minh. Theo nh n xét trên, ta có f (x) = x− p g(x) là m®t phân
thác chính quy nên ta suy ra f(x) ≥ f(1) mà f(1) = g(1) nên ta có
đpcm.
Bài toán 2.15 Cháng minh rang các hàm so dạng
f(x) = a + bx + cx2
+
2
cx
b
+
3x3
với a, b, c nguyên dương luôn đạt giá trị nhỏ nhat tại so nguyên dương.
Lài giai. Đe ý rang, f(x) là phân thác chính quy m®t bien áng với
moi a, b, c nguyên dương nên min f(x) = f(1).
Bài toán 2.16 Cháng minh rang các hàm so dạng
g(x) = 2x2
+ 3x3
+
1
x5
2
+
x4
.
Lài giai. Đe ý rang, g(x) là phân thác chính quy m®t bien nên
min g(x) = g(1)..
Bài toán 2.17 Cho các so thực α1, α2, . . . , α9. Tìm giá trị nhỏ nhat của
bieu thác
f(x) = xα1
+ 2xα2
+ 3xα3
+ · · · + 9xα9
, x ∈ R+
.
Lài giai. Ta có
f(x) xα1
+ 2xα2
+ 3xα3
+ · · · + 9xα9
α1 2α2 9α9
45
=
45
≥ x 45 x 45 . . . x 45 = 1 ⇔ f(x) ≥ 45.
V y min f(x) = 45 khi và chỉ khi x = 1.
Bài toán 2.18 Cháng minh rang hàm so
f(x, y) = 3x3
y8
+ 4x4
y9
+
5
x5y12
đạt giá trị nhỏ nhat tại c p so tự nhiên trên t p R+
× R+
..
Lài giai.
Đe ý rang, f(x, y) là phân thác chính quy hai bien, và các hàm so là
các phân thác thành phan là:
32
Viết đề tài giá sinh viên – ZALO:0973.287.149-TEAMLUANVAN.COM
f1(x) = 3x3
+ 4x4
+ 5x−5
; f2(y) = 3x8
+ 4x9
+ 5x−12
.
Tà đó, suy ra giá trị nhỏ nhat của f(x, y) trên t p R+
× R+
. đạt tại
x = 1, y = 1.
Bài toán 2.19 Cháng minh rang hàm so
f(x, y) = 3x3
y8
+ 4x4
y9
+
5
x5y12
đạt giá trị nhỏ nhat tại c p so tự nhiên trên t p R+
× R+
..
Lài giai.
Đe ý rang, f(x, y) là phân thác chính quy hai bien, và các hàm so là
các phân thác thành phan là:
f1(x) = 3x3
+ 4x4
+ 5x−5
; f2(y) = 3x8
+ 4x9
+ 5x−12
.
Tà đó, suy ra giá trị nhỏ nhat của f(x, y) trên t p R+
× R+
. đạt tại
x = 1, y = 1.
33
Viết đề tài giá sinh viên – ZALO:0973.287.149-TEAMLUANVAN.COM
n n 12
12
0
2 22 2n
2 2 22 2n
P(x) =
Σ
Ck
(2x)k
=
Σ
Ck
xk
.
Chương 3
M t so dạng toán liên quan
3.1. M t so dạng toán cfic trị trên t p so nguyên
Trong phan này ta đi xét m®t so bat đȁng thác phân thác trên t p
rời rạc với các đieu ki n ràng bu®c khác nhau.
Bài toán 3.1 Cho đa thác
P(x) = (1 + 2x)n
= a0 + a1x + a2x2
+ · · · + anxn
a1 a2 an
, biet a + + + · · · + = 4096.
Xét dãy so (ak), k = 0, 2, . . . , n. Tìm so hạng lớn nhat của dãy so (ak).
Lài giai.
Ta có
f
1
= a +
a1
+
a2
+ · · · +
an
Khi đó, ta có
⇔ 2 = 4096 ⇔ 2 = 2 ⇔ n = 12.
12 12
Suy ra
n
k=0
12
k=0
ak = Ck
2k
, k = 0, 1, 2, . . . , n.
So hạng ak lớn nhat khi và chỉ khi
ak ≥ ak+1
ak ≥ ak−1
(
0
34
Viết đề tài giá sinh viên – ZALO:0973.287.149-TEAMLUANVAN.COM
− ≥
2
Σ
N
Ck 2k ≥ C 2k+1 k!(12 − k)! (k + 1)!(11 − k)!
Vì k ∈ Z nên k = 8. V y a8 = 28
C8
Σ
n
i i+1
4
, ta có Cn−1
< Cn
,
n n−2
3
i=1 i=1 i=k
2 2 N N
0≤n≤N
N N
2
(
k+1
12!
2k
≥
12!
2k+1
⇔
Ck 2k ≥ Ck−1
2k−1
⇔ 12! k 12! k−1
12 12 2
k!(12 − k)!
2
(k − 1)!(13 − k)!
⇔
1
12 k
2
2
≥
k + 1
1 k
22
⇔ 26
k
≥ 13 − k k ≤
3
dãy (ak)
Bài toán 3.2 Cho so nguyên dương n > 1 và so thực p > 0. Xét dãy
các so {x1, . . . , xn} không âm thỏa mãn đieu ki n
n
Σ
−1
i=1
xi = p. Tìm giá trị
Lài giai. Đ t xk= max{x1, x2, . . . , xn}. Khi đó
n−1 k−1 n−1
Σ
xixi+1 =
Σ
xixi+1 +
Σ
xixi+1
Dau đȁng thác xảy ra khi và chỉ khi có hai so của dãy bang p
, các so
khác đeu bang 0. V y
n−1
p2
max x x = .
Bài toán 3.3 Cho so nguyên dương N. Tìm giá trị lớn nhat trong dãy
so Cn
(n = 0, 1, . . . , N).
Lài giai. Do Cn
= CN −n nên ta chỉ can xét dãy {Cn
} với n =
N
N N
N
N
nên max {Cn
} = C , p =
N
.
p
Bài toán 3.4 (Taiwan TST 2014) Cho đa thác với h so thực
P(x) = xn
+ an−2xn−2
+ an−3xn−3
+ · · · + a1x + a0, n ≥ 2.
Giả sả xk nghi m thực của phương trình P(x) = 0,k = 1, 2, . . . , n.
Cháng minh rang |x | ≤
r
2(1 − n)
a .
i=1
i=1
= 126720 là so hạng lớn nhat của
12
≥
12 12
lớn nhat của bieu thác M = xixi+1.
0, 1, . . . , . M t khác thì với n = 0, 1, . . . ,
k
35
Viết đề tài giá sinh viên – ZALO:0973.287.149-TEAMLUANVAN.COM
Σ
Σ x
i
i
n n i
Σ
x2
> n.(
n − 1
)
Σ
x2
⇔
Σ
x2
> (n − 1)
Σ
x2
1
+
1
+
1 2
a(b + c) + b(c + a)c(a + b)
i
Lài giai. Theo định lý Viète, ta có
n
i=1 xi = 0
Ta có
1≤i
Σ
<j≤n
Σ Σ
xixj = an−2.
Σ
n
!2
suy ra
2 xixj +
1≤i<j≤n i=1 x2
= xi
i=1
= 0,
Ta giả sả
an−2 =
−1
n
2 i
i=1
n
x2
>
2(1 − n)
a =
n − 1 Σ
x2
, ∀i = 1, 2, . . . , n.
Lay tőng, ta thu được
n n
i
i=1
n
n i
i=1
n
i
i=1 n
i
i=1
⇔ (2 − n)
Σ
x2
> 0 ⇔ n < 2,
i=1
mâu thuan với giả thiet. Ta có ngay đieu phải cháng minh.
Bài toán 3.5 (IMO 1995) Cho a, b, c là các so hǎu t dương thỏa mãn
đieu ki n abc = 1. Cháng minh rang
1 1 1 3
a3(b + c)
+
b3(c + a)
+
c3(a + b)
≥
2
.
Lài giai. Áp dụng bat đȁng thác AM-GM, ta có
1 1 1
a3(b + c)
+
b3(c + a)
+
c3(a + b)
≥ a b c
i=1
n
n−2
!
.
2
36
Viết đề tài giá sinh viên – ZALO:0973.287.149-TEAMLUANVAN.COM
≥
−
b
−
c
−
2
1
2
(ab + bc + ca)2
= .
a(b + c) + b(c + a) + c(a + b)
Ta can cháng minh bat đȁng thác
(ab + bc + ca)2 3
[a(b + c) + b(c + a) + c(a + b)]
2
⇔ ab + bc + ca ≥ 3 (luôn đúng theo AM-GM).
Dau bang xảy ra khi và chỉ khi a = b = c = 1.
Bài toán 3.6 (China Northern Mathematical Olympiad 2007) Cho a, b, c
là các so hǎu t dương thỏa mãn đieu ki n abc = 1. Cháng minh rang
với moi so nguyên k ≥ 2 thì
ak bk ck
3
Lài giai. Ta có
a + b
+
b + c
+
c + a
≥
2
.
ak bk ck
3
a + b
+
b + c
+
c + a
≥
2
⇔ ak−1 + bk−1
+ ck−1
≥
3 ak−1b
+ +
2 a + b
bk−1c
+
b + c
ck−1a
.
c + a
Áp dụng bat đȁng thác AM-GM, ta có
a + b ≥ 2
√
ab, b + c ≥ 2
√
bc, c + a ≥ 2
√
ca.
Vì v y phải cháng minh:
3
k 1 k
a 2 2 + b
3
1 k
2 2 + c
3
2a
1
+ 3 ≤ 2(ak−1
+ bk−1
+ ck−1
).
Theo bat đȁng thác AM-GM
ak−1
+ bk−1
+ ck−1
≥ 3
√
3
ak−1bk−1ck−1 = 3
và
k
(2k − 3)ak−1
+ bk−1
≥ (2k − 2)a
3
2b2
3
(2k − 3)bk−1
+ ck−1
≥ (2k − 2)b
k−
2c
1
−
37
Viết đề tài giá sinh viên – ZALO:0973.287.149-TEAMLUANVAN.COM
2a2 .
a b c
b c a a b c
+ b + c
a3
+
b3
+
c3
+ b + c
a3
+
b3
+
3
(2k − 3)ck−1
+ ak−1
≥ (2k − 2)c
k− 1
C®ng ba bat đȁng thác trên ta được đieu phải cháng minh.
Dau đȁng thác xảy ra khi a = b = c = 1.
Bài toán 3.7 (Brazilian Olympiad Revenge 2007) Cho a, b, c là các so
hǎu t dương thỏa mãn đieu ki n abc = 1. Cháng minh rang
a2
+b2
+c2
+
1
1 1 a+b+c
1
1 1
≥ 6+2
a b c b c a
.
a2
+
b2
+
c2
+2
Lài giai. Do abc = 1, ta có
+ + +
a b c
+ + + + +
b c a a b c
a2
+ b2
+ c2
+ 2
1
+
1
+
1
= a2
+ b2
+ c2
+ 2(ab + bc + ca) = (a + b + c)2
1 1
a2
+
b2
1
+
c2
+ 2(a + b + c) = a2
b2
+ b2
c2
+ c2
a2
+ 2abc(a + b + c) = (ab + bc + ca)2
.
Lại có:
2
a
+
b
+
c
+
b
+
c
+
a
+ 3
2(ab(a + b) + bc(b + c) + ca(c + a) + 3abc)
=
abc
= 2(a + b + c)(ab + bc + ca).
Áp dụng bat đȁng thác AM-GM ta được
(a+b+c)2
+(ab+bc+ca)2
≥ 2(a+b+c)(ab+bc+ca) (đieu phải cháng minh).
Dau đȁng thác xảy ra khi a = b = c = 1.
Bài toán 3.8 (Hongkong 2000) Với các so hǎu t dương a, b, c thỏa
mãn đieu ki n abc = 1. Cháng minh rang
1 + ab2
c3
+
1 + bc2
a3
+
1 + ca2
b3
≥
18
a3 + b3 + c3
.
Lài giai. Bat đȁng thác can cháng minh tương đương với bat đȁng
thác
1 1 1 bc2
ca
2
ab2
18
a3
+
b3
+
c3
+
a3
+
b3
+
c3
≥ a3 + b3 + c3
3 3 3 1 1 1 3 3 3 bc2
ca
2
ab2
c3
⇔ (a ) + (a ) ≥ 18
38
Viết đề tài giá sinh viên – ZALO:0973.287.149-TEAMLUANVAN.COM
c3
√ √ √
+ b + c
a3
+ 3
+ ≥ 3 3
= 9
2
2 2 2
4x2
4y
2
4z2
Áp dụng bat đȁng thác AM-GM, ta có
(a3
+ b3
+ c3
)
1
a3
1
+
b3
+
1
≥ 3
√
3
a3b3c3
r
3 1
a3b3c3
= 9
3 3 3 bc2
ca2
ab2 √
3 r
3 a3
b3
c3
C®ng ve với ve ta có đieu phải cháng minh.
Đȁng thác xảy ra khi và chỉ khi a = b = c = 1.
Bài toán 3.9 (APMO 2005) Với các so hǎu t dương x, y, z thỏa mãn
đieu ki n xyz = 8. Cháng minh rang
x2 y2 z2
4
(x3 + 1)(y3 + 1)
+
(y3 + 1)(z3 + 1)
+
(z3 + 1)(x3 + 1)
≥
3
.
Lài giai. Với moi a ≥ 0, áp dụng bat đȁng thác AM-GM, ta có
√
a3 + 1 =
√
(a + 1)(a2 − a + 1) ≤
a + 1 + a2
− a + 1
2
a2
+ 2
= .
2
Áp dụng bat đȁng thác này, ta có
x2 y2 z2
√
(x3 + 1)(y3 + 1)
+ √
(y3 + 1)(z3 + 1)
+ √
(z3 + 1)(x3 + 1)
≥
(x2 + 2)(y2 + 2)
+
(y2 + 2)(z2 + 2)
+
(z2 + 2)(x2 + 2)
.
Ta phải cháng minh
4x2
4y2
4z2
4
(x2 + 2)(y2 + 2)
+
(y2 + 2)(z2 + 2)
+
(z2 + 2)(x2 + 2)
≥
3
⇔ x (z + 2) + y2
(x2
+ 2) + z2
(y2
+ 2) ≥
1
(x2
3
+ 2)(y2
+ 2)(z2
+ 2)
⇔ x y + y2
z2
+ z2
x2
+ 2(x2
+ y2
+ z2
) ≥ x y z + 8 = 72.
Bat đȁng thác trên luôn đúng do
x2
y2
+y2
z2
+z2
x2
+2(x2
+y2
+z2
) ≥ 3
√
3
x4y4z4 +2.3
√
3
x2y2zz = 48 + 24.
Đȁng thác xảy ra khi và chỉ khi x = y = z = 2.
3
c
b
(a )
a3b3c3
a3b3c
2
2 2
39
Viết đề tài giá sinh viên – ZALO:0973.287.149-TEAMLUANVAN.COM
2011
Σ −
A =
(k − n
x
)
C x (1 − x)
−
n−1
n−2
Σ
= C x (1 − x) = [x
+
(
1− x)] = 1
= (nx)2
Σ
Ck
xk
(1 − x)n−k
+
Σ
k2
Ck
xk
(1 − x)n−k
− 2nx
Σ
(k − nx)2
Ck
x1 − xk
A1 =
Σ
Ck
xk
(1 − x)n−k
=
Σ
Ck
xk
(1 − x)n−k
n−1 n−1
= nx
Σ
Ck−1
xk−1
(1 − x)n−k
+ n
Σ
k − 1Ck−1
xk
(1 − x)n−k
A
−
k=1 k=1
3.2. M t so dạng toán ve so tfi nhiên tfi các đe thi
Olympic
Ta xét m®t so bài toán liên quan đen công thưsc tő hợp tà các đe thi
Olympic.
Bài toán 3.10 (Olympic 30/4/2011) Cho hàm so
2011
F(x) =
Σ
(k − 2011x)2
Ck
xk
(1 − x)2011−k
.
k=0
Tìm giá trị nhỏ nhat của hàm so F(x) trên [0; 1]
n
2 k k n k
n
k=0
n n n
Xét
n
k=0
n
n
k=0
n
n
k=0
n
k=0n
n
k=1 n
= n
Σ
Ck−1
xk
(1 − x)n−k
= nx
Σ
Ck−1
xk−1
(1 − x)n−k
= nx[x + (1 x)]n−1
= nx
n n
A =
Σ
k2
Ck
xk
(1 − x)
n−k
=
Σ
k2
Ck
xk
(1 − x)n−k
2 n
k=0n
n
k=1
= n
Σ
kCk−1
xk
(1 − x)n−k
k=1
n n
k=1
n−1
n k=1
n−1
= nx + n(n − 1)
Σ
Ck−2
xk
(1 − x)n−k
= nx + n(n − 1)x2
k=2
n
k k n k n
3 n
k=0
V y A = (nx)2
+ nx + n(n − 1)x2
− 2(nx)2
= nx(1 − x).
40
Viết đề tài giá sinh viên – ZALO:0973.287.149-TEAMLUANVAN.COM
C
n
C
Σ
n+4
2(n + 3)(n + 4)
n
n
ta có f(x) ≤ 2011 Dau đȁng thác xảy ra khi x = .
Áp dụng ket quả trên ta thu được f(x) = 2011x(1 − x).
Do x ∈ [0; 1] nên x ≥ 0, 1 − x ≥ 0. Áp dụng bat đȁng thác AM - GM,
V y max f(x) =
[0;1]
2011 1
4
⇔ x =
2
Nh n xét 3.1 Ta có the thay so 2011 trong hàm so f(x) bởi m®t so
thực dương bat kì.
Xuat phát tà cách viet tőng
Σn
(−1)k k
dưới dạng
phân thác p(n)
q(n)
k=0
k3 + 9k2 + 26k + 24
, trong đó p(n), q(n) là các đa thác với h so nguyên, ta
có the xây dựng được bài toán cực trị liên quan đen tő hợp là bài toán
sau:
Bài toán 3.11 Tìm giá trị nhỏ nhat bieu thác
n k k 4 3 2
f(n) =
Σ
(−1) Cn +
2n − 10n − 64n + 272n + 959
.
Lài giai. Đ t
S(n) =
Σ
(−1)k k
Ta có
k=0
k3 + 9k2 + 26k + 24
nên
k3
+ 9k2
+ 26k + 24 = (k + 2)(k + 3)(k + 4)
n k
S(n) =
(−1) .n!
k=0
k!.(n − k)!(k + 2)(k + 3)(k + 4)
Đ t
=
k=0
(−1)n
!.(n + 4)!
(k + 4)!.(n − k)!
k + 1
(n + 1)(n + 2)(n + 3)(n + 4)
n
T(n) = (n + 1)(n + 2)(n + 3)(n + 4)S(n) =
Σ
Ck+4
(k + 1).
k=0
n
k=0
x + (1 − x)
2
1
2
k3 + 9k2 + 26k + 24
Σ
41
Viết đề tài giá sinh viên – ZALO:0973.287.149-TEAMLUANVAN.COM
Σ
(−1) C
Σ
= (x − 1) = 0 ⇒ −n
(−1) C
Σ
−
Σ
= (−1) nC
Σ
= n
(−1) C
Σ
= −n (−1)
nC
n+4
Σ
= (−1) C (k + 1)
n
Σ
C
Σ
= C
Σ
j − 3
(−1) C
Σ
(−1)i
Ci
.i =
Σ
(−1)i i.n!
+ (−1)0 0.n!
n 1 = 0
j
Ta có
và
n
i i
n
i=0
n
i j j
n−1
j=0
n n
i=0
n
i=1
n
i!(n − i)! 0!n!
n
= ( 1)i n!
i=1
(i − 1)!(n − i)!
i i−1
n−1
i=1
Do đó n
n
i i−1
n−1
i=1
n
i−1 i−1
−
i=1
T(n) =
Σ
(−1)k
Ck+4
(k + 1)
k=0
n
k+4 k+4
n+4
k=0
=
Σ
(−1)k+4
Ck+4
(k + 1) − (−3 + 2(n + 4) − C2
)
k=−4
n+4
= n+4
n+4
(j − 3) − 2n + 8 − 3 −
n+4
(n + 4)(n + 3)
2
j=0
n+4
j
n+4
j=0
n+4
j j
n+4
j=0
1 2
—
2
(4n + 10 − n − 7n − 12)
Suy ra
= 0 + 0 +
1
(n2
2
+ 3n + 2) =
1
(n + 1)(n + 2)
2
T(n)
S(n) = = dfrac(n + 1)(n + 2)(n + 1)(n + 2)(n + 3)(n +
V y
(n + 1)(n + 2)(n + 3)(n + 4)
1
S(n) =
Do đó
f(n) =
1
2(n + 3)(n + 4)
2(n + 3)(n + 4)
2n4
− 10n3
− 64n2
+ 272n + 959
2(n + 3)(n + 4)
+
= 0
42
Viết đề tài giá sinh viên – ZALO:0973.287.149-TEAMLUANVAN.COM
−
≥
2n4
− 10n3
− 64n2
+ 272n + 960
2(n + 3)(n + 4)
2(n + 3)(n + 4)(n2
12n + 40)
=
2(n + 3)(n + 4)
Ta có
= n2
− 12n + 40
n2
−12n+40 = (n−6)2
+4 ≥ 4, ∀n ∈ N. Dau đȁng thác xảy ra khi n = 6
V y f(n) đạt giá trị nhỏ nhat bang 4 khi n = 6.
Bài toán 3.12 (Bulgari TST 2003)
Với ba so nguyên dương a, b, c thỏa mãn đieu ki n a + b + c = 3. Cháng
minh rang
a b c 3
1 + b2
+
1 + c2
+
1 + a2
≥
2
.
Lài giai. Ta không the dùng trực tiep bat đȁng thác AM-GM với mau
so vì bat đȁng thác sau đó sě đői chieu. Tuy nhiên ta có the dùng lại
bat đȁng thác đó theo cách khác.
Áp dụng bat đȁng thác AM-GM, ta có
a ab2
ab2
ab
1 + b2
= a −
1 + b2
≥ a −
2b
= a −
2
.
Tương tự, ta có
b bc
1 + c2
≥ b −
2
c ac
1 + a2
≥ c −
2
.
C®ng ve với ve ba bat đȁng thác trên ta có
a b c 1 ab + bc + ca
1 + b2
+
1 + c2
+
1 + a2
≥ a+b+c−
2
(ab+bc+ca) = 3−
2
.
Ta lại có
Do đó
a b
ab + bc + ca ≤
c
(a + b + c)2
= 3.
3
1 ab + bc + ca 3
1 + b2
+
1 + c2
+
1 + a2
≥ a+b+c−
2
(ab+bc+ca) = 3−
2 2
.
Đȁng thác xảy ra khi và chỉ khi a = b = c = 1.
=
43
Viết đề tài giá sinh viên – ZALO:0973.287.149-TEAMLUANVAN.COM
1
Bài toán 3.13 (Olympic 30/4/2009) Với moi b® n so nguyên dương
a1, a2, . . . , an có tőng bang 2009. Đ t A là tőng tat cả các so hạng có
dạng
1
a1(a1 + a2)(a1 + a2 + a3) . . . (a1 + a2 + · · · + an)
trong đó k1, k2, . . . , kn là m®t hoán vị bat kì của {1, 2, . . . , n} . Tìm giá trị nhỏ nhat
Lài giai. Xét m®t b® bat kì n so nguyên dương a1, a2, . . . , an có tőng
bang 2009
Ta cháng minh bang quy nạp
+) Với n = 1, (3.1)đúng
1
A =
a1a2 . . . an
(3.1)
+) Giả sả (3.1)đúng đen n − 1
Với moi n so nguyên dương a1, a2, . . . , an
1
Trong A, đ t
a1 + a2 + · · · + an
A =
làm thàa so chung, ta được
1
,
trong đó B gom n! so hạng.
a1 + a2 + · · · + an
Trong B ta nhóm các so hạng không cháa a1 thành tőng B1 gom (n−1)!
so hạng áng với (n − 1)! hoán vị của {2, 3 . . . , n}
Do đó theo giả thiet quy nạp thì B =
a2a3 . . . an
Tiep tục như the với a2, a3, . . . , an ta được
1 1 1
B = + + · · · + =
a1 + a2 · · · + an
a2a3 . . . an
Tà đó suy ra A =
a1a3 . . . an
1
a1a2 . . . an
a1a2 . . . an−1 a1a2 . . . an
V y theo nguyên lí quy nạp (3.1) đúng với moi n ≥ 1.
+) Neu a1 = 1, loại a1 = 1, the a2 = 1 bởi a′
2 = 1 + a2 thì tőng các so
ai không đői, còn tích tăng lên.
+) Neu ai ≥ 5 the a1 bởi 2(a1 − 2) thì tőng các so ai không đői, tích
tăng lên vì 2(a1 − 2) = 2a1 − 4 > a1
+) Neu có m®t so ai = 4, the so đó bởi 2 + 2 thì tőng và tích các so ai
không đői
44
Viết đề tài giá sinh viên – ZALO:0973.287.149-TEAMLUANVAN.COM
+) Neu có ba so 2, the ba so đó bang hai so 3 thì tőng không đői, tích
tăng lên
V y đe tích p của các so ai lớn nhat thì phải chon không quá hai so 2,
các so khác bang 3 Do 2009 = 3.669 + 2 nên maxP = 2.3669
. Suy ra
1
minA =
2.3669
.
45
Viết đề tài giá sinh viên – ZALO:0973.287.149-TEAMLUANVAN.COM
Ket lu n
Lu n văn “M®t so phương pháp tìm cực trị của các hàm phân
thác sinh bởi so tự nhiên” đã trình bày nhǎng van đe sau:
- Lu n văn trình bày chi tiet các tính chat cơ bản của đa thác
với h so nguyên, phân thác hǎu t với h so nguyên, các đong
nhat thác và bat đȁng thác dạng phân thác hǎu t với h so
nguyên.
- Tiep theo trình bày chi tiet các dạng toán tìm cực trị trong
lớp các hàm phân thác.
- Cuoi cùng, lu n văn trình bày các dạng toán liên quan được
chon loc tà các đe thi HSG quoc gia, Olympic khu vực và quoc
te.
46
Viết đề tài giá sinh viên – ZALO:0973.287.149-TEAMLUANVAN.COM
Tài li u tham khảo
[A] Tieng Vi t
[1] Nguyen Văn M u (2002), Đa thúc đại so và phân thúc hũu ty, NXB
Giáo dục.
[2] Nguyen Văn M u (2016), N®i suy đa thúc, NXB ĐHQG Hà N®i.
[3] Nguyen Văn M u, Trịnh Đào Chien, Tran Nam Dũng, Nguyen Đăng
Phat (2008), Chuyên đe chon loc ve đa thúc và áp dựng, NXB Giáo
dục.
[4] Nguyen Văn Nho, OLYMPIC toán hoc Châu Á Thái Bình Dương,
NXB Giáo dục.
[B] Tieng Anh
[5] Paulo Ney de Sausa, Jorge- Nume Silva (1998), Berkeley Problems
in Mathematics, Springer.
[6] Pólya G., Szgo G., 1956, Problems and theorems from Analysis,
Moscow.
[7] Radulescu T-L.T. , Radulescu V.D., Andreescu T. (2009). Problems
in Real Analysis: Advanced calculus on the real axis. Springer.
[8] Titu Andreescu (2004), Mathematical Olympiad Treasures,
Birkhauser Boston, USA.
ĐẠI HOC THÁI NGUYÊN
TRƯ NG ĐẠI HOC KHOA HOC
Viết đề tài giá sinh viên – ZALO:0973.287.149-TEAMLUANVAN.COM
NGUYEN KIM THU
M T SO PHƯƠNG PHÁP TÌM CỰC
TR± CỦA CÁC HÀM PHÂN THỨC
SINH B I SO TỰ NHIÊN
LU N VĂN THẠC SĨ TOÁN HOC
THÁI NGUYÊN - 2018
ĐẠI HOC THÁI NGUYÊN
TRƯ NG ĐẠI HOC KHOA HOC
Viết đề tài giá sinh viên – ZALO:0973.287.149-TEAMLUANVAN.COM
NGUYEN KIM THU
M T SO PHƯƠNG PHÁP TÌM CỰC
TR± CỦA CÁC HÀM PHÂN THỨC
SINH B I SO TỰ NHIÊN
Chuyên ngành: PHƯƠNG PHÁP TOÁN SƠ CAP
Mã so: 8 46 01 13
LU N VĂN THẠC SĨ TOÁN HOC
Người hướng dan khoa hoc: GS.TSKH. Nguyen Văn M u
THÁI NGUYÊN - 2018

More Related Content

Similar to Một số phương pháp tìm cực trị của các hàm phân thức Sinh bởi số tự nhiên.docx

Hàm Đơn Đi›U, Tựa Đơn Đi›U Và Một Số Ứng Dụng Của Phép Đơn Đi›U Hóa Hàm Số.docx
Hàm Đơn Đi›U, Tựa Đơn Đi›U Và Một Số Ứng Dụng Của Phép Đơn Đi›U Hóa Hàm Số.docxHàm Đơn Đi›U, Tựa Đơn Đi›U Và Một Số Ứng Dụng Của Phép Đơn Đi›U Hóa Hàm Số.docx
Hàm Đơn Đi›U, Tựa Đơn Đi›U Và Một Số Ứng Dụng Của Phép Đơn Đi›U Hóa Hàm Số.docxDV Viết Luận văn luanvanmaster.com ZALO 0973287149
 
Một số dạng toán về đa thức qua các kỳ thi Olympic 6732069.pdf
Một số dạng toán về đa thức qua các kỳ thi Olympic 6732069.pdfMột số dạng toán về đa thức qua các kỳ thi Olympic 6732069.pdf
Một số dạng toán về đa thức qua các kỳ thi Olympic 6732069.pdfTieuNgocLy
 
06 mat102-bai 3-v1.0
06 mat102-bai 3-v1.006 mat102-bai 3-v1.0
06 mat102-bai 3-v1.0Yen Dang
 
Tổng quát về tích phân
Tổng quát về tích phân Tổng quát về tích phân
Tổng quát về tích phân Hoàng Hải Huy
 
Bai giang-toan-kinh-te-tin-hoc
Bai giang-toan-kinh-te-tin-hocBai giang-toan-kinh-te-tin-hoc
Bai giang-toan-kinh-te-tin-hocLê Ngọc Huyền
 

Similar to Một số phương pháp tìm cực trị của các hàm phân thức Sinh bởi số tự nhiên.docx (20)

Hàm Đơn Đi U, Tựa Đơn Đi U Và M T So Ứng Dụng Của Phép Đơn Đi U Hóa Hàm So.docx
Hàm Đơn Đi U, Tựa Đơn Đi U Và M T So Ứng Dụng Của Phép Đơn Đi U Hóa Hàm So.docxHàm Đơn Đi U, Tựa Đơn Đi U Và M T So Ứng Dụng Của Phép Đơn Đi U Hóa Hàm So.docx
Hàm Đơn Đi U, Tựa Đơn Đi U Và M T So Ứng Dụng Của Phép Đơn Đi U Hóa Hàm So.docx
 
Bat Phương Trình Hàm Sinh B I Các Đại Lư Ng Trung Bình B C Tùy Ý Và Các Dạng ...
Bat Phương Trình Hàm Sinh B I Các Đại Lư Ng Trung Bình B C Tùy Ý Và Các Dạng ...Bat Phương Trình Hàm Sinh B I Các Đại Lư Ng Trung Bình B C Tùy Ý Và Các Dạng ...
Bat Phương Trình Hàm Sinh B I Các Đại Lư Ng Trung Bình B C Tùy Ý Và Các Dạng ...
 
Hàm Đơn Đi›U, Tựa Đơn Đi›U Và Một Số Ứng Dụng Của Phép Đơn Đi›U Hóa Hàm Số.docx
Hàm Đơn Đi›U, Tựa Đơn Đi›U Và Một Số Ứng Dụng Của Phép Đơn Đi›U Hóa Hàm Số.docxHàm Đơn Đi›U, Tựa Đơn Đi›U Và Một Số Ứng Dụng Của Phép Đơn Đi›U Hóa Hàm Số.docx
Hàm Đơn Đi›U, Tựa Đơn Đi›U Và Một Số Ứng Dụng Của Phép Đơn Đi›U Hóa Hàm Số.docx
 
Về Tổng Gauss Và Một Số Ứng Dụng.docx
Về Tổng Gauss Và Một Số Ứng Dụng.docxVề Tổng Gauss Và Một Số Ứng Dụng.docx
Về Tổng Gauss Và Một Số Ứng Dụng.docx
 
Bat Đang Thức V I Hàm Loi B Ph N Và Ứng Dụng.docx
Bat Đang Thức V I Hàm Loi B Ph N Và Ứng Dụng.docxBat Đang Thức V I Hàm Loi B Ph N Và Ứng Dụng.docx
Bat Đang Thức V I Hàm Loi B Ph N Và Ứng Dụng.docx
 
Một số dạng toán về đa thức qua các kỳ thi Olympic 6732069.pdf
Một số dạng toán về đa thức qua các kỳ thi Olympic 6732069.pdfMột số dạng toán về đa thức qua các kỳ thi Olympic 6732069.pdf
Một số dạng toán về đa thức qua các kỳ thi Olympic 6732069.pdf
 
Toán Tử Sai Phân Và Ứng Dụng Vào Giải Toán Sơ Cấp.docx
Toán Tử Sai Phân Và Ứng Dụng Vào Giải Toán Sơ Cấp.docxToán Tử Sai Phân Và Ứng Dụng Vào Giải Toán Sơ Cấp.docx
Toán Tử Sai Phân Và Ứng Dụng Vào Giải Toán Sơ Cấp.docx
 
M T So Dạng Toán Cực Tr± Trong L P Hàm Mũ Và Hàm Hyperbolic.docx
M T So Dạng Toán Cực Tr± Trong L P Hàm Mũ Và Hàm Hyperbolic.docxM T So Dạng Toán Cực Tr± Trong L P Hàm Mũ Và Hàm Hyperbolic.docx
M T So Dạng Toán Cực Tr± Trong L P Hàm Mũ Và Hàm Hyperbolic.docx
 
M T So Dạng Toán Ve Dãy So Sinh B I Các Hàm So Sơ Cap.docx
M T So Dạng Toán Ve Dãy So Sinh B I Các Hàm So Sơ Cap.docxM T So Dạng Toán Ve Dãy So Sinh B I Các Hàm So Sơ Cap.docx
M T So Dạng Toán Ve Dãy So Sinh B I Các Hàm So Sơ Cap.docx
 
Bất đẳng thức Trong lớp các hàm lượng giác và lượng giác ngược.docx
Bất đẳng thức Trong lớp các hàm lượng giác và lượng giác ngược.docxBất đẳng thức Trong lớp các hàm lượng giác và lượng giác ngược.docx
Bất đẳng thức Trong lớp các hàm lượng giác và lượng giác ngược.docx
 
Giá Trị Trung Bình Với Hàm Tùy Ý Và Một Số Lớp Hàm Lồi Liên Quan.docx
Giá Trị Trung Bình Với Hàm Tùy Ý Và Một Số Lớp Hàm Lồi Liên Quan.docxGiá Trị Trung Bình Với Hàm Tùy Ý Và Một Số Lớp Hàm Lồi Liên Quan.docx
Giá Trị Trung Bình Với Hàm Tùy Ý Và Một Số Lớp Hàm Lồi Liên Quan.docx
 
Một số phương pháp giải các đề thi olympic Về phương trình diophant.docx
Một số phương pháp giải các đề thi olympic Về phương trình diophant.docxMột số phương pháp giải các đề thi olympic Về phương trình diophant.docx
Một số phương pháp giải các đề thi olympic Về phương trình diophant.docx
 
06 mat102-bai 3-v1.0
06 mat102-bai 3-v1.006 mat102-bai 3-v1.0
06 mat102-bai 3-v1.0
 
Ve H Phương Trình Phi Tuyen Và Ứng Dụng.docx
Ve H Phương Trình Phi Tuyen Và Ứng Dụng.docxVe H Phương Trình Phi Tuyen Và Ứng Dụng.docx
Ve H Phương Trình Phi Tuyen Và Ứng Dụng.docx
 
Tổng quát về tích phân
Tổng quát về tích phân Tổng quát về tích phân
Tổng quát về tích phân
 
Phương trình diophantine dạng X2 − dy2 = ±4.docx
Phương trình diophantine dạng X2 − dy2 = ±4.docxPhương trình diophantine dạng X2 − dy2 = ±4.docx
Phương trình diophantine dạng X2 − dy2 = ±4.docx
 
Bai giang-toan-kinh-te-tin-hoc
Bai giang-toan-kinh-te-tin-hocBai giang-toan-kinh-te-tin-hoc
Bai giang-toan-kinh-te-tin-hoc
 
Ve Ho Chuan Tac Các Hàm Phân Hình.docx
Ve Ho Chuan Tac Các Hàm Phân Hình.docxVe Ho Chuan Tac Các Hàm Phân Hình.docx
Ve Ho Chuan Tac Các Hàm Phân Hình.docx
 
Đang thức, bat đang thức tích phân trong l p đa thức và phân thức hữu ty và m...
Đang thức, bat đang thức tích phân trong l p đa thức và phân thức hữu ty và m...Đang thức, bat đang thức tích phân trong l p đa thức và phân thức hữu ty và m...
Đang thức, bat đang thức tích phân trong l p đa thức và phân thức hữu ty và m...
 
Về Định Lý Van Der Waerden, Số Ramsey Và Tập Đơn Sắc.docx
Về Định Lý Van Der Waerden, Số Ramsey Và Tập Đơn Sắc.docxVề Định Lý Van Der Waerden, Số Ramsey Và Tập Đơn Sắc.docx
Về Định Lý Van Der Waerden, Số Ramsey Và Tập Đơn Sắc.docx
 

More from DV Viết Luận văn luanvanmaster.com ZALO 0973287149

More from DV Viết Luận văn luanvanmaster.com ZALO 0973287149 (20)

Ảnh Hưởng Của Marketing Quan Hệ Đến Lòng Trung Thành Của Khách Hàng.Tình Huốn...
Ảnh Hưởng Của Marketing Quan Hệ Đến Lòng Trung Thành Của Khách Hàng.Tình Huốn...Ảnh Hưởng Của Marketing Quan Hệ Đến Lòng Trung Thành Của Khách Hàng.Tình Huốn...
Ảnh Hưởng Của Marketing Quan Hệ Đến Lòng Trung Thành Của Khách Hàng.Tình Huốn...
 
Phát triển nguồn nhân lực tại Uỷ ban nhân dân huyện Trà Bồng, tỉnh Quảng Ngãi...
Phát triển nguồn nhân lực tại Uỷ ban nhân dân huyện Trà Bồng, tỉnh Quảng Ngãi...Phát triển nguồn nhân lực tại Uỷ ban nhân dân huyện Trà Bồng, tỉnh Quảng Ngãi...
Phát triển nguồn nhân lực tại Uỷ ban nhân dân huyện Trà Bồng, tỉnh Quảng Ngãi...
 
Báo cáo tốt Nghiệp tài chính hợp nhất tại tổng công ty Indochina gol...
Báo cáo tốt Nghiệp  tài chính hợp nhất tại tổng công ty Indochina gol...Báo cáo tốt Nghiệp  tài chính hợp nhất tại tổng công ty Indochina gol...
Báo cáo tốt Nghiệp tài chính hợp nhất tại tổng công ty Indochina gol...
 
Tạo động lực thúc đẩy nhân viên làm việc tại ngân hàng TMCP Ngoại Thương Việt...
Tạo động lực thúc đẩy nhân viên làm việc tại ngân hàng TMCP Ngoại Thương Việt...Tạo động lực thúc đẩy nhân viên làm việc tại ngân hàng TMCP Ngoại Thương Việt...
Tạo động lực thúc đẩy nhân viên làm việc tại ngân hàng TMCP Ngoại Thương Việt...
 
Phát triển công nghiệp trên địa bàn Thành phố Tam Kỳ, Tỉnh Quảng Na...
Phát triển công nghiệp trên địa bàn Thành phố Tam Kỳ, Tỉnh Quảng Na...Phát triển công nghiệp trên địa bàn Thành phố Tam Kỳ, Tỉnh Quảng Na...
Phát triển công nghiệp trên địa bàn Thành phố Tam Kỳ, Tỉnh Quảng Na...
 
Giải pháp phát triển cho vay xuất nhập khẩu tại ngân hàng NN&PTNN ch...
Giải pháp phát triển cho vay xuất nhập khẩu tại ngân hàng NN&PTNN ch...Giải pháp phát triển cho vay xuất nhập khẩu tại ngân hàng NN&PTNN ch...
Giải pháp phát triển cho vay xuất nhập khẩu tại ngân hàng NN&PTNN ch...
 
Hoàn thiện công tác lập báo cáo tài chính hợp nhất tại tổng công ...
Hoàn thiện công tác lập báo cáo tài chính hợp nhất tại tổng công ...Hoàn thiện công tác lập báo cáo tài chính hợp nhất tại tổng công ...
Hoàn thiện công tác lập báo cáo tài chính hợp nhất tại tổng công ...
 
Luận Văn Thạc Sĩ Quản trị thành tích nhân viên tại Cục Hải quan TP Đà Nẵng.doc
Luận Văn Thạc Sĩ  Quản trị thành tích nhân viên tại Cục Hải quan TP Đà Nẵng.docLuận Văn Thạc Sĩ  Quản trị thành tích nhân viên tại Cục Hải quan TP Đà Nẵng.doc
Luận Văn Thạc Sĩ Quản trị thành tích nhân viên tại Cục Hải quan TP Đà Nẵng.doc
 
Hoàn thiện công tác quản lý thuế thu nhập cá nhân tại cục thuế Tỉ...
Hoàn thiện công tác quản lý thuế thu nhập cá nhân tại cục thuế Tỉ...Hoàn thiện công tác quản lý thuế thu nhập cá nhân tại cục thuế Tỉ...
Hoàn thiện công tác quản lý thuế thu nhập cá nhân tại cục thuế Tỉ...
 
Đề Tài Phát triển bền vững nông nghiệp Huyện Ba Tơ, Tỉnh Quảng Ngãi....
Đề Tài Phát triển bền vững nông nghiệp Huyện Ba Tơ, Tỉnh Quảng Ngãi....Đề Tài Phát triển bền vững nông nghiệp Huyện Ba Tơ, Tỉnh Quảng Ngãi....
Đề Tài Phát triển bền vững nông nghiệp Huyện Ba Tơ, Tỉnh Quảng Ngãi....
 
Hoàn thiện công tác bảo trợ xã hội trên địa bàn huyện Phong Điền, tỉnh Thừa T...
Hoàn thiện công tác bảo trợ xã hội trên địa bàn huyện Phong Điền, tỉnh Thừa T...Hoàn thiện công tác bảo trợ xã hội trên địa bàn huyện Phong Điền, tỉnh Thừa T...
Hoàn thiện công tác bảo trợ xã hội trên địa bàn huyện Phong Điền, tỉnh Thừa T...
 
Đề Tài Luận VănPhát triển sản phẩm du lịch tại thành phố Đà Nẵng.doc
Đề Tài Luận VănPhát triển sản phẩm du lịch tại thành phố Đà Nẵng.docĐề Tài Luận VănPhát triển sản phẩm du lịch tại thành phố Đà Nẵng.doc
Đề Tài Luận VănPhát triển sản phẩm du lịch tại thành phố Đà Nẵng.doc
 
Đào tạo nghề cho lao động thuộc diện thu hồi đất trên địa bàn Thàn...
Đào tạo nghề cho lao động thuộc diện thu hồi đất trên địa bàn Thàn...Đào tạo nghề cho lao động thuộc diện thu hồi đất trên địa bàn Thàn...
Đào tạo nghề cho lao động thuộc diện thu hồi đất trên địa bàn Thàn...
 
Tóm Tắt Luận Văn Thạc Sĩ Quản Trị Kinh Doanh Xây dựng chính sách Marketing tạ...
Tóm Tắt Luận Văn Thạc Sĩ Quản Trị Kinh Doanh Xây dựng chính sách Marketing tạ...Tóm Tắt Luận Văn Thạc Sĩ Quản Trị Kinh Doanh Xây dựng chính sách Marketing tạ...
Tóm Tắt Luận Văn Thạc Sĩ Quản Trị Kinh Doanh Xây dựng chính sách Marketing tạ...
 
Đề Tài Nghiên cứu rủi ro cảm nhận đối với mua hàng thời trang trực tuyến.docx
Đề Tài Nghiên cứu rủi ro cảm nhận đối với mua hàng thời trang trực tuyến.docxĐề Tài Nghiên cứu rủi ro cảm nhận đối với mua hàng thời trang trực tuyến.docx
Đề Tài Nghiên cứu rủi ro cảm nhận đối với mua hàng thời trang trực tuyến.docx
 
Giải pháp nâng cao động lực thúc đẩy người lao động tại công ty khai...
Giải pháp nâng cao động lực thúc đẩy người lao động tại công ty khai...Giải pháp nâng cao động lực thúc đẩy người lao động tại công ty khai...
Giải pháp nâng cao động lực thúc đẩy người lao động tại công ty khai...
 
Giải pháp phát triển dịch vụ ngân hàng điện tử tại ngân hàng đầu ...
Giải pháp phát triển dịch vụ ngân hàng điện tử tại ngân hàng đầu ...Giải pháp phát triển dịch vụ ngân hàng điện tử tại ngân hàng đầu ...
Giải pháp phát triển dịch vụ ngân hàng điện tử tại ngân hàng đầu ...
 
Giải pháp phát triển dịch vụ ngân hàng điện tử tại ngân hàng đầu ...
Giải pháp phát triển dịch vụ ngân hàng điện tử tại ngân hàng đầu ...Giải pháp phát triển dịch vụ ngân hàng điện tử tại ngân hàng đầu ...
Giải pháp phát triển dịch vụ ngân hàng điện tử tại ngân hàng đầu ...
 
Quản trị quan hệ khách hàng tại Chi nhánh Viettel Đà Nẵng – Tập đoàn Viễn thô...
Quản trị quan hệ khách hàng tại Chi nhánh Viettel Đà Nẵng – Tập đoàn Viễn thô...Quản trị quan hệ khách hàng tại Chi nhánh Viettel Đà Nẵng – Tập đoàn Viễn thô...
Quản trị quan hệ khách hàng tại Chi nhánh Viettel Đà Nẵng – Tập đoàn Viễn thô...
 
Đề Tài Đánh giá thành tích đội ngũ giảng viên trường Đại Học Phạm ...
Đề Tài Đánh giá thành tích đội ngũ giảng viên trường Đại Học Phạm ...Đề Tài Đánh giá thành tích đội ngũ giảng viên trường Đại Học Phạm ...
Đề Tài Đánh giá thành tích đội ngũ giảng viên trường Đại Học Phạm ...
 

Recently uploaded

NHững vấn đề chung về Thuế Tiêu thụ đặc biệt.ppt
NHững vấn đề chung về Thuế Tiêu thụ đặc biệt.pptNHững vấn đề chung về Thuế Tiêu thụ đặc biệt.ppt
NHững vấn đề chung về Thuế Tiêu thụ đặc biệt.pptphanai
 
Chương 6: Dân tộc - Chủ nghĩa xã hội khoa học
Chương 6: Dân tộc - Chủ nghĩa xã hội khoa họcChương 6: Dân tộc - Chủ nghĩa xã hội khoa học
Chương 6: Dân tộc - Chủ nghĩa xã hội khoa họchelenafalet
 
Đề thi tin học HK2 lớp 3 Chân Trời Sáng Tạo
Đề thi tin học HK2 lớp 3 Chân Trời Sáng TạoĐề thi tin học HK2 lớp 3 Chân Trời Sáng Tạo
Đề thi tin học HK2 lớp 3 Chân Trời Sáng Tạowindcances
 
ĐỀ SỐ 1 Của sở giáo dục đào tạo tỉnh NA.pdf
ĐỀ SỐ 1 Của sở giáo dục đào tạo tỉnh NA.pdfĐỀ SỐ 1 Của sở giáo dục đào tạo tỉnh NA.pdf
ĐỀ SỐ 1 Của sở giáo dục đào tạo tỉnh NA.pdflevanthu03031984
 
NHẬN XÉT LUẬN VĂN THẠC SĨ: Các nhân tố ảnh hưởng đến hiệu quả hoạt động của n...
NHẬN XÉT LUẬN VĂN THẠC SĨ: Các nhân tố ảnh hưởng đến hiệu quả hoạt động của n...NHẬN XÉT LUẬN VĂN THẠC SĨ: Các nhân tố ảnh hưởng đến hiệu quả hoạt động của n...
NHẬN XÉT LUẬN VĂN THẠC SĨ: Các nhân tố ảnh hưởng đến hiệu quả hoạt động của n...lamluanvan.net Viết thuê luận văn
 
[123doc] - ao-dai-truyen-thong-viet-nam-va-xuong-xam-trung-quoc-trong-nen-van...
[123doc] - ao-dai-truyen-thong-viet-nam-va-xuong-xam-trung-quoc-trong-nen-van...[123doc] - ao-dai-truyen-thong-viet-nam-va-xuong-xam-trung-quoc-trong-nen-van...
[123doc] - ao-dai-truyen-thong-viet-nam-va-xuong-xam-trung-quoc-trong-nen-van...VnTh47
 
Giới Thiệu Về Kabala | Hành Trình Thấu Hiểu Bản Thân | Kabala.vn
Giới Thiệu Về Kabala | Hành Trình Thấu Hiểu Bản Thân | Kabala.vnGiới Thiệu Về Kabala | Hành Trình Thấu Hiểu Bản Thân | Kabala.vn
Giới Thiệu Về Kabala | Hành Trình Thấu Hiểu Bản Thân | Kabala.vnKabala
 
ĐỀ KIỂM TRA CUỐI KÌ 2 BIÊN SOẠN THEO ĐỊNH HƯỚNG ĐỀ BGD 2025 MÔN TOÁN 10 - CÁN...
ĐỀ KIỂM TRA CUỐI KÌ 2 BIÊN SOẠN THEO ĐỊNH HƯỚNG ĐỀ BGD 2025 MÔN TOÁN 10 - CÁN...ĐỀ KIỂM TRA CUỐI KÌ 2 BIÊN SOẠN THEO ĐỊNH HƯỚNG ĐỀ BGD 2025 MÔN TOÁN 10 - CÁN...
ĐỀ KIỂM TRA CUỐI KÌ 2 BIÊN SOẠN THEO ĐỊNH HƯỚNG ĐỀ BGD 2025 MÔN TOÁN 10 - CÁN...Nguyen Thanh Tu Collection
 
30 ĐỀ PHÁT TRIỂN THEO CẤU TRÚC ĐỀ MINH HỌA BGD NGÀY 22-3-2024 KỲ THI TỐT NGHI...
30 ĐỀ PHÁT TRIỂN THEO CẤU TRÚC ĐỀ MINH HỌA BGD NGÀY 22-3-2024 KỲ THI TỐT NGHI...30 ĐỀ PHÁT TRIỂN THEO CẤU TRÚC ĐỀ MINH HỌA BGD NGÀY 22-3-2024 KỲ THI TỐT NGHI...
30 ĐỀ PHÁT TRIỂN THEO CẤU TRÚC ĐỀ MINH HỌA BGD NGÀY 22-3-2024 KỲ THI TỐT NGHI...Nguyen Thanh Tu Collection
 
Luận văn 2024 Tạo động lực lao động tại Trung tâm nghiên cứu gia cầm Thụy Phương
Luận văn 2024 Tạo động lực lao động tại Trung tâm nghiên cứu gia cầm Thụy PhươngLuận văn 2024 Tạo động lực lao động tại Trung tâm nghiên cứu gia cầm Thụy Phương
Luận văn 2024 Tạo động lực lao động tại Trung tâm nghiên cứu gia cầm Thụy Phươnglamluanvan.net Viết thuê luận văn
 
CÁC NHÂN TỐ ẢNH HƯỞNG ĐẾN HIỆU QUẢ HOẠT ĐỘNG CỦA NGÂN HÀNG THƯƠNG MẠI CỔ PHẦN...
CÁC NHÂN TỐ ẢNH HƯỞNG ĐẾN HIỆU QUẢ HOẠT ĐỘNG CỦA NGÂN HÀNG THƯƠNG MẠI CỔ PHẦN...CÁC NHÂN TỐ ẢNH HƯỞNG ĐẾN HIỆU QUẢ HOẠT ĐỘNG CỦA NGÂN HÀNG THƯƠNG MẠI CỔ PHẦN...
CÁC NHÂN TỐ ẢNH HƯỞNG ĐẾN HIỆU QUẢ HOẠT ĐỘNG CỦA NGÂN HÀNG THƯƠNG MẠI CỔ PHẦN...lamluanvan.net Viết thuê luận văn
 
30 ĐỀ PHÁT TRIỂN THEO CẤU TRÚC ĐỀ MINH HỌA BGD NGÀY 22-3-2024 KỲ THI TỐT NGHI...
30 ĐỀ PHÁT TRIỂN THEO CẤU TRÚC ĐỀ MINH HỌA BGD NGÀY 22-3-2024 KỲ THI TỐT NGHI...30 ĐỀ PHÁT TRIỂN THEO CẤU TRÚC ĐỀ MINH HỌA BGD NGÀY 22-3-2024 KỲ THI TỐT NGHI...
30 ĐỀ PHÁT TRIỂN THEO CẤU TRÚC ĐỀ MINH HỌA BGD NGÀY 22-3-2024 KỲ THI TỐT NGHI...Nguyen Thanh Tu Collection
 
bài tập lớn môn kiến trúc máy tính và hệ điều hành
bài tập lớn môn kiến trúc máy tính và hệ điều hànhbài tập lớn môn kiến trúc máy tính và hệ điều hành
bài tập lớn môn kiến trúc máy tính và hệ điều hànhdangdinhkien2k4
 
TỔNG HỢP HƠN 100 ĐỀ THI THỬ TỐT NGHIỆP THPT HÓA HỌC 2024 - TỪ CÁC TRƯỜNG, TRƯ...
TỔNG HỢP HƠN 100 ĐỀ THI THỬ TỐT NGHIỆP THPT HÓA HỌC 2024 - TỪ CÁC TRƯỜNG, TRƯ...TỔNG HỢP HƠN 100 ĐỀ THI THỬ TỐT NGHIỆP THPT HÓA HỌC 2024 - TỪ CÁC TRƯỜNG, TRƯ...
TỔNG HỢP HƠN 100 ĐỀ THI THỬ TỐT NGHIỆP THPT HÓA HỌC 2024 - TỪ CÁC TRƯỜNG, TRƯ...Nguyen Thanh Tu Collection
 
Giáo trình xây dựng thực đơn. Ths Hoang Ngoc Hien.pdf
Giáo trình xây dựng thực đơn. Ths Hoang Ngoc Hien.pdfGiáo trình xây dựng thực đơn. Ths Hoang Ngoc Hien.pdf
Giáo trình xây dựng thực đơn. Ths Hoang Ngoc Hien.pdf4pdx29gsr9
 
Mở rộng hoạt động cho vay tiêu dùng tại Ngân hàng TMCP Hàng Hải Việt Nam (Mar...
Mở rộng hoạt động cho vay tiêu dùng tại Ngân hàng TMCP Hàng Hải Việt Nam (Mar...Mở rộng hoạt động cho vay tiêu dùng tại Ngân hàng TMCP Hàng Hải Việt Nam (Mar...
Mở rộng hoạt động cho vay tiêu dùng tại Ngân hàng TMCP Hàng Hải Việt Nam (Mar...lamluanvan.net Viết thuê luận văn
 
Tiểu luận tổng quan về Mối quan hệ giữa chu kỳ kinh tế và đầu tư trong nền ki...
Tiểu luận tổng quan về Mối quan hệ giữa chu kỳ kinh tế và đầu tư trong nền ki...Tiểu luận tổng quan về Mối quan hệ giữa chu kỳ kinh tế và đầu tư trong nền ki...
Tiểu luận tổng quan về Mối quan hệ giữa chu kỳ kinh tế và đầu tư trong nền ki...lamluanvan.net Viết thuê luận văn
 
GIỮ GÌN VÀ PHÁT HUY GIÁ TRỊ MỘT SỐ BÀI HÁT DÂN CA CÁC DÂN TỘC BẢN ĐỊA CHO HỌC...
GIỮ GÌN VÀ PHÁT HUY GIÁ TRỊ MỘT SỐ BÀI HÁT DÂN CA CÁC DÂN TỘC BẢN ĐỊA CHO HỌC...GIỮ GÌN VÀ PHÁT HUY GIÁ TRỊ MỘT SỐ BÀI HÁT DÂN CA CÁC DÂN TỘC BẢN ĐỊA CHO HỌC...
GIỮ GÌN VÀ PHÁT HUY GIÁ TRỊ MỘT SỐ BÀI HÁT DÂN CA CÁC DÂN TỘC BẢN ĐỊA CHO HỌC...lamluanvan.net Viết thuê luận văn
 
TỔNG HỢP HƠN 100 ĐỀ THI THỬ TỐT NGHIỆP THPT TIẾNG ANH 2024 - TỪ CÁC TRƯỜNG, ...
TỔNG HỢP HƠN 100 ĐỀ THI THỬ TỐT NGHIỆP THPT TIẾNG ANH 2024 - TỪ CÁC TRƯỜNG, ...TỔNG HỢP HƠN 100 ĐỀ THI THỬ TỐT NGHIỆP THPT TIẾNG ANH 2024 - TỪ CÁC TRƯỜNG, ...
TỔNG HỢP HƠN 100 ĐỀ THI THỬ TỐT NGHIỆP THPT TIẾNG ANH 2024 - TỪ CÁC TRƯỜNG, ...Nguyen Thanh Tu Collection
 

Recently uploaded (20)

NHững vấn đề chung về Thuế Tiêu thụ đặc biệt.ppt
NHững vấn đề chung về Thuế Tiêu thụ đặc biệt.pptNHững vấn đề chung về Thuế Tiêu thụ đặc biệt.ppt
NHững vấn đề chung về Thuế Tiêu thụ đặc biệt.ppt
 
Chương 6: Dân tộc - Chủ nghĩa xã hội khoa học
Chương 6: Dân tộc - Chủ nghĩa xã hội khoa họcChương 6: Dân tộc - Chủ nghĩa xã hội khoa học
Chương 6: Dân tộc - Chủ nghĩa xã hội khoa học
 
Đề thi tin học HK2 lớp 3 Chân Trời Sáng Tạo
Đề thi tin học HK2 lớp 3 Chân Trời Sáng TạoĐề thi tin học HK2 lớp 3 Chân Trời Sáng Tạo
Đề thi tin học HK2 lớp 3 Chân Trời Sáng Tạo
 
ĐỀ SỐ 1 Của sở giáo dục đào tạo tỉnh NA.pdf
ĐỀ SỐ 1 Của sở giáo dục đào tạo tỉnh NA.pdfĐỀ SỐ 1 Của sở giáo dục đào tạo tỉnh NA.pdf
ĐỀ SỐ 1 Của sở giáo dục đào tạo tỉnh NA.pdf
 
Trích dẫn theo Harvard với Microsoft Word
Trích dẫn theo Harvard với Microsoft WordTrích dẫn theo Harvard với Microsoft Word
Trích dẫn theo Harvard với Microsoft Word
 
NHẬN XÉT LUẬN VĂN THẠC SĨ: Các nhân tố ảnh hưởng đến hiệu quả hoạt động của n...
NHẬN XÉT LUẬN VĂN THẠC SĨ: Các nhân tố ảnh hưởng đến hiệu quả hoạt động của n...NHẬN XÉT LUẬN VĂN THẠC SĨ: Các nhân tố ảnh hưởng đến hiệu quả hoạt động của n...
NHẬN XÉT LUẬN VĂN THẠC SĨ: Các nhân tố ảnh hưởng đến hiệu quả hoạt động của n...
 
[123doc] - ao-dai-truyen-thong-viet-nam-va-xuong-xam-trung-quoc-trong-nen-van...
[123doc] - ao-dai-truyen-thong-viet-nam-va-xuong-xam-trung-quoc-trong-nen-van...[123doc] - ao-dai-truyen-thong-viet-nam-va-xuong-xam-trung-quoc-trong-nen-van...
[123doc] - ao-dai-truyen-thong-viet-nam-va-xuong-xam-trung-quoc-trong-nen-van...
 
Giới Thiệu Về Kabala | Hành Trình Thấu Hiểu Bản Thân | Kabala.vn
Giới Thiệu Về Kabala | Hành Trình Thấu Hiểu Bản Thân | Kabala.vnGiới Thiệu Về Kabala | Hành Trình Thấu Hiểu Bản Thân | Kabala.vn
Giới Thiệu Về Kabala | Hành Trình Thấu Hiểu Bản Thân | Kabala.vn
 
ĐỀ KIỂM TRA CUỐI KÌ 2 BIÊN SOẠN THEO ĐỊNH HƯỚNG ĐỀ BGD 2025 MÔN TOÁN 10 - CÁN...
ĐỀ KIỂM TRA CUỐI KÌ 2 BIÊN SOẠN THEO ĐỊNH HƯỚNG ĐỀ BGD 2025 MÔN TOÁN 10 - CÁN...ĐỀ KIỂM TRA CUỐI KÌ 2 BIÊN SOẠN THEO ĐỊNH HƯỚNG ĐỀ BGD 2025 MÔN TOÁN 10 - CÁN...
ĐỀ KIỂM TRA CUỐI KÌ 2 BIÊN SOẠN THEO ĐỊNH HƯỚNG ĐỀ BGD 2025 MÔN TOÁN 10 - CÁN...
 
30 ĐỀ PHÁT TRIỂN THEO CẤU TRÚC ĐỀ MINH HỌA BGD NGÀY 22-3-2024 KỲ THI TỐT NGHI...
30 ĐỀ PHÁT TRIỂN THEO CẤU TRÚC ĐỀ MINH HỌA BGD NGÀY 22-3-2024 KỲ THI TỐT NGHI...30 ĐỀ PHÁT TRIỂN THEO CẤU TRÚC ĐỀ MINH HỌA BGD NGÀY 22-3-2024 KỲ THI TỐT NGHI...
30 ĐỀ PHÁT TRIỂN THEO CẤU TRÚC ĐỀ MINH HỌA BGD NGÀY 22-3-2024 KỲ THI TỐT NGHI...
 
Luận văn 2024 Tạo động lực lao động tại Trung tâm nghiên cứu gia cầm Thụy Phương
Luận văn 2024 Tạo động lực lao động tại Trung tâm nghiên cứu gia cầm Thụy PhươngLuận văn 2024 Tạo động lực lao động tại Trung tâm nghiên cứu gia cầm Thụy Phương
Luận văn 2024 Tạo động lực lao động tại Trung tâm nghiên cứu gia cầm Thụy Phương
 
CÁC NHÂN TỐ ẢNH HƯỞNG ĐẾN HIỆU QUẢ HOẠT ĐỘNG CỦA NGÂN HÀNG THƯƠNG MẠI CỔ PHẦN...
CÁC NHÂN TỐ ẢNH HƯỞNG ĐẾN HIỆU QUẢ HOẠT ĐỘNG CỦA NGÂN HÀNG THƯƠNG MẠI CỔ PHẦN...CÁC NHÂN TỐ ẢNH HƯỞNG ĐẾN HIỆU QUẢ HOẠT ĐỘNG CỦA NGÂN HÀNG THƯƠNG MẠI CỔ PHẦN...
CÁC NHÂN TỐ ẢNH HƯỞNG ĐẾN HIỆU QUẢ HOẠT ĐỘNG CỦA NGÂN HÀNG THƯƠNG MẠI CỔ PHẦN...
 
30 ĐỀ PHÁT TRIỂN THEO CẤU TRÚC ĐỀ MINH HỌA BGD NGÀY 22-3-2024 KỲ THI TỐT NGHI...
30 ĐỀ PHÁT TRIỂN THEO CẤU TRÚC ĐỀ MINH HỌA BGD NGÀY 22-3-2024 KỲ THI TỐT NGHI...30 ĐỀ PHÁT TRIỂN THEO CẤU TRÚC ĐỀ MINH HỌA BGD NGÀY 22-3-2024 KỲ THI TỐT NGHI...
30 ĐỀ PHÁT TRIỂN THEO CẤU TRÚC ĐỀ MINH HỌA BGD NGÀY 22-3-2024 KỲ THI TỐT NGHI...
 
bài tập lớn môn kiến trúc máy tính và hệ điều hành
bài tập lớn môn kiến trúc máy tính và hệ điều hànhbài tập lớn môn kiến trúc máy tính và hệ điều hành
bài tập lớn môn kiến trúc máy tính và hệ điều hành
 
TỔNG HỢP HƠN 100 ĐỀ THI THỬ TỐT NGHIỆP THPT HÓA HỌC 2024 - TỪ CÁC TRƯỜNG, TRƯ...
TỔNG HỢP HƠN 100 ĐỀ THI THỬ TỐT NGHIỆP THPT HÓA HỌC 2024 - TỪ CÁC TRƯỜNG, TRƯ...TỔNG HỢP HƠN 100 ĐỀ THI THỬ TỐT NGHIỆP THPT HÓA HỌC 2024 - TỪ CÁC TRƯỜNG, TRƯ...
TỔNG HỢP HƠN 100 ĐỀ THI THỬ TỐT NGHIỆP THPT HÓA HỌC 2024 - TỪ CÁC TRƯỜNG, TRƯ...
 
Giáo trình xây dựng thực đơn. Ths Hoang Ngoc Hien.pdf
Giáo trình xây dựng thực đơn. Ths Hoang Ngoc Hien.pdfGiáo trình xây dựng thực đơn. Ths Hoang Ngoc Hien.pdf
Giáo trình xây dựng thực đơn. Ths Hoang Ngoc Hien.pdf
 
Mở rộng hoạt động cho vay tiêu dùng tại Ngân hàng TMCP Hàng Hải Việt Nam (Mar...
Mở rộng hoạt động cho vay tiêu dùng tại Ngân hàng TMCP Hàng Hải Việt Nam (Mar...Mở rộng hoạt động cho vay tiêu dùng tại Ngân hàng TMCP Hàng Hải Việt Nam (Mar...
Mở rộng hoạt động cho vay tiêu dùng tại Ngân hàng TMCP Hàng Hải Việt Nam (Mar...
 
Tiểu luận tổng quan về Mối quan hệ giữa chu kỳ kinh tế và đầu tư trong nền ki...
Tiểu luận tổng quan về Mối quan hệ giữa chu kỳ kinh tế và đầu tư trong nền ki...Tiểu luận tổng quan về Mối quan hệ giữa chu kỳ kinh tế và đầu tư trong nền ki...
Tiểu luận tổng quan về Mối quan hệ giữa chu kỳ kinh tế và đầu tư trong nền ki...
 
GIỮ GÌN VÀ PHÁT HUY GIÁ TRỊ MỘT SỐ BÀI HÁT DÂN CA CÁC DÂN TỘC BẢN ĐỊA CHO HỌC...
GIỮ GÌN VÀ PHÁT HUY GIÁ TRỊ MỘT SỐ BÀI HÁT DÂN CA CÁC DÂN TỘC BẢN ĐỊA CHO HỌC...GIỮ GÌN VÀ PHÁT HUY GIÁ TRỊ MỘT SỐ BÀI HÁT DÂN CA CÁC DÂN TỘC BẢN ĐỊA CHO HỌC...
GIỮ GÌN VÀ PHÁT HUY GIÁ TRỊ MỘT SỐ BÀI HÁT DÂN CA CÁC DÂN TỘC BẢN ĐỊA CHO HỌC...
 
TỔNG HỢP HƠN 100 ĐỀ THI THỬ TỐT NGHIỆP THPT TIẾNG ANH 2024 - TỪ CÁC TRƯỜNG, ...
TỔNG HỢP HƠN 100 ĐỀ THI THỬ TỐT NGHIỆP THPT TIẾNG ANH 2024 - TỪ CÁC TRƯỜNG, ...TỔNG HỢP HƠN 100 ĐỀ THI THỬ TỐT NGHIỆP THPT TIẾNG ANH 2024 - TỪ CÁC TRƯỜNG, ...
TỔNG HỢP HƠN 100 ĐỀ THI THỬ TỐT NGHIỆP THPT TIẾNG ANH 2024 - TỪ CÁC TRƯỜNG, ...
 

Một số phương pháp tìm cực trị của các hàm phân thức Sinh bởi số tự nhiên.docx

  • 1. ĐẠI HỌC THÁI NGUYÊN TRƢỜNG ĐẠI HỌC KHOA HỌC --------------------------- Tải tài liệu tại sividoc.com Viết đề tài giá sinh viên – ZALO:0973.287.149-TEAMLUANVAN.COM NGUYỄN KIM THU MỘT SỐ PHƢƠNG PHÁP TÌM CỰC TRỊ CỦA CÁC HÀM PHÂN THỨC SINH BỞI SỐ TỰ NHIÊN LUẬN VĂN THẠC SĨ TOÁN HỌC THÁI NGUYÊN - 2018
  • 2. ĐẠI HỌC THÁI NGUYÊN TRƢỜNG ĐẠI HỌC KHOA HỌC --------------------------- Tải tài liệu tại sividoc.com Viết đề tài giá sinh viên – ZALO:0973.287.149-TEAMLUANVAN.COM NGUYỄN KIM THU MỘT SỐ PHƢƠNG PHÁP TÌM CỰC TRỊ CỦA CÁC HÀM PHÂN THỨC SINH BỞI SỐ TỰ NHIÊN Chuyên ngành: Phƣơng pháp Toán sơ cấp Mã số: 8 46 01 13 LUẬN VĂN THẠC SĨ TOÁN HỌC NGƯỜI HƯỚNG DẪN KHOA HỌC (Xác nhận) GS.TSKH. Nguyễn Văn Mậu THÁI NGUYÊN - 2018
  • 3. i Viết đề tài giá sinh viên – ZALO:0973.287.149-TEAMLUANVAN.COM Mnc lnc M ĐAU iii 1 Phân thfíc hfiu t v i h so nguyên 1 1.1. Tính chat cơ bản của đa thác với h so nguyên ..................... 1 1.2. Phân thác hǎu tỉ với h so nguyên và phân thác nh n giá trị hǎu tỉ................................................................................4 1.3. Bieu dien đơn vị thành tőng của các phân so Ai C p với mau so nguyên dương đ c bi t ............................................... 8 2 Các phương pháp giải toán cfic trị dạng phân thfíc sinh b i so hfiu t 12 2.1. M®t so phương pháp giải bài toán cực trị của đa thác và phân thác hǎu t với h so nguyên....................................... 12 2.1.1. Phương pháp so sánh b c hai.................................... 12 2.1.2. Phương pháp so sánh phân thác dạng b c hai trên b c nhat....................................................................15 2.1.3. Phương pháp tìm cực trị với ràng bu®c theo tőng các so ...............................................................................21 2.2. Sả dụng phân thác chính quy giải các bài toán cực trị liên quan.......................................................................................27 3 M t so dạng toán liên quan 32 3.1. M®t so dạng toán cực trị trên t p so nguyên....................... 32 3.2. M®t so dạng toán ve so tự nhiên tà các đe thi Olympic ........38 KET LU N 44
  • 4. ii Viết đề tài giá sinh viên – ZALO:0973.287.149-TEAMLUANVAN.COM TÀI LI U THAM KHẢO 45
  • 5. iii Viết đề tài giá sinh viên – ZALO:0973.287.149-TEAMLUANVAN.COM M ĐAU Chuyên đe ve đa thác và phân thác là m®t chuyên đe rat quan trong ở b c trung hoc phő thông và trung hoc cơ sở. Các tính chat của đa thác và phân thác liên quan ch t chě với các tính chat của so nguyên và so hǎu t . M®t trong các phương pháp khảo sát đa thác và phân thác hǎu t rat hǎu hi u là vi c sả dụng các công cụ hǎu ích tà vi c khảo sát các tính chat so hoc của các so tự nhiên và so hǎu t . Trong các kì thi hoc sinh giỏi toán các cap, các bài toán liên quan tới đa thác và phân thác với h so nguyên (ta goi chung là phân thác sinh bởi so tự nhiên) thường xuyên được đe c p. Nhǎng dạng toán này thường được xem là thu®c loại khó can các kien thác sâu sac ve so hoc ket hợp với các kien thác ve đa thác và phân thác thường không nam trong chương trình chính thong của chương trình toán b c trung hoc phő thông. Đe đáp áng nhu cau boi dương giáo viên và boi dương hoc sinh giỏi ve chuyên đe đa thác và phân thác với h so nguyên và h so hǎu t , em chon đe tài lu n văn “M®t so phương pháp tìm cực trị của các hàm phân thác sinh bởi so tự nhiên”. Mục tiêu của lu n văn nham h thong m®t so kien thác ve so hoc và đa thác với h so nguyên và cung cap m®t so phương pháp tìm cực trị của các hàm phân thác sinh bởi so tự nhiên. Tiep theo, xét các bài toán cực trị, khảo sát phương trình, bat phương trình cùng m®t so dạng liên quan. Lu n văn gom phan mở đau, ket lu n và ba chương. Chương 1. Phân thác hǎu t với h so nguyên. Chương 2. Các phương pháp giải toán cực trị dạng phân thác sinh
  • 6. iv Viết đề tài giá sinh viên – ZALO:0973.287.149-TEAMLUANVAN.COM bởi so hǎu t . Chương 3. M®t so dạng toán liên quan. Tiep theo, trong các chương đeu trình bày h thong các bài t p áp dụng và giải các đe thi HSG quoc gia và Olympic liên quan.
  • 7. 1 Viết đề tài giá sinh viên – ZALO:0973.287.149-TEAMLUANVAN.COM ∈ Chương 1 Phân thfíc hfiu t v i h so nguyên 1.1. Tính chat cơ bản của đa thfíc v i h so nguyên Trong phan này, trình bày m®t so tính chat cơ bản của đa thác với h so nguyên. Định nghĩa 1.1 (xem [1]-[2]) Cho L ⊂ R. Đa thác P(x) ∈ L[x] được goi là khả quy trên L[x] neu ton tại đa thác Q(x) và T (x) cùng thu®c L[x] với các b c lớn hơn 0 sao cho P (x) = Q(x).T (x). Trong trường hợp ngược lại thì được goi là bat khả quy trên L[x]. Định nghĩa 1.2 (xem [1]-[2]) T p hợp tat cả các đa thác khả quy trên L[x] được ký hi u là L∗ [x]. Tính chat 1.1 Moi đa thác P (x) ∈ R[x] với b c lớn hơn bang 2 đeu phân tích được thành nhân tả b c nhat và nhân tả b c hai nên cũng có the coi P(x) ∈ R∗ [x]. Định nghĩa 1.3 (xem [1]-[2]) Đa thác thu®c Z[x] được goi là đa thác nguyên bản neu b® các h so của nó nguyên to cùng nhau (có the không đôi m®t nguyên to cùng nhau). Tính chat 1.2 Neu f(x) Z[x] thì ton tại duy nhat m®t đa thác a nguyên bản và m®t phân so toi giản , a ∈ Z, b ∈ N∗ sao cho f(x) = a b b f1(x). Bo đe 1.1 (Bő đe Gauss) Tích của hai đa thúc nguyên bản là m®t đa thúc nguyên bản.
  • 8. 2 Viết đề tài giá sinh viên – ZALO:0973.287.149-TEAMLUANVAN.COM Σ m m 1 j!(m − j)! 1 2 α1!.α2! 1 2 1 (m − αn)!.αn! n ChGng minh. Cho hai đa thác nguyên bản P (x) = anxn an−1xn−1 + · · · + a1x + a0 và Q(x) = bmxm + bm−1xm−1 + · · · + b1x + b0 thì P (x).Q(x) = cm+nxm+n + cm+n−1xm+n−1 + · · · + c1x + c0. Giả sả tích trên không nguyên bản thì ton tại m®t so nguyên to p là ước chung của các h so c0, c1, . . . , cm+n. Vì P nguyên bản nêu goi i là so nhỏ nhat mà ai không chia het cho p và j là so nhỏ nhat sao cho bj không chia het cho p. Khi đó, xét cj+i ta thay h so tương áng không chia het cho p, vô lý. V y tích trên nguyên bản. Tính chat 1.3 Neu đa thác P(x) ∈ Z[x], deg P > 1 mà không thu®c Z∗ [x] thì nó cũng không thu®c Q∗ [x]. Định lj 1.1 (xem [1]-[2]) Cho các đa thúc f(x) = anxn + an−1xn−1 + · · · + a1x + a0 ∈ Z[x], an 0, a, b là hai so nguyên khác nhau. Khi đó, f(a) − f(b).(a − b). Bo đe 1.2 (Khai trien Newton) Cho n và m là các so nguyên dương. Với bat kỳ x = (x1, x2, · · · , xn) trong Rn , ta có (x1 + x2 + · · · + xn )m = |α|=m m! xα , (1.1) α! trong đó α! = α1!α2! · · · αn! với α = (α1, α2, · · · , αn) trong Nn , xα = xα1 xα2 . . . xαn và tőng chạy qua tat cả α có the có trong Nn thóa mãn 1 2 n |α| = α1 + α2 + · · · + αn = m. ChGng minh. Với n = 2 theo nhị thác Newton, ta có (x +x )m = Σ m! xj xm−j = Σ m! xα1 xα2 , α = j, α = m−j. Giả sả đȁng thác (1.18) đã đúng cho đen n − 1. Đ t X = x1 + x2 + .. + xn−1, α′ = (α1, α2, . . . , αn). Khi đó ta có m (x + x + · · · + x )m = (X + x )m = Σ m! Xm−αn xαn αn=0 |α|=m j=0 2 1 2 2 n n
  • 9. 3 Viết đề tài giá sinh viên – ZALO:0973.287.149-TEAMLUANVAN.COM m Σ 1 2 1 1 1 αn=0 |α′=m−αn = Σ m! xαn .(x + x + · · · + x )m−αn αn=0 m (m − αn)!.αn! n−1 = Σ m! xαn . Σ (m − αn)! xα′ = Σ m! xα1 xα2 ............ xαn . |α|=m Bő đe được cháng minh. α1!α2! . . . αn! 1 2 n Định lj 1.2 (Khai trien Taylor, xem [1]-[2]) Cho m®t đa thúc n f(x) = ajxj . (1.2) j=0 Khi đó, h so thú j của f(x) có the được bieu dien bới a = 1 f(j) (0), (1.3) j j! trong đó f(j) (0) úng với đạo hàm cap j tại 0 Bo đe 1.3 Cho n là m®t so nguyên dương. Ta đ¾t g(x) = Khi đó g(n) (0) = n!. ChGng minh. Ta có x + x2 + · · · + n xn n . (1.4) g(x) = xn 1 1 + 2 x + · · · + n xn−1 n = xn h(x), (1.5) trong đó h(x) = 1 1 + 2 x + · · · + n xn−1 n . (1.6) Áp dụng công thác Leibniz 2 n (m − αn)!.αn! n α1!α2! . . . αn−1! 1
  • 10. 4 Viết đề tài giá sinh viên – ZALO:0973.287.149-TEAMLUANVAN.COM n → ∈ g(n) (x) = Σ n! d n−j xn . d j h(x) = Do đó, ta thu được j=0 Σ j=0 (n − j)!j! n!n! (n − j)!j!j! dx xj . dx d j h(x). dx (1.7) g(n) (0) = n!h(0) = n!. (1.8) Bő đe được cháng minh. 1.2. Phân thfíc hfiu tỉ v i h so nguyên và phân thfíc nh n giá trị hfiu tỉ Tiep theo, ta nhac lại m®t so tính chat của phân thác hǎu tỉ với h so nguyên và các dạng phân thác nh n giá trị hǎu tỉ trên t p so tự nhiên. Định nghĩa 1.4 Hàm so f: R R được goi là phân thác hǎu tỉ neu P (x) ton tại các đa thác P(x), Q(x) sao cho f (x) = Q (x) (1). Khi P(x) và Q(x) là các đa thác nguyên to cùng nhau thì (1) được goi là phân thác hǎu tỉ chính tac. Neu đa thác P(x) và Q(x) là các đa thác có h so hǎu tỉ thì bang P′ (x) vi c quy đong mau so ta sě đưa f(x) ve dạng f (x) = Q′ (x) , trong đó P′(x) và Q′(x) là các đa thác có h so nguyên. Do v y phân thác hǎu P (x) tỉ f (x) = Q (x) được goi là phân thác hǎu tỉ có h so nguyên neu như P(x), Q(x) ∈ Q[x]. Bài toán 1.1 Cho phân thác hǎu tỉ f (x) = Cháng minh rang a, b Q. 1 1 ax + b ∈ Q với moi x ∈ Z. 1 Lài giai. Vì f (x) = ax + b ∈ Q với moi x ∈ Z nên ax + b = với f (x) moi x ∈ Z. V y ax + b ∈ Q[x] hay a, b ∈ Q. n
  • 11. 5 Viết đề tài giá sinh viên – ZALO:0973.287.149-TEAMLUANVAN.COM Bài toán 1.2 Cháng minh rang neu f (x) = C 1 ax + b ∈ Q với moi x ∈ Z thì f(x) có dạng f(x) = Lài giai. Vì f (x) = Ax + B với A, B, C thu®c Z 1 ax + b ∈ Q với moi x ∈ Z nên ta có a, b ∈ Q ⇒ a = m ,b = n e f (m, n, e, f ∈ Z). Khi đó 1 f(x) = m e x + nf = = mfx + ne C . Ax + B n f ax + b Bài toán 1.3 Cho phân thác hǎu tỉ f (x) = cx + d ∈ Q với moi x ∈ Z. Ax + B Cháng minh rang f(x) có the bieu dien dưới dạng f (x) = với A, B, C, D ∈ Z. Cx + D (1) Lài giai. Neu ad − bc = 0 thì f(x) = const nên bieu dien (1) là hien nhiên. Xét trường hợp ad − bc 0: Neu c = 0 thì bieu dien (1) là hien nhiên. Neu c /= 0 thì sả dụng phân tích f(x) − f(0) = dạng bieu dien (1). 1 ta sě được αx + β Nh n xét rang ket quả của bài toán cũng đúng trong trường hợp bài toán tőng quát. Bài toán 1.4 Cháng minh rang neu phân thác f(x) = ax + b cx + d ∈ Q, ∀x ∈ Z thì f(x) có dạng f(x) = Ax + B Cx + D trong đó A, B, C, D là các so nguyên. M Q(x) − Q(1) e Q(x) − N Lài giai. Ta có E = x − 1 cx + d ∈ Q ∀x ∈ Z nên = x − 1 . Cx + D Ta có M, N, E, C, D là các so nguyên nên de dàng suy ra đpcm. Bài toán 1.5 Cháng minh rang neu ax2 + bx + c f(x) = dx + e ∈ Q
  • 12. 6 Viết đề tài giá sinh viên – ZALO:0973.287.149-TEAMLUANVAN.COM / D với moi x ∈ Z thì f(x) có dạng f(x) = thu®c Z. Ax2 + Bx + C với A, B, C, D, E Dx + E Lài giai. Ta có Q(x) − Q (1) px + q = ∈ Q ∀x ∈ Z. Suy ra M Q(x) − N x − 1 M′x + N′ cx + d ′ ′ ′′ ′′ = x − 1 tà đó ta suy ra đpcm. , (M, N, M , N , M , N M′′x + N′′ ∈ Z) Bài toán 1.6 Cháng minh rang neu ax2 + bx + c f(x) = dx + e ∈ Q với moi x ∈ Z thì f(x) có dạng f(x) = Cx +D ho c f(x) = Ax2 +Bx+ C ∈ Z với moi x ∈ Z, trong đó C, D thu®c Z. Lài giai. Neu d = 0 thì ta có ngay f(x) = Ax2 + Bx + C ∈ Z với moi x ∈ Z. Neu d = 0 khi đó ta có Q(x) − Q(0) x = a1x + b1 dx + e ∈ Z với moi x ∈ Z. Suy ra Q(x) − M A1 A2 = + ∈ Z, ∀x ∈ Z, trong đó A1, A2, D, E, M ∈ Z ⇒ A2 = 0 ⇒ A1.D nên ta có đpcm. Bài toán 1.7 Cho phân thác hǎu tỉ f(x) = P (x) Q(x) ∈ Q ∀x ∈ Z, (P(x), Q(x)) = 1. Cháng minh rang f(x) có the bieu dien được dưới dạng phân thác của hai đa thác với h so nguyên. ChGng minh. Giả sả: P(x) = a0 + a1x + · · · + amxm Q(x) = b0 + b1x + · · · + bnxn Tại x = j (j = 0, 1, . . . , m + n) hàm f (x) nh n các giá trị hǎu tỉ tương áng là cj. Khi đó ta có h phương trình tuyen tính với m + n + 2 ȁn: a0, a1, . . . , am, b0, b1, . . . , bn dạng a0 + a1k + · · · + amkm − b0ck − b1ckk − · · · − bnckkn = 0 trong đó k = 0, 1, . . . , m + n. Hai nghi m của h này cho ta hai c p đa x Dx + E
  • 13. 7 Viết đề tài giá sinh viên – ZALO:0973.287.149-TEAMLUANVAN.COM Q − Q . . i Q . . . . j . . − j . . p . . − p . . − p . . Xét hi u x + qi − 1 + qi = (x + qi) (1 + qi) ≤ 0 do đó .x + qk . ≤ − k=j+1 thác P (x), Q(x) và P1(x), Q1(x) có tính chat P (k) − ckQ(k) = 0, P1(k) − ckQ1(k) = 0, ∀k = 0, m + n. Hai c p nghi m này cho ta đa thác g(x) = P (x)Q1(x) − P1(x)Q(x), degg(x) ≤ m + n nh n giá trị 0 tại m+n+1 điem nên g(x) ≡ 0. Do P(x) và Q(x) nguyên to cùng nhau nên P (x) = cP1(x); Q(x) = cQ1(x). V y h đã cho chỉ có m®t nghi m với sự sai khác m®t thàa so, nghi m đã nh n được là các so hǎu tỉ. Bài toán 1.8 Cho p là m®t so nguyên dương, q ∈ [0, 1]. Giả sả x ∈ qp+1, 1 và f(x) = p k=1 x qk x + qk . Cháng minh rang |f(x)| ≤ p k=1 1 − qk 1 + qk Lài giai. Ta có 0 < qp+1 < qp < · · · < q < 1. Với qj+1 ≤ x ≤ qj thì khi i .x − qi . x − qi i ≥ j + 1 ta có x ≥ q nên = x + q x + qi . x − qi 1 − qi 2qi (x − 1) Q p .x − qk . p k=j+1 1 − qk .1 + qk . Với k = 1, 2, . . . , j ta có x − qj−(k−1) .x + qj−(k−1) . qj−(k−1) x . x + qj−(k−1) Ta lại có qj−(k−1) − x 1 − qk 2 qj−(k−1) − x nên x + qj−(k−1) − 1 + qk = (1 + qk) x + qj−(k−1) ≤ 0 Tà đó ta được k Y =1 x qk .x + qk . ≤ k Y =1 1 − qk .1 + qk . k Y =1 x qk .x + qk . ≤ k Y =1 1 qk .1 + qk . ⇔ |f(x)| ≤ k Y =1 1 − qk .1 + qk . . . . =
  • 14. 8 Viết đề tài giá sinh viên – ZALO:0973.287.149-TEAMLUANVAN.COM 1.3. Bieu dien đơn vị thành tong của các phân so Ai C p v i mȁu so nguyên dương đ c bi t Ta xét các cách bieu dien của 1 như là tőng của các phân so đơn vị. Ví dụ, ta có the dùng 2, 3 và 6 cho và 3, 4, 4, 8,và 24 cho 1 1 1 + + 2 3 6 = 1 (1.9) 1 1 1 + + 3 4 4 1 1 + + 8 24 = 1. (1.10) Ta đã biet rang m®t phân so tùy ý luôn có the được viet thành tőng của các phân so đơn vị. Tiep theo, đe có các bieu dien này, ta khảo sát lời giải của phương trình Diophantine dạng 1 1 + x1 x2 1 + · · · + x = 1, (1.11) trong đó các ȁn xj là các so nguyên không nhat thiet phải phân bi t với j = 1, 2, · · · , n. Bảng 1:Tő hợp có the có của α với n = 6 α1 α2 α3 α4 α5 α6 Mau so 6 0 0 0 0 0 6! = 720 4 1 0 0 0 0 4!.2 = 48 3 0 1 0 0 0 3!.3 = 18 2 2 0 0 0 0 2!.2!.22 = 16 2 0 0 1 0 0 2!.4 = 8 1 1 1 0 0 0 2.3 = 6 1 0 0 0 5 0 5 0 3 0 0 0 0 3!.23 = 48 0 1 0 1 0 0 2.4 = 8 0 0 2 0 0 0 2!.32 = 18 0 0 0 0 0 1 6 Đe giải thích cho ket quả áng với n = 6, ta xét các tő hợp có the có n
  • 15. 9 Viết đề tài giá sinh viên – ZALO:0973.287.149-TEAMLUANVAN.COM Y Σ Y 1 Σ 2 1 x x n Σ 2 n 1 2 1 của α = (α1, α2, α3, α4, α5, α6), αj ∈ N, (1.12) thỏa mãn đieu ki n α1 + 2α2 + 3α3 + 4α4 + 5α5 + 6α6 = 6. (1.13) Tiep theo, xét đại lượng 6 αj!jαj = α1!1α1 .α!.2α2 . · · · .α6!.6α6 (1.14) j=1 Tính tőng của các nghịch đảo của các đại lượng trong bảng so trong Bảng 1, ta thu được 1 + 720 1 1 1 1 + + + 48 18 16 8 1 1 + + + 6 5 1 1 + + 48 8 1 1 + 18 6 = 1. (1.15) Tà ví dụ trên, ta phát bieu ket quả sau. Định lj 1.3 (xem [1]) Với moi so nguyên dương n, ta luôn có đȁng thúc n αj!jαj = 1, (1.16) α∈Sn j=1 trong đó tőng trên Sn chạy qua tat cả α = (α1, α2, · · · , αn) có the có trong Nn thóa mãn đieu ki n ChGng minh. ta có n jαj = n. (1.17) j=1 x1 + x2 + · · · + m n n |α|=m m! xα1 . α! n 1 α2 2 2 . · · · . 1 αn n n = m! Σ Y 1 x jαj . |α|=m j=1 αj!jαj j (1.18) Cho m = n và xj = t với j = 1, 2, · · · , n dan đen t + t2 + · · · + n tn n = n! | Σ α|=n Y j=1 1 αj!jαj tjαj . (1.19) x 1 2 1 n =
  • 16. 10 Viết đề tài giá sinh viên – ZALO:0973.287.149-TEAMLUANVAN.COM Σ Y 1 Σ Y 1 So sánh h so của tn trong cả hai đong nhat thác. ta thu được n! đoi với ve trái. M t khác h so tn trong ve phải là tőng của tat cả so hạng với tn mà được viet bởi n , (1.20) αj!jαj α∈Sn j=1 trong đó tőng chạy qua tat cả α có the có trong Sn xác định bởi Sn = {α ∈ Nn ; α1 + 2α2 + · · · + nαn = n} . (1.21) Do đó, ta thu được n αj!jαj = 1 (1.22) α∈Sn j=1 và ta có đieu phải cháng minh. Bài toán 1.9 Xét phương trình vô định sau đây α1 + 2α2 + 3α3 + 4α4 = 4. Tính tőng nghịch đảo các đại lượng Πα = α1!1α1 .α2!2α2 .α3!3α3 .α4!4α4 , trong đó α = (α1, α2, α3, α4) là các nghi m nguyên không âm phân bi t của phương trình đã cho. Lài giai. De thay rang phương trình đã cho có các nghi m nguyên không âm chỉ là α = (α1, α2, α3, α4) = (4, 0, 0, 0), (0, 0, 0, 1), (1, 0, 1, 0), (0, 2, 0, 0), (2, 1, 0, 0). Tiep theo, tính các đại lượng Πα = α1!1α1 .α2!2α2 .α3!3α3 .α4!4α4 , α = (α1, α2, α3, α4). Ta có Π1 = 4!14 .0!20 .0!30 .0!40 = 24, Π2 = 0!10 .0!20 .0!30 .1!41 = 4,
  • 17. 11 Viết đề tài giá sinh viên – ZALO:0973.287.149-TEAMLUANVAN.COM Suy ra Π3 = 1!11 .0!20 .1!31 .0!40 = 3, Π4 = 0!10 .2!22 .0!30 .0!40 = 8, Π5 = 2!12 .1!21 .0!30 .0!40 = 4. 1 1 + Π1 Π2 1 1 + + Π3 Π4 1 1 1 1 1 = + + + + 24 4 3 8 4 = 1. Bài toán 1.10 Xét phương trình vô định sau đây α1 + 2α2 + 3α3 + 4α4 + 5α5 = 5. Tính tőng nghịch đảo các đại lượng Πα = α1!1α1 .α2!2α2 .α3!3α3 .α4!4α4 .α5!5α5 , trong đó α = (α1, α2, α3, α4, α5) là các nghi m nguyên không âm phân bi t của phương trình đã cho. Lài giai. De thay rang phương trình đã cho có các nghi m nguyên không âm chỉ là α = (α1, α2, α3, α4, α5) = (5, 0, 0, 0, 0), (0, 0, 0, 0, 1), (0, 1, 1, 0, 0), (1, 2, 0, 0, 0), (2, 0, 1, 0, 0), (1, 0, 0, 1, 0), (3, 0, 1, 0, 0). Tiep theo, ta tính các đại lượng Πα = α1!1α1 .α2!2α2 .α3!3α3 .α4!4α4 .α5!5α5 , α = (α1, α2, α3, α4, α5). Ta có Suy ra Π1 = 5!15 .0!20 .0!30 .0!40 .0!.50 = 120, Π2 = 0!10 .0!20 .0!30 .1!40 .1!51 = 5, Π3 = 0!11 .1!21 .1!31 .0!40 .0!50 = 6, Π4 = 1!11 .2!22 .0!30 .0!40 .0!50 = 8, Π5 = 2!12 .0!20 .1!31 .0!40 .0!50 = 4, Π6 = 1!11 .0!20 .0!30 .1!41 .0!50 = 4, Π7 = 3!13 .0!20 .1!31 .0!40 .0!50 = 18. 1 1 + Π1 Π2 1 1 1 + + + Π3 Π4 Π5 1 1 + + Π6 Π7 1 1 = + + 120 5 1 1 1 + + + 6 8 4 1 1 + 4 18 = 1.
  • 18. 12 Viết đề tài giá sinh viên – ZALO:0973.287.149-TEAMLUANVAN.COM Chương 2 Các phương pháp giải toán cfic trị dạng phân thfíc sinh b i so hfiu t 2.1. M t so phương pháp giải bài toán cfic trị của đa thfíc và phân thfíc hfiu t v i h so nguyên Trong phan này ta khảo sát lớp phân thác hǎu t với h so nguyên. Đe giải các bài toán cực trị tương áng ta xét m®t so phương pháp đ c thù như so sánh b c hai và so sánh dạng dạng phân thác b c hai trên b c nhat tőng quát. 2.1.1. Phương pháp so sánh b c hai Ta đã biet, đoi với dạng tam thác b c hai f(x) = ax2 + bx + c, a > 0. ta luôn có f(x) ≥ f(y) + (2ax + b)(x − y), ∀x, y ∈ R. (∗) Th t v y, ta thay f(x) ≥ f(y) + (2ax + b)(x − y) ⇔ a(x − y)2 ≥ 0, ∀x, y ∈ R. Sả dụng phương pháp so sánh b c hai, ta giải quyet được các dạng toán cực trị ở các nút nguyên trên trục toa đ® mà các phương pháp truyen thong không thực hi n được.
  • 19. 13 Viết đề tài giá sinh viên – ZALO:0973.287.149-TEAMLUANVAN.COM Bài toán 2.1 Xét các so tự nhi n a, b, c thỏa mãn đieu ki n a ≥ 9 a + b ≥ 11 a + b + c ≥ 12. Tìm giá trị nhỏ nhat của bieu thác M = a2 + b2 + c2 . Lài giai. Áp dụng bat đȁng thác (*) áng với f(x) = x2 , ta thu được a2 ≥ 92 + 2 × 9(a − 9), ∀a ∈ N. Tương tự, ta cũng có b2 ≥ 22 + 2 × 2(b − 2), ∀b ∈ N và Tà đó, suy ra c2 ≥ 1 + 2(c − 1), ∀c ∈ N. M = a2 + b2 + c2 ≥ 92 + 22 + 1 + 2[9(a − 9) + 2(b − 2) + (c − 1)]. M t khác, theo giả thiet thì 9(a−9)+2(b−2)+(c−1) = (a−9+b−2+c−1)+(a−9+b−2)+7(a−9) ≥ 0. Suy ra M ≥ 92 + 22 + 1 = 86 nên min M = 86 khi a = 9, b = 2, c = 1. Bài toán 2.2 Xét các so tự nhi n a, b, c thỏa mãn đieu ki n a ≥ 9 a + b ≥ 11 a + b + c ≥ 12. Tìm giá trị nhỏ nhat của bieu thác M = f(a) + f(b) + f(c), trong đó f(x) = 3x2 + 2x − 10.
  • 20. 14 Viết đề tài giá sinh viên – ZALO:0973.287.149-TEAMLUANVAN.COM Lài giai. Áp dụng bat đȁng thác (*), ta có f(a) ≥ f(9) + (6 × 9 + 2)(a − 9), ∀a ∈ N. Tương tự, ta cũng có f(b) ≥ f(2) + (6 × 2 + 2)(b − 2), ∀a ∈ N. và Tà đó, suy ra f(a) ≥ f(9) + (6 × 1 + 2)(c − 1), ∀a ∈ N. M = f(a)+f(b)+f(c) ≥ f(9)+f(2)+f(1)+[(6×9+2)(a−9)+(6×2+2)(b−2)+(6×1+2 M t khác, theo giả thiet thì (6×9+2)(a−9)+(6×2+2)(b−2)+(6×1+2)(c−1) = 8(a−9+b−2+c−1)+6(a−9+b−2)+ Suy ra M ≥ f(9) + f(2) + f(1) = 237 nên min M = 237 khi a = 9, b = 2, c = 1. Bài toán 2.3 Xét các so tự nhi n a, b, c thỏa mãn đieu ki n c ≤ b ≤ a ≤ 9 a + b ≤ 11 a + b + c ≤ 12. Tìm giá trị lớn nhat của bieu thác M = a2 + b2 + c2 . Lài giai. Áp dụng bat đȁng thác (*) áng với f(x) = x2 , ta thu được 92 ≥ a2 + 2 × a(9 − a), ∀a ∈ N. Tương tự, ta cũng có 22 ≥ b2 + 2 × b(2 − b), ∀a ∈ N. và 1 ≥ c2 + 2 × 1(1 − c), ∀a ∈ N.
  • 21. 15 Viết đề tài giá sinh viên – ZALO:0973.287.149-TEAMLUANVAN.COM ≤ ∞ Tà đó, suy ra M = a2 + b2 + c2 ≤ 92 + 22 + 1 − 2[a(9 − a) + b(2 − b) + c(1 − c)]. M t khác, theo giả thiet thì a(9−a)+b(2−b)+c(1−c) = (a−b)(9−a)+(b−c)(9−a+2−b)+c(9−a+2−b+1−c) ≥ 0. Suy ra M ≤ 92 + 22 + 1 = 86 nên max M = 86 khi a = 9, b = 2, c = 1. 2.1.2. Phương pháp so sánh phân thfíc dạng b c hai trên b c nhat Trước het ta xét bài toán ve xác định các khoảng (α, β) với (−∞ ≤ α < β + ) sao cho áng với moi hàm so b c hai g(x) = ax2 + bx + g(x) c, a > 0, hàm so có dạng f(x) = thác x − d , g(d) /= 0, ta đeu có bat đȁng ho c f(u) f′(v) ≥ f(u) f (v) f′(v) + u − v, ∀u, v ∈ (α, β), u f(v) v (∗∗) f′(v) ≤ f′(v) + u − v, ∀u, v ∈ (α, β), u = / v (∗ ∗ ∗) và xét các áp dụng liên quan. Ý tưởng chính của phương pháp là sả dụng phép so sánh phân thác b c hai trên b c nhat tőng quát. Đe ý rang, trong hau het các dạng toán cực trị có ràng bu®c theo tőng ho c tích trên t p so tự nhiên thì dau đȁng thác xảy ra không ở b® so bang nhau. Vì v y, ta can đen m®t công cụ hǎu hi u là định lý Karamata cho các b® so được sap thá tự. Trước het, ta nhac lại bat đȁng thác Karamata [1-3]. Định lj 2.1 (Bat đȁng thúc Karamata) Xét hai dãy so {xk, yk ∈ (a, b), k = 1, 2, . . . , n}, thóa mãn đieu ki n x1 ≥ x2 ≥ · · · ≥ xn, y1 ≥ y2 ≥ · · · ≥ yn
  • 22. 16 Viết đề tài giá sinh viên – ZALO:0973.287.149-TEAMLUANVAN.COM x1 + x2 ≥ y1 + y2 và x1 ≥ y1 . . . . . . x1 + x2 + · · · + xn−1 ≥ y1 + y2 + · · · + yn−1 x1 + x2 + · · · + xn = y1 + y2 + · · · + yn Khi đó, úng với moi hàm so loi khả vi ϕ(x)(ϕ′′(x) ≥ 0) trên (α, β), ta đeu có ϕ(x1) + ϕ(x2) + · · · + ϕ(xn) ≥ ϕ(y1) + ϕ(y2) + · · · + ϕ(yn). Đȁng thúc xảy ra khi xi = yi, i = 1, 2, . . . n. Ta cũng phát bieu tương tự đoi với hàm so lõm bang cách đői chieu dau bat đȁng thác. Nh n xét 2.1 Neu hàm so y = ϕ(x) liên tục và có đạo hàm cap 2 trên (α, β) và ϕ′′(x) ≥ 0, ∀x ∈ (α, β) thì ϕ(x) ≥ ϕ′(x0)(x − x0) + ϕ(x0), ∀x, x0 ∈ (α, β). Neu ϕ′′(x) ≤ 0, ∀x ∈ I(a; b) thì ϕ(x) ≤ ϕ′(x0)(x − x0) + ϕ(x0), ∀x, x0 ∈ (α, β). Đȁng thác trong hai bat đȁng thác trên xảy ra khi x = x0. Nh n xét rang đe x − d 0 với moi x ∈ (α, β) thì d ∈ / (α, β). Đieu này tương đương với ho c d ≤ α ho c d ≥ β. Tà đó suy ra H quả 2.1 Đieu ki n can đe có bat đȁng thác (∗∗) với moi u, v ∈ (α, β) và u /= v là ho c d ≤ α ho c d ≥ β. Tiep theo, ta xác định đieu ki n đủ đe có (∗∗) . Ta có f(x) = ax + ad + b + và g(d) x − d f′(x) = a − g(d) , f′′(x) = 2g(d) . (2.1) (x − d)2 (x − d)3
  • 23. 17 Viết đề tài giá sinh viên – ZALO:0973.287.149-TEAMLUANVAN.COM a Trường hợp 1. Xét g(d) < 0. Khi đó f′(x) > 0 với moi x f′′(x) > 0 khi x > d và f′′(x) < 0 khi x < d. V y nên khi x < d thì f′′(x) < 0 và f′(x) > 0. d. Ta có Theo Bő đe cơ bản, ta có (∗ ∗ ∗) luôn đúng áng với moi u, v < d, u /= v. Khi x > d thì f′′(x) > 0 và f′(x) > 0. và (∗∗) luôn đúng với moi u, v > d, u /= v. Trường hợp 2. Xét g(d) > 0. Khi đó f′′(x) < 0 khi x > d và f′′(x) > 0 khi x < d. Xét phương trình f′(x) = 0 ⇔ x1,2 = d ∓ r g(d) , x < d < x2. Khi đó f′(x) > 0 khi x > x2 ho c x < x1 và f′(x) < 0 ⇔ x1 < x < x2. V y nên - khi x < β ≤ x1 thì f′(x) > 0 và f′′(x) > 0. Khi đó (∗∗) đúng với moi u, v ∈ (α, β), - khi x > α ≥ x2 thì f′(x) > 0 và f′′(x) < 0. Khi đó (∗ ∗ ∗) đúng với moi u, v ∈ (α, β), - khi x ∈ (x1, d) thì f′(x) < 0 và f′′(x) > 0. Khi đó (∗ ∗ ∗) đúng với moi u, v ∈ (x1, d), - khi x ∈ (d, x2) thì f′(x) < 0 và f′′(x) < 0. Khi đó (∗∗) đúng với moi u, v(d, x2). V y ta có the phát bieu các ket quả nh n được như sau. g(x) Định lj 2.2 Giả sủ f(x) = với g(x) = ax2 + bx + c, a > 0 và g(d) < 0. x − d Khi đó, đieu ki n can và đủ (đoi với (α, β)) đe có bat đȁng thúc (∗∗) là α ≥ d và đieu ki n can và đủ (đoi với (α, β)) đe có bat đȁng thúc (∗ ∗ ∗) là β ≤ d. Tương tự, ta cũng có ket quả sau g(x) Định lj 2.3 Giả sủ f(x) = với g(x) = ax2 + bx + c, a > 0 và g(d) < 0 x − d Khi đó, đieu ki n can và đủ (đoi với (α, β)) đe có bat đȁng thúc (∗∗) là α ≤ x1 ho¾c d ≤ α < β < x2, tương tự, đieu ki n can và đủ (đoi với 1
  • 24. 18 Viết đề tài giá sinh viên – ZALO:0973.287.149-TEAMLUANVAN.COM Σ − Σ − ≥ r Σ − Σ − ≤ − − − (α, β)) đe x1,2 = d ∓ có bat đȁng thúc (∗ ∗ ∗) là α ≥ x2 ho¾c x1 ≤ α < β ≤ d với g(d) . a Tà các định lý 2.2-2.3, ta phát bieu các h quả đe sả dụng trong các bài toán áp dụng ở phan sau khi b® so gom các so nguyên. g(x) H quả 2.2 Giả sủ f(x) = x − d với g(x) = ax2 + bx + c, a > 0 có g(d) < 0 và α ≥ d ho¾c g(d) > 0 và α ≤ x1 ho¾c d ≤ α < β < x2. Khi đó, với moi b® so u1, u2, u1 ∈ (α, β) và v1, v2, v3 ∈ (α, β) sao cho u1 + u2 + u3 = v1 + v2 + v3, ta đeu có g(uk) 3 g(vk) 3 uk d g(d) vk d . g(d) k=1 a − (v — d)2 k=1 a − (v — d)2 g(x) H quả 2.3 Giả sủ f(x) = x − d với g(x) = ax2 + bx + c, a > 0 có g(d) < 0 và β ≤ d. ho¾c g(d) > 0 và α ≥ x2 ho¾c x1 ≤ α < β ≤ d với g(d) x1,2 = d ∓ a . Khi đó, với moi b® so u1, u2, u1 ∈ (α, β) và v1, v2, v3 ∈ (α, β) sao cho u1 + u2 + u3 = v1 + v2 + v3, ta đeu có g(uk) 3 g(vk) 3 uk d g(d) vk d . g(d) k=1 a − (v — d)2 k=1 a − (v — d)2 Bài toán 2.4 Xét các so x, y, z ≥ 1 có x + y + z = 10. Cháng minh bat đȁng thác 9(x 1)2 + 4(x + 1) Lài giai. Xét hàm so 4(y 1)2 + 3(y + 1) 9(z 1)2 8(z + 1) ≥ 157 . 30 (x − 1)2 4 Ta có f(x) = = x − 3 + . x + 1 x + 1 f′ (x) = (x − 1)(x + 3) (x + 1)2 ≥ 0, ∀x ≥ 1 r k k k k
  • 25. 19 Viết đề tài giá sinh viên – ZALO:0973.287.149-TEAMLUANVAN.COM 9 , − − − − − − 4 − − = x − 3 + . và f′′(x) = 8 (x + 1)3 > 0, ∀x ≥ 1. Theo h quả 2.2, ta có f(u) f′(v) ≥ f (v) f′(v) + u − v, ∀u, v ∈ [1, +∞), u = / v (∗∗) Chon 3 so v ∈ [1, +∞) sao cho f′(v) ∈ ,4 ; 3 ; 8 và có tőng bang 10. 9 Ta thu được v1 = 2, v2 = 3, v3 = 5. Tiep theo, the vào (∗∗), ta được hay 9(x 1)2 4(x + 1) ≥ 4(y 1)2 3(y + 1) ≥ 9(z 1)2 8(z + 1) ≥ 9(x 1)2 4(x + 1) ≥ 4(y 1)2 3(y + 1) ≥ 9(z 1)2 8(z + 1) ≥ f(2) f′(2) + x − 2, ∀x ∈ [1, +∞) f(3) f′(3) + y − 3, ∀y ∈ [1, +∞) f(5) f′(5) + z − 2, ∀z ∈ [1, +∞) 1/2 5/9 + x − 2, ∀x ∈ [1, +∞) 1 3/4 + y − 3, ∀y ∈ [1, +∞) 8/3 8/9 + z − 2, ∀z ∈ [1, +∞) C®ng các ve tương áng ta thu được 9(x − 1)2 4(y − 1)2 9(z − 1)2 1/2 1 8/3 157 + 4(x + 1) đpcm. + 3(y + 1) 8(z + 1) ≥ + + = , 5/9 3/4 8/9 30 Bài toán 2.5 Xét các so x, y, z ∈ (−1, 1) có x + y + z = 1. Tìm giá trị lớn nhat của bieu thác M := 9(x − 1)2 5(x + 1) 4(y 1)2 + 5(y + 1) 49(z 1)2 + . 95(z + 1) Lài giai. Xét hàm so f(x) = (x − 1)2 4 x + 1 x + 1
  • 26. 20 Viết đề tài giá sinh viên – ZALO:0973.287.149-TEAMLUANVAN.COM 5 5 95 1 1 1 Ta có và f′ (x) = (x − 1)(x + 3) (x + 1)2 < 0, ∀x ∈ (−1, 1) f′′(x) = 8 (x + 1)3 > 0, ∀x ∈ (−1, 1). Theo h quả 2.3, ta có f(u) f′(v) ≤ f (v) f′(v) + u − v, ∀u, v ∈ (−1, 1), u v (∗ ∗ ∗) Chon 3 so v ∈ (−1, 1) sao cho f′(v) ∈ , − 9 ; − 4 ; − 49, và có tőng bang 1. Ta thu được v1 = ta được 2 ,v2 = 3 ,v3 = 6 . Tiep theo, the vào (∗ ∗ ∗), 9(x − 1)2 f(1/2) 1 5(x + 1) ≥ f′(1/2) + x − 2 , ∀x ∈ (−1, 1) 4(y − 1)2 f(1/3) 1 5(y + 1) ≥ f′(1/3) + y − 3 , ∀y ∈ (−1, 1) 49(z − 1)2 f(1/6) 1 hay 95(z + 1) ≥ f′(1/6) + z − 6 , ∀z ∈ (−1, 1) 9(x − 1)2 9 1 — 5(x + 1) ≥ − 30 + x − 2 , ∀x ∈ (−1, 1) 4(y − 1)2 4 1 — 5(y + 1) ≥ − 15 + y − 3 , ∀y ∈ (−1, 1) 49(z − 1)2 35 1 hay — 95(z + 1) ≥ − 114 + z − 6 , ∀z ∈ (−1, 1) 9(x − 1)2 9 1 5(x + 1) ≤ 30 − x + 2 , ∀x ∈ (−1, 1) 4(y − 1)2 4 1 5(y + 1) ≤ 15 − y + 3 , ∀y ∈ (−1, 1) 49(z − 1)2 35 1 95(z + 1) ≤ 114 − z + 6 , ∀z ∈ (−1, 1)
  • 27. 21 Viết đề tài giá sinh viên – ZALO:0973.287.149-TEAMLUANVAN.COM ≤ − − − C®ng các ve tương áng ta thu được 9(x 1)2 M = 5(x + 1) 4(y 1)2 + 5(y + 1) 49(z 1)2 + 95(z + 1) 9 4 35 V y max M = 9 4 + + 30 5 ≤ 35 114 + + . 30 5 114 đạt được khi x = 1 , y = 2 1 1 , z = . 3 6 2.1.3. Phương pháp tìm cfic trị v i ràng bu c theo tong các so Đoi với các bài toán tìm cực trị của m®t so bieu thác với ràng bu®c theo tőng của các so tự nhiên, ta thường sả dụng phương pháp làm đeu. Bài toán 2.6 Cho các so nguyên dương a, b, c có a + b + c = 100. Tìm giá trị lớn nhat của M = abc. Lài giai. đây, vì ba so a, b, c là các so nguyên dương nên tà giả thiet suy ra chúng không the bang nhau. Giả sả a ≥ b ≥ c thì a ≥ 34. Áp dụng bat đȁng thác AM-GM cho b® so 33a, 34b, 34c, ta thu được √ 3 33a.34b.34c 33a + 34b + 34c 3 = 34 × 100 − a 3 34 × 100 − 34 3 = 33 × 34. Suy ra M ≤ 34 × 332 . V y max M = 34 × 332 . Tiep theo, ta xét bài toán tìm cực tieu. Bài toán 2.7 Cho các so nguyên dương a, b, c có a + b + c = 100. Tìm giá trị nhỏ nhat của M = abc. Lài giai. Ta cháng minh min M = 98. Giả sả a ≥ b ≥ c thì a ≥ 34. Neu c ≥ 2 thì b ≥ 2 nên abc ≥ 34 × 2 × 2 > 98. ≤
  • 28. 22 Viết đề tài giá sinh viên – ZALO:0973.287.149-TEAMLUANVAN.COM a(c + a) b(a + b) Xét c = 1. Khi đó a + b = 99 và a ≥ 50. Neu b ≥ 2 thì abc ≥ 50 × 2 × 1 > 98. Khi b = 1 thì a = 98 và abc = 98. V y nên min M = 98. Bài toán 2.8 Cho các so nguyên dương a, b, c có a + b + c = 9. Cháng minh rang ab bc ca 3 Lài giai. Đ t c(c + b) + a(a + c) + b(a + b) ≥ 2 . ab M = c(c + b) bc + a(a + c) ca + . b(a + b) Theo bat đȁng thác AM-GM, ta có r ab + r bc + r ac 2 c a = s ab b √ b + c + s bc √ c + a + r ac √ a + b 2 ≤ h ab + bc + ca i [2(a + b + c)] = M[2(a + b + c)]. M t khác cũng theo bat đȁng thác AM-GM, thì (a + b + c)2 ≥ 3(ab + bc + ca) nên r ab + r bc + r ac 2 ≥ 3(a + b + c). Suy ra M ≥ c a b 3 , hay 2 ab bc ca 3 c(c + b) + a(a + c) + b(a + b) ≥ 2 . Dau đȁng thác xảy ra khi a = b = c = 3. Bài toán 2.9 Cho x, y, z là các so nguyên thỏa mãn đieu ki n x+y+z = 6. Cháng minh rang 8x + 8y + 8z 4x+1 + 4y+1 + 4z+1 ≥ 1. c(b + c) c(c + b) a(a + c) b(a + b)
  • 29. 23 Viết đề tài giá sinh viên – ZALO:0973.287.149-TEAMLUANVAN.COM 3 3 2 3 Lài giai. Đ t 2x = a, 2y = b, 2z = c, (a, b, c > 0) ⇒ abc = 64. Đieu can cháng minh trở thành a3 + b3 + c3 ≥ 4a2 + 4b2 + 4c2 . Áp dụng bat đȁng thác AM-GM ta được: a3 + b3 + c3 ≥ 3.abc = 3.64 a3 + a3 + 64 ≥ 12a2 b3 + b3 + 64 ≥ 12b2 c3 + c3 + 64 ≥ 12c2 3 3 2 2 2 ⇒ 3(a + b + c ) + 3.64 ≥ 3.64 + 12(a + b + c ) 3 2 2 ⇔ a + b + c ≥ 4(a + b + c ) (đieu phải cháng minh). Dau đȁng thác xảy ra khi a = b = c = 4 khi đó x = y = z = 2. Bài toán 2.10 Với ba so nguyên dương a, b, c thỏa mãn đieu ki n a + b + c = 9. Cháng minh rang a b c 1 9 + b2 + 9 + c2 + 9 + a2 ≥ 2 . Lài giai. Áp dụng bat đȁng thác AM-GM, ta có 9a ab2 ab2 ab 9 + b2 = a − 9 + b2 ≥ a − 6b = a − 6 . Tương tự, ta có 9b bc 9 + c2 ≥ b − 6 9c ac 9 + a2 ≥ c − 6 . C®ng ve với ve ba bat đȁng thác trên ta có 9a 9b 9c 1 ab + bc + ca 9 + b2 + 9 + c2 + 9 + a2 ≥ a+b+c− 6 (ab+bc+ca) = 9− 6 . Ta lại có Do đó ab + bc + ca ≤ (a + b + c)2 = 27. 3 9a 9b 9c 1 9 9 + b2 + 9 + c2 + 9 + a2 ≥ 9 − 6 (ab + bc + ca) ≥ 2 . a b c 1 ⇒ 9 + b2 + 9 + c2 + 9 + a2 ≥ 2 . Đȁng thác xảy ra khi và chỉ khi a = b = c = 3.
  • 30. 24 Viết đề tài giá sinh viên – ZALO:0973.287.149-TEAMLUANVAN.COM Σ − − − Bài toán 2.11 Cho m,n là hai so nguyên dương với m ≥ n ≥ 2. Giả sả x1; x2; . . . xn là các so nguyên dương có tőng bang m. Tìm giá trị lớn 1 nhat và giá trị nhỏ nhat của bieu thác S = n . kxk k=1 Lài giai. Vì có hǎu hạn b® so (x1, x2, . . . xn) thỏa mãn đieu ki n của bài toán nên ta có hǎu hạn giá trị của S. Do đó ton tại giá trị nhỏ nhat N và giá trị lớn nhat L của S. Ta cháng minh L đạt được khi xn = m + 1 − n, x1 = x2 = · · · = xn−1 = 1. Th t v y, giả sả L đạt được với b® so (x1, x2, . . . xn) mà xn ≤ m+1−n. Khi đó ton tại xi > 1 (1 ≤ i ≤ n − 1). Đ t x′ i = xi − 1, x′ n = xn + 1, x′ k = xk (k /= n; i; 1 ≤ k ≤ n − 1) . Khi đó b® so x1 ′ ; x2 ′ ; . . . x′ n thỏa mãn đieu ki n của bài toán. Do ix′ i + nx′ n suy ra = i (xi − 1) + n (xn + 1) = ixi + nxn + n − i > ixi + nxn n n Σ kx′ k > Σ kxk. k=1 Đieu này trái với giả thiet của L. V y k=1 L = n(n − 1) + n (m + 1 n) = 2 2mn + n n2 . 2 Cháng minh tương tự, ta có N đạt được khi x1 = m + 1 − n, x2 = x3 = · · · = xn = 1 hay (n2 + 2m n) N = . 2 Bài toán 2.12 Cho x, y ∈ N∗ ; x + y = 2n + 1 với n là hang so nguyên dương cho trước. Tìm giá trị lớn nhat và giá trị lớn nhat của bieu thác P = x!y! Lài giai. Ta cháng minh bieu thác P đạt giá trị lớn nhat L = (2n)! và giá trị nhỏ nhat N = n!(n + 1)!
  • 31. 25 Viết đề tài giá sinh viên – ZALO:0973.287.149-TEAMLUANVAN.COM − − Th t v y giả sả N đạt được với b® so (x; y) thỏa mãn đieu ki n của bài toán áng với x < n. Đ t x′ = x + 1; y′ = y − 1 Khi đó, ta có b® so(x′; y′) thỏa mãn đieu ki n của bài toán. Và x′!y′! = (x + 1)!(y 1)! = x!y! x + 1 y < x!y! = N vì x + 1 < n + 1 < y Trái với giả thiet của N. V y giá trị nhỏ nhat N = n!(n + 1)! Giả sả L đạt được với b® so (x; y) thỏa mãn đieu ki n của bài toán mà 1 < x < y. Đ t x′ = x − 1; y′ = y + 1. Khi đó, ta có b® so (x′; y′) thỏa mãn đieu ki n của bài toán và x′!y′! = (x 1)!(y + 1)! = x!y! y + 1 x > x!y! = L do y + 1 > y > x, trái với giả thiet. V y giá trị lớn nhat L = (2n)! Bài toán 2.13 Xét phương trình vô định sau đây α1 + 2α2 + 3α3 + 4α4 = 4. Tìm giá trị nhỏ nhat của tőng các nghịch đảo của các đại lượng Πα = α1!1α1 .α2!2α2 .α3!3α3 .α4!4α4 , trong đó α = (α1, α2, α3, α4) là các nghi m nguyên không âm phân bi t của phương trình đã cho. Lài giai. De thay rang phương trình đã cho có các nghi m nguyên không âm chỉ là α = (α1, α2, α3, α4) = (4, 0, 0, 0), (0, 0, 0, 1), (1, 0, 1, 0), (0, 2, 0, 0), (2, 1, 0, 0). Tiep theo, tính các đại lượng Πα = α1!1α1 .α2!2α2 .α3!3α3 .α4!4α4 , α = (α1, α2, α3, α4).
  • 32. 26 Viết đề tài giá sinh viên – ZALO:0973.287.149-TEAMLUANVAN.COM Ta có Suy ra Π1 = 4!14 .0!20 .0!30 .0!40 = 24, Π2 = 0!10 .0!20 .0!30 .1!41 = 4, Π3 = 1!11 .0!20 .1!31 .0!40 = 3, Π4 = 0!10 .2!22 .0!30 .0!40 = 8, Π5 = 2!12 .1!21 .0!30 .0!40 = 4. T := 1 1 + Π1 Π2 1 1 + + Π3 Π4 1 1 = + 24 4 1 1 1 + + + 3 8 4 = 1. V y nên min T = 1. Bài toán 2.14 Xét phương trình vô định sau đây α1 + 2α2 + 3α3 + 4α4 + 5α5 = 5. Tìm giá trị nhỏ nhat của tőng các nghịch đảo của các đại lượng Πα = α1!1α1 .α2!2α2 .α3!3α3 .α4!4α4 .α5!5α5 , trong đó α = (α1, α2, α3, α4, α5) là các nghi m nguyên không âm phân bi t của phương trình đã cho. Lài giai. De thay rang phương trình đã cho có các nghi m nguyên không âm chỉ là α = (α1, α2, α3, α4, α5) = (5, 0, 0, 0, 0), (0, 0, 0, 0, 1), (0, 1, 1, 0, 0), (1, 2, 0, 0, 0), (2, 0, 1, 0, 0), (1, 0, 0, 1, 0), (3, 0, 1, 0, 0). Tiep theo, ta tính các đại lượng Πα = α1!1α1 .α2!2α2 .α3!3α3 .α4!4α4 .α5!5α5 , α = (α1, α2, α3, α4, α5). Ta có Π1 = 5!15 .0!20 .0!30 .0!40 .0!.50 = 120, Π2 = 0!10 .0!20 .0!30 .1!40 .1!51 = 5, Π3 = 0!11 .1!21 .1!31 .0!40 .0!50 = 6, Π4 = 1!11 .2!22 .0!30 .0!40 .0!50 = 8,
  • 33. 27 Viết đề tài giá sinh viên – ZALO:0973.287.149-TEAMLUANVAN.COM Σ Σ Σ ∈ ∈ Π5 = 2!12 .0!20 .1!31 .0!40 .0!50 = 4, Π6 = 1!11 .0!20 .0!30 .1!41 .0!50 = 4, Π7 = 3!13 .0!20 .1!31 .0!40 .0!50 = 18. Suy ra 1 S := Π1 1 1 1 + + + Π2 Π3 Π4 1 1 1 + + + Π5 Π6 Π7 1 = + 120 1 1 1 + + 5 6 8 1 1 1 + + + 4 4 18 = 1. V y nên min S = 1. 2.2. Sfi dnng phân thfíc chính quy giải các bài toán cfic trị liên quan Định nghĩa 2.1 Hàm f(x) xác định trên t p các so thực dương, không đong nhat với 0 được goi là phân thác chính quy neu f(x) = Σ n n k=1 akx α k (1) Ví dn 2.1 De dàng kiem cháng các hàm so sau đây là phân thác chính quy: f (x) = 1 + 2x + x2 + 1 + 1 ; f (x) = 2x2 + 3x3 + 1 + 2 . 1 x x3 1 x5 x4 Định nghĩa 2.2 Hàm f(x) xác định trên t p các so thực dương, không đong nhat với 0 được goi là phân thác chính quy hǎu tỉ neu f(x) = n k=1 akx α n k (1) trong đó ak N, αk Z, k = 1, n; k=1 akαk = 0. Tà định nghĩa, de dàng ta suy ra các tính chat sau: Tính chat 2.1 Neu f (x) là hàm phân thác chính quy, thì f (x) > 0 áng với moi x > 0. Tính chat 2.2 Neu f (x) và g(x) là các hàm phân thác chính quy, thì với moi c p so dương α, β hàm so h(x) = αf(x) + βg(x) cũng là hàm phân thác chính quy. Tính chat 2.3 Neu f (x) và g(x) là các hàm phân thác chính quy, thì hàm so h(x) = f(g(x)) cũng là hàm phân thác chính quy. k=1 trong đó ak ≥ 0, k = 1, n; akαk = 0.
  • 34. 28 Viết đề tài giá sinh viên – ZALO:0973.287.149-TEAMLUANVAN.COM Σ t¾p x1, . . . , xm > 0 dạng f(x1, x2, . . . , xm) = đó akx k1 x k2 . . . x km m Σ Tính chat 2.4 Neu f(x) là hàm phân thác chính quy, thì hàm so h(x) = [f(x)]m , (m ∈ N∗ ) cũng là hàm phân thác chính quy. Định nghĩa 2.3 Hàm so f(x1, x2, . . . , xm) không đong nhat 0, xác định trên t p các so thực dương, được goi là hàm phân thác chính quy hǎu n tỉ m bien neu f(x1, x2, . . . , xm) = akx α k1 xαk2 . . . xαkm trong đó 1 2 m k=1 ak ∈ N, αkj ∈ Z, k = 1, n, j = 1, m a1α11 + a2α21 + · · · + anαn1 = 0 a1α12 + a2α22 + · · · + anαn2 = 0 . . . a1α1n + a2α2n + · · · + anαnm = 0 Tà định nghĩa trên, ta có các tính chat sau: Tính chat 2.5 Hàm so f(x1, x2, . . . , xm) là hàm phân thác chính quy hǎu tỉ khi và chỉ khi các hàm phân thác thành phan của f(1, 1, . . . , xk, 1, . . . , 1) cũng là hàm phân thác chính quy hǎu tỉ. Ví dn 2.2 De dàng kiem tra được hàm so f(x, y) = 3x3 y8 + 4x4 y9 + 5 x5y12 chính quy, và các hàm so là các phân thác thành phan là: f1(x) = 3x3 + 4x4 + 5x−5 ; f2(y) = 3x8 + 4x9 + 5x−12 . Định nghĩa 2.4 Hàm so f (x1, x2, . . . , xm) là hàm phân thác chính quy khi và chỉ khi các hàm phân thác thành phan của f (x1, x2, . . . , xm) cũng là các hàm phân thác chính quy. Định lj 2.4 Với mői hàm phân thúc f(x1, x2, . . . , xm) chính quy trên n α α α ak ∈ N, αkj ∈ Z, k = 1, n, j = 1, m a1α11 + a2α21 + · · · + anαn1 = 0 a1α12 + a2α22 + · · · + anαn2 = 0 . . . a1α1n + a2α2n + · · · + anαnm = 0 2 1 k=1 trong
  • 35. 29 Viết đề tài giá sinh viên – ZALO:0973.287.149-TEAMLUANVAN.COM Σ Σ Σ Σ Σ Σ Σ Σ q Σ Σ Σ αk − p = ak p = q − p. p = 0. Σ q q Σ Σ ta đeu có f(x1, . . . , xm) ≥ n k=1 ak. ChGng minh. Áp dụng bat đȁng thác AM-GM, ta có n akxαk1 xαk2 . . . xαkm f(x1, x2, . . . , xm) 1 2 m = k=1 a1 + a2 + . . . + an a1 + a2 + . . . + an n ≥ x1k=1 akαk1 n x2k=1 akαk2 n . . . xmk=1 akαkn 1 a1+a2+...+an = 1 n do k=1 akαkj = 0, j = 1, m . Dau "=" xảy ra khi và chỉ khi x1 = x2 = · · · = xm = 1. H quả 2.4 Với mői hàm phân thúc f(x1, x2, . . . , xm) chính quy trên t¾p x1, . . . , xm > 0 ta đeu có: min f(x1, x2, . . . , xm) = f(1, 1, . . . , 1). Nh n xét 2.2 Với moi hàm phân thác dạng g(x) = n k=1 akx α k , ak ≥ 0, n n đ t ak = p, akαk = q thì hàm so k=1 k=1 f(x) = x−p g(x) là m®t hàm phân thác chính quy. ChGng minh. Ta có n n f(x) = x− g(x) = x− a xαk = a xαk− q nên Σ p q Σ p k k=1 Σ k p k=1 q q Tà đó ta thu được định lý quan trong sau đây. n Định lj 2.5 Moi hàm phân thúc dạng g(x) = k=1 akx α k , ak ≥ 0, k = 1, n q đeu có tính chat g(x) ≥ g(1)xp , ∀x > 0 trong đó n k=1 ak = p, akαk − k=1 a k ! n n n k=1 k=1
  • 36. 30 Viết đề tài giá sinh viên – ZALO:0973.287.149-TEAMLUANVAN.COM n k=1 ak αk = q.
  • 37. 31 Viết đề tài giá sinh viên – ZALO:0973.287.149-TEAMLUANVAN.COM q ChGng minh. Theo nh n xét trên, ta có f (x) = x− p g(x) là m®t phân thác chính quy nên ta suy ra f(x) ≥ f(1) mà f(1) = g(1) nên ta có đpcm. Bài toán 2.15 Cháng minh rang các hàm so dạng f(x) = a + bx + cx2 + 2 cx b + 3x3 với a, b, c nguyên dương luôn đạt giá trị nhỏ nhat tại so nguyên dương. Lài giai. Đe ý rang, f(x) là phân thác chính quy m®t bien áng với moi a, b, c nguyên dương nên min f(x) = f(1). Bài toán 2.16 Cháng minh rang các hàm so dạng g(x) = 2x2 + 3x3 + 1 x5 2 + x4 . Lài giai. Đe ý rang, g(x) là phân thác chính quy m®t bien nên min g(x) = g(1).. Bài toán 2.17 Cho các so thực α1, α2, . . . , α9. Tìm giá trị nhỏ nhat của bieu thác f(x) = xα1 + 2xα2 + 3xα3 + · · · + 9xα9 , x ∈ R+ . Lài giai. Ta có f(x) xα1 + 2xα2 + 3xα3 + · · · + 9xα9 α1 2α2 9α9 45 = 45 ≥ x 45 x 45 . . . x 45 = 1 ⇔ f(x) ≥ 45. V y min f(x) = 45 khi và chỉ khi x = 1. Bài toán 2.18 Cháng minh rang hàm so f(x, y) = 3x3 y8 + 4x4 y9 + 5 x5y12 đạt giá trị nhỏ nhat tại c p so tự nhiên trên t p R+ × R+ .. Lài giai. Đe ý rang, f(x, y) là phân thác chính quy hai bien, và các hàm so là các phân thác thành phan là:
  • 38. 32 Viết đề tài giá sinh viên – ZALO:0973.287.149-TEAMLUANVAN.COM f1(x) = 3x3 + 4x4 + 5x−5 ; f2(y) = 3x8 + 4x9 + 5x−12 . Tà đó, suy ra giá trị nhỏ nhat của f(x, y) trên t p R+ × R+ . đạt tại x = 1, y = 1. Bài toán 2.19 Cháng minh rang hàm so f(x, y) = 3x3 y8 + 4x4 y9 + 5 x5y12 đạt giá trị nhỏ nhat tại c p so tự nhiên trên t p R+ × R+ .. Lài giai. Đe ý rang, f(x, y) là phân thác chính quy hai bien, và các hàm so là các phân thác thành phan là: f1(x) = 3x3 + 4x4 + 5x−5 ; f2(y) = 3x8 + 4x9 + 5x−12 . Tà đó, suy ra giá trị nhỏ nhat của f(x, y) trên t p R+ × R+ . đạt tại x = 1, y = 1.
  • 39. 33 Viết đề tài giá sinh viên – ZALO:0973.287.149-TEAMLUANVAN.COM n n 12 12 0 2 22 2n 2 2 22 2n P(x) = Σ Ck (2x)k = Σ Ck xk . Chương 3 M t so dạng toán liên quan 3.1. M t so dạng toán cfic trị trên t p so nguyên Trong phan này ta đi xét m®t so bat đȁng thác phân thác trên t p rời rạc với các đieu ki n ràng bu®c khác nhau. Bài toán 3.1 Cho đa thác P(x) = (1 + 2x)n = a0 + a1x + a2x2 + · · · + anxn a1 a2 an , biet a + + + · · · + = 4096. Xét dãy so (ak), k = 0, 2, . . . , n. Tìm so hạng lớn nhat của dãy so (ak). Lài giai. Ta có f 1 = a + a1 + a2 + · · · + an Khi đó, ta có ⇔ 2 = 4096 ⇔ 2 = 2 ⇔ n = 12. 12 12 Suy ra n k=0 12 k=0 ak = Ck 2k , k = 0, 1, 2, . . . , n. So hạng ak lớn nhat khi và chỉ khi ak ≥ ak+1 ak ≥ ak−1 ( 0
  • 40. 34 Viết đề tài giá sinh viên – ZALO:0973.287.149-TEAMLUANVAN.COM − ≥ 2 Σ N Ck 2k ≥ C 2k+1 k!(12 − k)! (k + 1)!(11 − k)! Vì k ∈ Z nên k = 8. V y a8 = 28 C8 Σ n i i+1 4 , ta có Cn−1 < Cn , n n−2 3 i=1 i=1 i=k 2 2 N N 0≤n≤N N N 2 ( k+1 12! 2k ≥ 12! 2k+1 ⇔ Ck 2k ≥ Ck−1 2k−1 ⇔ 12! k 12! k−1 12 12 2 k!(12 − k)! 2 (k − 1)!(13 − k)! ⇔ 1 12 k 2 2 ≥ k + 1 1 k 22 ⇔ 26 k ≥ 13 − k k ≤ 3 dãy (ak) Bài toán 3.2 Cho so nguyên dương n > 1 và so thực p > 0. Xét dãy các so {x1, . . . , xn} không âm thỏa mãn đieu ki n n Σ −1 i=1 xi = p. Tìm giá trị Lài giai. Đ t xk= max{x1, x2, . . . , xn}. Khi đó n−1 k−1 n−1 Σ xixi+1 = Σ xixi+1 + Σ xixi+1 Dau đȁng thác xảy ra khi và chỉ khi có hai so của dãy bang p , các so khác đeu bang 0. V y n−1 p2 max x x = . Bài toán 3.3 Cho so nguyên dương N. Tìm giá trị lớn nhat trong dãy so Cn (n = 0, 1, . . . , N). Lài giai. Do Cn = CN −n nên ta chỉ can xét dãy {Cn } với n = N N N N N nên max {Cn } = C , p = N . p Bài toán 3.4 (Taiwan TST 2014) Cho đa thác với h so thực P(x) = xn + an−2xn−2 + an−3xn−3 + · · · + a1x + a0, n ≥ 2. Giả sả xk nghi m thực của phương trình P(x) = 0,k = 1, 2, . . . , n. Cháng minh rang |x | ≤ r 2(1 − n) a . i=1 i=1 = 126720 là so hạng lớn nhat của 12 ≥ 12 12 lớn nhat của bieu thác M = xixi+1. 0, 1, . . . , . M t khác thì với n = 0, 1, . . . , k
  • 41. 35 Viết đề tài giá sinh viên – ZALO:0973.287.149-TEAMLUANVAN.COM Σ Σ x i i n n i Σ x2 > n.( n − 1 ) Σ x2 ⇔ Σ x2 > (n − 1) Σ x2 1 + 1 + 1 2 a(b + c) + b(c + a)c(a + b) i Lài giai. Theo định lý Viète, ta có n i=1 xi = 0 Ta có 1≤i Σ <j≤n Σ Σ xixj = an−2. Σ n !2 suy ra 2 xixj + 1≤i<j≤n i=1 x2 = xi i=1 = 0, Ta giả sả an−2 = −1 n 2 i i=1 n x2 > 2(1 − n) a = n − 1 Σ x2 , ∀i = 1, 2, . . . , n. Lay tőng, ta thu được n n i i=1 n n i i=1 n i i=1 n i i=1 ⇔ (2 − n) Σ x2 > 0 ⇔ n < 2, i=1 mâu thuan với giả thiet. Ta có ngay đieu phải cháng minh. Bài toán 3.5 (IMO 1995) Cho a, b, c là các so hǎu t dương thỏa mãn đieu ki n abc = 1. Cháng minh rang 1 1 1 3 a3(b + c) + b3(c + a) + c3(a + b) ≥ 2 . Lài giai. Áp dụng bat đȁng thác AM-GM, ta có 1 1 1 a3(b + c) + b3(c + a) + c3(a + b) ≥ a b c i=1 n n−2 ! . 2
  • 42. 36 Viết đề tài giá sinh viên – ZALO:0973.287.149-TEAMLUANVAN.COM ≥ − b − c − 2 1 2 (ab + bc + ca)2 = . a(b + c) + b(c + a) + c(a + b) Ta can cháng minh bat đȁng thác (ab + bc + ca)2 3 [a(b + c) + b(c + a) + c(a + b)] 2 ⇔ ab + bc + ca ≥ 3 (luôn đúng theo AM-GM). Dau bang xảy ra khi và chỉ khi a = b = c = 1. Bài toán 3.6 (China Northern Mathematical Olympiad 2007) Cho a, b, c là các so hǎu t dương thỏa mãn đieu ki n abc = 1. Cháng minh rang với moi so nguyên k ≥ 2 thì ak bk ck 3 Lài giai. Ta có a + b + b + c + c + a ≥ 2 . ak bk ck 3 a + b + b + c + c + a ≥ 2 ⇔ ak−1 + bk−1 + ck−1 ≥ 3 ak−1b + + 2 a + b bk−1c + b + c ck−1a . c + a Áp dụng bat đȁng thác AM-GM, ta có a + b ≥ 2 √ ab, b + c ≥ 2 √ bc, c + a ≥ 2 √ ca. Vì v y phải cháng minh: 3 k 1 k a 2 2 + b 3 1 k 2 2 + c 3 2a 1 + 3 ≤ 2(ak−1 + bk−1 + ck−1 ). Theo bat đȁng thác AM-GM ak−1 + bk−1 + ck−1 ≥ 3 √ 3 ak−1bk−1ck−1 = 3 và k (2k − 3)ak−1 + bk−1 ≥ (2k − 2)a 3 2b2 3 (2k − 3)bk−1 + ck−1 ≥ (2k − 2)b k− 2c 1 −
  • 43. 37 Viết đề tài giá sinh viên – ZALO:0973.287.149-TEAMLUANVAN.COM 2a2 . a b c b c a a b c + b + c a3 + b3 + c3 + b + c a3 + b3 + 3 (2k − 3)ck−1 + ak−1 ≥ (2k − 2)c k− 1 C®ng ba bat đȁng thác trên ta được đieu phải cháng minh. Dau đȁng thác xảy ra khi a = b = c = 1. Bài toán 3.7 (Brazilian Olympiad Revenge 2007) Cho a, b, c là các so hǎu t dương thỏa mãn đieu ki n abc = 1. Cháng minh rang a2 +b2 +c2 + 1 1 1 a+b+c 1 1 1 ≥ 6+2 a b c b c a . a2 + b2 + c2 +2 Lài giai. Do abc = 1, ta có + + + a b c + + + + + b c a a b c a2 + b2 + c2 + 2 1 + 1 + 1 = a2 + b2 + c2 + 2(ab + bc + ca) = (a + b + c)2 1 1 a2 + b2 1 + c2 + 2(a + b + c) = a2 b2 + b2 c2 + c2 a2 + 2abc(a + b + c) = (ab + bc + ca)2 . Lại có: 2 a + b + c + b + c + a + 3 2(ab(a + b) + bc(b + c) + ca(c + a) + 3abc) = abc = 2(a + b + c)(ab + bc + ca). Áp dụng bat đȁng thác AM-GM ta được (a+b+c)2 +(ab+bc+ca)2 ≥ 2(a+b+c)(ab+bc+ca) (đieu phải cháng minh). Dau đȁng thác xảy ra khi a = b = c = 1. Bài toán 3.8 (Hongkong 2000) Với các so hǎu t dương a, b, c thỏa mãn đieu ki n abc = 1. Cháng minh rang 1 + ab2 c3 + 1 + bc2 a3 + 1 + ca2 b3 ≥ 18 a3 + b3 + c3 . Lài giai. Bat đȁng thác can cháng minh tương đương với bat đȁng thác 1 1 1 bc2 ca 2 ab2 18 a3 + b3 + c3 + a3 + b3 + c3 ≥ a3 + b3 + c3 3 3 3 1 1 1 3 3 3 bc2 ca 2 ab2 c3 ⇔ (a ) + (a ) ≥ 18
  • 44. 38 Viết đề tài giá sinh viên – ZALO:0973.287.149-TEAMLUANVAN.COM c3 √ √ √ + b + c a3 + 3 + ≥ 3 3 = 9 2 2 2 2 4x2 4y 2 4z2 Áp dụng bat đȁng thác AM-GM, ta có (a3 + b3 + c3 ) 1 a3 1 + b3 + 1 ≥ 3 √ 3 a3b3c3 r 3 1 a3b3c3 = 9 3 3 3 bc2 ca2 ab2 √ 3 r 3 a3 b3 c3 C®ng ve với ve ta có đieu phải cháng minh. Đȁng thác xảy ra khi và chỉ khi a = b = c = 1. Bài toán 3.9 (APMO 2005) Với các so hǎu t dương x, y, z thỏa mãn đieu ki n xyz = 8. Cháng minh rang x2 y2 z2 4 (x3 + 1)(y3 + 1) + (y3 + 1)(z3 + 1) + (z3 + 1)(x3 + 1) ≥ 3 . Lài giai. Với moi a ≥ 0, áp dụng bat đȁng thác AM-GM, ta có √ a3 + 1 = √ (a + 1)(a2 − a + 1) ≤ a + 1 + a2 − a + 1 2 a2 + 2 = . 2 Áp dụng bat đȁng thác này, ta có x2 y2 z2 √ (x3 + 1)(y3 + 1) + √ (y3 + 1)(z3 + 1) + √ (z3 + 1)(x3 + 1) ≥ (x2 + 2)(y2 + 2) + (y2 + 2)(z2 + 2) + (z2 + 2)(x2 + 2) . Ta phải cháng minh 4x2 4y2 4z2 4 (x2 + 2)(y2 + 2) + (y2 + 2)(z2 + 2) + (z2 + 2)(x2 + 2) ≥ 3 ⇔ x (z + 2) + y2 (x2 + 2) + z2 (y2 + 2) ≥ 1 (x2 3 + 2)(y2 + 2)(z2 + 2) ⇔ x y + y2 z2 + z2 x2 + 2(x2 + y2 + z2 ) ≥ x y z + 8 = 72. Bat đȁng thác trên luôn đúng do x2 y2 +y2 z2 +z2 x2 +2(x2 +y2 +z2 ) ≥ 3 √ 3 x4y4z4 +2.3 √ 3 x2y2zz = 48 + 24. Đȁng thác xảy ra khi và chỉ khi x = y = z = 2. 3 c b (a ) a3b3c3 a3b3c 2 2 2
  • 45. 39 Viết đề tài giá sinh viên – ZALO:0973.287.149-TEAMLUANVAN.COM 2011 Σ − A = (k − n x ) C x (1 − x) − n−1 n−2 Σ = C x (1 − x) = [x + ( 1− x)] = 1 = (nx)2 Σ Ck xk (1 − x)n−k + Σ k2 Ck xk (1 − x)n−k − 2nx Σ (k − nx)2 Ck x1 − xk A1 = Σ Ck xk (1 − x)n−k = Σ Ck xk (1 − x)n−k n−1 n−1 = nx Σ Ck−1 xk−1 (1 − x)n−k + n Σ k − 1Ck−1 xk (1 − x)n−k A − k=1 k=1 3.2. M t so dạng toán ve so tfi nhiên tfi các đe thi Olympic Ta xét m®t so bài toán liên quan đen công thưsc tő hợp tà các đe thi Olympic. Bài toán 3.10 (Olympic 30/4/2011) Cho hàm so 2011 F(x) = Σ (k − 2011x)2 Ck xk (1 − x)2011−k . k=0 Tìm giá trị nhỏ nhat của hàm so F(x) trên [0; 1] n 2 k k n k n k=0 n n n Xét n k=0 n n k=0 n n k=0 n k=0n n k=1 n = n Σ Ck−1 xk (1 − x)n−k = nx Σ Ck−1 xk−1 (1 − x)n−k = nx[x + (1 x)]n−1 = nx n n A = Σ k2 Ck xk (1 − x) n−k = Σ k2 Ck xk (1 − x)n−k 2 n k=0n n k=1 = n Σ kCk−1 xk (1 − x)n−k k=1 n n k=1 n−1 n k=1 n−1 = nx + n(n − 1) Σ Ck−2 xk (1 − x)n−k = nx + n(n − 1)x2 k=2 n k k n k n 3 n k=0 V y A = (nx)2 + nx + n(n − 1)x2 − 2(nx)2 = nx(1 − x).
  • 46. 40 Viết đề tài giá sinh viên – ZALO:0973.287.149-TEAMLUANVAN.COM C n C Σ n+4 2(n + 3)(n + 4) n n ta có f(x) ≤ 2011 Dau đȁng thác xảy ra khi x = . Áp dụng ket quả trên ta thu được f(x) = 2011x(1 − x). Do x ∈ [0; 1] nên x ≥ 0, 1 − x ≥ 0. Áp dụng bat đȁng thác AM - GM, V y max f(x) = [0;1] 2011 1 4 ⇔ x = 2 Nh n xét 3.1 Ta có the thay so 2011 trong hàm so f(x) bởi m®t so thực dương bat kì. Xuat phát tà cách viet tőng Σn (−1)k k dưới dạng phân thác p(n) q(n) k=0 k3 + 9k2 + 26k + 24 , trong đó p(n), q(n) là các đa thác với h so nguyên, ta có the xây dựng được bài toán cực trị liên quan đen tő hợp là bài toán sau: Bài toán 3.11 Tìm giá trị nhỏ nhat bieu thác n k k 4 3 2 f(n) = Σ (−1) Cn + 2n − 10n − 64n + 272n + 959 . Lài giai. Đ t S(n) = Σ (−1)k k Ta có k=0 k3 + 9k2 + 26k + 24 nên k3 + 9k2 + 26k + 24 = (k + 2)(k + 3)(k + 4) n k S(n) = (−1) .n! k=0 k!.(n − k)!(k + 2)(k + 3)(k + 4) Đ t = k=0 (−1)n !.(n + 4)! (k + 4)!.(n − k)! k + 1 (n + 1)(n + 2)(n + 3)(n + 4) n T(n) = (n + 1)(n + 2)(n + 3)(n + 4)S(n) = Σ Ck+4 (k + 1). k=0 n k=0 x + (1 − x) 2 1 2 k3 + 9k2 + 26k + 24 Σ
  • 47. 41 Viết đề tài giá sinh viên – ZALO:0973.287.149-TEAMLUANVAN.COM Σ (−1) C Σ = (x − 1) = 0 ⇒ −n (−1) C Σ − Σ = (−1) nC Σ = n (−1) C Σ = −n (−1) nC n+4 Σ = (−1) C (k + 1) n Σ C Σ = C Σ j − 3 (−1) C Σ (−1)i Ci .i = Σ (−1)i i.n! + (−1)0 0.n! n 1 = 0 j Ta có và n i i n i=0 n i j j n−1 j=0 n n i=0 n i=1 n i!(n − i)! 0!n! n = ( 1)i n! i=1 (i − 1)!(n − i)! i i−1 n−1 i=1 Do đó n n i i−1 n−1 i=1 n i−1 i−1 − i=1 T(n) = Σ (−1)k Ck+4 (k + 1) k=0 n k+4 k+4 n+4 k=0 = Σ (−1)k+4 Ck+4 (k + 1) − (−3 + 2(n + 4) − C2 ) k=−4 n+4 = n+4 n+4 (j − 3) − 2n + 8 − 3 − n+4 (n + 4)(n + 3) 2 j=0 n+4 j n+4 j=0 n+4 j j n+4 j=0 1 2 — 2 (4n + 10 − n − 7n − 12) Suy ra = 0 + 0 + 1 (n2 2 + 3n + 2) = 1 (n + 1)(n + 2) 2 T(n) S(n) = = dfrac(n + 1)(n + 2)(n + 1)(n + 2)(n + 3)(n + V y (n + 1)(n + 2)(n + 3)(n + 4) 1 S(n) = Do đó f(n) = 1 2(n + 3)(n + 4) 2(n + 3)(n + 4) 2n4 − 10n3 − 64n2 + 272n + 959 2(n + 3)(n + 4) + = 0
  • 48. 42 Viết đề tài giá sinh viên – ZALO:0973.287.149-TEAMLUANVAN.COM − ≥ 2n4 − 10n3 − 64n2 + 272n + 960 2(n + 3)(n + 4) 2(n + 3)(n + 4)(n2 12n + 40) = 2(n + 3)(n + 4) Ta có = n2 − 12n + 40 n2 −12n+40 = (n−6)2 +4 ≥ 4, ∀n ∈ N. Dau đȁng thác xảy ra khi n = 6 V y f(n) đạt giá trị nhỏ nhat bang 4 khi n = 6. Bài toán 3.12 (Bulgari TST 2003) Với ba so nguyên dương a, b, c thỏa mãn đieu ki n a + b + c = 3. Cháng minh rang a b c 3 1 + b2 + 1 + c2 + 1 + a2 ≥ 2 . Lài giai. Ta không the dùng trực tiep bat đȁng thác AM-GM với mau so vì bat đȁng thác sau đó sě đői chieu. Tuy nhiên ta có the dùng lại bat đȁng thác đó theo cách khác. Áp dụng bat đȁng thác AM-GM, ta có a ab2 ab2 ab 1 + b2 = a − 1 + b2 ≥ a − 2b = a − 2 . Tương tự, ta có b bc 1 + c2 ≥ b − 2 c ac 1 + a2 ≥ c − 2 . C®ng ve với ve ba bat đȁng thác trên ta có a b c 1 ab + bc + ca 1 + b2 + 1 + c2 + 1 + a2 ≥ a+b+c− 2 (ab+bc+ca) = 3− 2 . Ta lại có Do đó a b ab + bc + ca ≤ c (a + b + c)2 = 3. 3 1 ab + bc + ca 3 1 + b2 + 1 + c2 + 1 + a2 ≥ a+b+c− 2 (ab+bc+ca) = 3− 2 2 . Đȁng thác xảy ra khi và chỉ khi a = b = c = 1. =
  • 49. 43 Viết đề tài giá sinh viên – ZALO:0973.287.149-TEAMLUANVAN.COM 1 Bài toán 3.13 (Olympic 30/4/2009) Với moi b® n so nguyên dương a1, a2, . . . , an có tőng bang 2009. Đ t A là tőng tat cả các so hạng có dạng 1 a1(a1 + a2)(a1 + a2 + a3) . . . (a1 + a2 + · · · + an) trong đó k1, k2, . . . , kn là m®t hoán vị bat kì của {1, 2, . . . , n} . Tìm giá trị nhỏ nhat Lài giai. Xét m®t b® bat kì n so nguyên dương a1, a2, . . . , an có tőng bang 2009 Ta cháng minh bang quy nạp +) Với n = 1, (3.1)đúng 1 A = a1a2 . . . an (3.1) +) Giả sả (3.1)đúng đen n − 1 Với moi n so nguyên dương a1, a2, . . . , an 1 Trong A, đ t a1 + a2 + · · · + an A = làm thàa so chung, ta được 1 , trong đó B gom n! so hạng. a1 + a2 + · · · + an Trong B ta nhóm các so hạng không cháa a1 thành tőng B1 gom (n−1)! so hạng áng với (n − 1)! hoán vị của {2, 3 . . . , n} Do đó theo giả thiet quy nạp thì B = a2a3 . . . an Tiep tục như the với a2, a3, . . . , an ta được 1 1 1 B = + + · · · + = a1 + a2 · · · + an a2a3 . . . an Tà đó suy ra A = a1a3 . . . an 1 a1a2 . . . an a1a2 . . . an−1 a1a2 . . . an V y theo nguyên lí quy nạp (3.1) đúng với moi n ≥ 1. +) Neu a1 = 1, loại a1 = 1, the a2 = 1 bởi a′ 2 = 1 + a2 thì tőng các so ai không đői, còn tích tăng lên. +) Neu ai ≥ 5 the a1 bởi 2(a1 − 2) thì tőng các so ai không đői, tích tăng lên vì 2(a1 − 2) = 2a1 − 4 > a1 +) Neu có m®t so ai = 4, the so đó bởi 2 + 2 thì tőng và tích các so ai không đői
  • 50. 44 Viết đề tài giá sinh viên – ZALO:0973.287.149-TEAMLUANVAN.COM +) Neu có ba so 2, the ba so đó bang hai so 3 thì tőng không đői, tích tăng lên V y đe tích p của các so ai lớn nhat thì phải chon không quá hai so 2, các so khác bang 3 Do 2009 = 3.669 + 2 nên maxP = 2.3669 . Suy ra 1 minA = 2.3669 .
  • 51. 45 Viết đề tài giá sinh viên – ZALO:0973.287.149-TEAMLUANVAN.COM Ket lu n Lu n văn “M®t so phương pháp tìm cực trị của các hàm phân thác sinh bởi so tự nhiên” đã trình bày nhǎng van đe sau: - Lu n văn trình bày chi tiet các tính chat cơ bản của đa thác với h so nguyên, phân thác hǎu t với h so nguyên, các đong nhat thác và bat đȁng thác dạng phân thác hǎu t với h so nguyên. - Tiep theo trình bày chi tiet các dạng toán tìm cực trị trong lớp các hàm phân thác. - Cuoi cùng, lu n văn trình bày các dạng toán liên quan được chon loc tà các đe thi HSG quoc gia, Olympic khu vực và quoc te.
  • 52. 46 Viết đề tài giá sinh viên – ZALO:0973.287.149-TEAMLUANVAN.COM Tài li u tham khảo [A] Tieng Vi t [1] Nguyen Văn M u (2002), Đa thúc đại so và phân thúc hũu ty, NXB Giáo dục. [2] Nguyen Văn M u (2016), N®i suy đa thúc, NXB ĐHQG Hà N®i. [3] Nguyen Văn M u, Trịnh Đào Chien, Tran Nam Dũng, Nguyen Đăng Phat (2008), Chuyên đe chon loc ve đa thúc và áp dựng, NXB Giáo dục. [4] Nguyen Văn Nho, OLYMPIC toán hoc Châu Á Thái Bình Dương, NXB Giáo dục. [B] Tieng Anh [5] Paulo Ney de Sausa, Jorge- Nume Silva (1998), Berkeley Problems in Mathematics, Springer. [6] Pólya G., Szgo G., 1956, Problems and theorems from Analysis, Moscow. [7] Radulescu T-L.T. , Radulescu V.D., Andreescu T. (2009). Problems in Real Analysis: Advanced calculus on the real axis. Springer. [8] Titu Andreescu (2004), Mathematical Olympiad Treasures, Birkhauser Boston, USA.
  • 53. ĐẠI HOC THÁI NGUYÊN TRƯ NG ĐẠI HOC KHOA HOC Viết đề tài giá sinh viên – ZALO:0973.287.149-TEAMLUANVAN.COM NGUYEN KIM THU M T SO PHƯƠNG PHÁP TÌM CỰC TR± CỦA CÁC HÀM PHÂN THỨC SINH B I SO TỰ NHIÊN LU N VĂN THẠC SĨ TOÁN HOC THÁI NGUYÊN - 2018
  • 54. ĐẠI HOC THÁI NGUYÊN TRƯ NG ĐẠI HOC KHOA HOC Viết đề tài giá sinh viên – ZALO:0973.287.149-TEAMLUANVAN.COM NGUYEN KIM THU M T SO PHƯƠNG PHÁP TÌM CỰC TR± CỦA CÁC HÀM PHÂN THỨC SINH B I SO TỰ NHIÊN Chuyên ngành: PHƯƠNG PHÁP TOÁN SƠ CAP Mã so: 8 46 01 13 LU N VĂN THẠC SĨ TOÁN HOC Người hướng dan khoa hoc: GS.TSKH. Nguyen Văn M u THÁI NGUYÊN - 2018